TEST BANK For Analysis for Financial Management, 12e Robert C. Higgins

Page 1


Chapter 01 Test Bank 1. Current liabilities are defined as liabilities with a maturity of less than one year. TRUE Accessibility: Keyboard Navigation Difficulty: 1 Easy Gradable: automatic

2. A decline in the Net fixed assets account between year-end 2016 and year-end 2017 is a clear indication that fixed assets were sold during 2017. FALSE Accessibility: Keyboard Navigation Difficulty: 2 Medium Gradable: automatic

3. When reporting financial performance for tax purposes, U.S. companies prefer to use accelerated depreciation methods over the straight-line method. TRUE Accessibility: Keyboard Navigation Difficulty: 2 Medium Gradable: automatic

4. Accounting rules require U.S. companies to depreciate research and development (R&D) expenditures using the straight-line method. FALSE Accessibility: Keyboard Navigation Difficulty: 1 Easy Gradable: automatic

5. You can construct a sources and uses statement for 2017 if you have a company’s year-end balance sheets for 2017 and 2018. FALSE Accessibility: Keyboard Navigation Difficulty: 1 Easy Gradable: automatic

6. A reduction in long-term debt is a use of cash. TRUE Accessibility: Keyboard Navigation Difficulty: 1 Easy Gradable: automatics

7. The accrual principle requires that revenue not be recognized until payment from a sale is received. FALSE Accessibility: Keyboard Navigation Difficulty: 1 Easy Gradable: automatic

1-1


8. An increase in cash and cash equivalents should appear as a source of cash on the sources and uses statement. FALSE Accessibility: Keyboard Navigation Difficulty: 2 Medium Gradable: automatic

9. A cash flow statement places each source or use of cash into one of three broad categories: operating activities, investing activities, or financing activities. TRUE Accessibility: Keyboard Navigation Difficulty: 1 Easy Gradable: automatic

10. The cost of equity is usually reported on the income statement right below interest expense. FALSE Accessibility: Keyboard Navigation Difficulty: 1 Easy Gradable: automatic

11. Which of the following statements concerning the cash flow production cycle is true? A. The profits reported in a given time period equal the cash flows generated. B. A company’s operations and finances are independent of each other. C. Financial statements have nothing to do with reality. D. The movement of cash to inventory, to accounts receivable, and back to cash is known as the firm’s working capital cycle. E. A profitable company will always have sufficient cash to meet its obligations. Accessibility: Keyboard Navigation Difficulty: 2 Medium Gradable: automatic

12. Which of the following statements concerning a firm’s cash flows and profits is false? A. Managers must be at least as concerned with cash flows as with profits. B. A company that sells merchandise at a profit will generate cash soon enough to replenish cash flows required for continued production. C. The cash flows generated in a given time period can differ from the profits reported. D. Profits are no assurance that cash flow will be sufficient to maintain solvency. E. Due to required cash investments in current assets, fast-growing and profitable companies can literally "grow broke". Accessibility: Keyboard Navigation Difficulty: 2 Medium Gradable: automatic

13. Which of the following is NOT a typical reason for differences between profits and cash flow? A. Goodwill B. Depreciation expense C. Changes in accounts receivable D. Accrual accounting practices Accessibility: Keyboard Navigation Difficulty: 2 Medium Gradable: automatic

1-2 Copyright © 2018 McGraw-Hill Education. All rights reserved. No reproduction or distribution without the prior written consent of McGraw-Hill Education.


14. Which one of the following is the financial statement that shows a financial snapshot, taken at a point in time, of all the assets the company owns and all the claims against those assets? A. income statement B. creditor’s statement C. balance sheet D. cash flow statement E. sources and uses statement Accessibility: Keyboard Navigation Difficulty: 1 Easy Gradable: automatic

15. A balance sheet reports the value of a firm’s assets, liabilities, and equity A. over an annual period. B. over any period of time. C. at any point in time. D. at the end of the year. Accessibility: Keyboard Navigation Difficulty: 1 Easy Gradable: automatic

16. A company sells used equipment with a book value of $100,000 for $250,000 cash. How would this transaction affect the company’s balance sheet? A. Equity rises $250,000; net plant and equipment falls $250,000. B. Cash rises $250,000; net plant and equipment falls $100,000; equity rises $150,000. C. Cash rises $250,000; accounts receivable falls $100,000; goodwill rises $150,000. D. Cash rises $250,000; net plant and equipment falls $250,000. Accessibility: Keyboard Navigation Difficulty: 2 Medium Gradable: automatic

17. A company purchases a new $10 million building financed half with cash and half with a bank loan. How would this transaction affect the company’s balance sheet? A. Net plant and equipment rises $10 million; cash falls $10 million; bank debt rises $5 million. B. Net plant and equipment rises $5 million; cash falls $10 million; bank debt rises $5 million. C. Net plant and equipment rises $5 million; cash falls $5 million; bank debt rises $5 million. D. Net plant and equipment rises $10 million; cash falls $5 million; bank debt rises $5 million. Accessibility: Keyboard Navigation Difficulty: 2 Medium Gradable: automatic

18. Which one of the following is the financial statement that summarizes a firm’s revenue and expenses over a period of time? A. income statement B. balance sheet C. cash flow statement D. sources and uses statement E. market value statement Accessibility: Keyboard Navigation Difficulty: 1 Easy Gradable: automatic

1-3 Copyright © 2018 McGraw-Hill Education. All rights reserved. No reproduction or distribution without the prior written consent of McGraw-Hill Education.


19. The sources and uses of cash over a stated period of time are reflected in the A. income statement. B. balance sheet. C. shareholders’ equity statement. D. cash flow statement. E. statement of operating position. Accessibility: Keyboard Navigation Difficulty: 1 Easy Gradable: automatic

20. Which one of the following is a source of cash? A. increase in accounts receivable B. decrease in notes payable C. decrease in common stock D. increase in inventory E. increase in accounts payable Accessibility: Keyboard Navigation Difficulty: 1 Easy Gradable: automatic

21. Which one of the following is a use of cash? A. increase in notes payable B. increase in inventory C. increase in long-term debt D. decrease in accounts receivable E. increase in common stock Accessibility: Keyboard Navigation Difficulty: 1 Easy Gradable: automatic

22. Which one of the following is a source of cash? A. decrease in accounts receivable B. decrease in common stock C. decrease in long-term debt D. decrease in accounts payable E. increase in inventory Accessibility: Keyboard Navigation Difficulty: 1 Easy Gradable: automatic

23. Which of the following would NOT be considered a use of cash? A. Dividends paid B. A decrease in accounts payable C. Depreciation D. An increase in the cash and marketable securities account Accessibility: Keyboard Navigation Difficulty: 2 Medium Gradable: automatic

1-4 Copyright © 2018 McGraw-Hill Education. All rights reserved. No reproduction or distribution without the prior written consent of McGraw-Hill Education.


24. Which one of the following is the financial statement that summarizes changes in the company’s cash balance over a period of time? A. income statement B. balance sheet C. cash flow statement D. shareholders’ equity statement E. market value statement Accessibility: Keyboard Navigation Difficulty: 1 Easy Gradable: automatic

25. Which of the following is NOT a major category on the cash flow statement? A. Cash flows from selling activities B. Cash flows from operating activities C. Cash flows from financing activities D. Cash flows from investing activities

Accessibility: Keyboard Navigation Difficulty: 1 Easy Gradable: automatic

26. The book value of a firm is A. equivalent to the firm’s market value provided that the firm has some fixed assets. B. based on historical cost. C. generally greater than the market value when fixed assets are included. D. more of a financial than an accounting valuation. E. adjusted to the market value whenever the market value exceeds the stated book value. Accessibility: Keyboard Navigation Difficulty: 1 Easy Gradable: automatic

27. Which of the following is a reason why a company’s market value of equity differs from its book value of equity? A. Shareholders are keenly aware of book values but have little interest in market values. B. Accountants’ charges for the cost of equity are often higher than they should be. C. Fair value accounting is becoming more widely used. D. Values of assets on the balance sheet typically reflect historical cost, adjusted for appropriate depreciation. Accessibility: Keyboard Navigation Difficulty: 2 Medium Gradable: automatic

28. Depreciation expense A. reduces both taxes and net income. B. increases net fixed assets as shown on the balance sheet. C. is a noncash item that increases net income. D. decreases current assets, net income, and operating cash flows. Accessibility: Keyboard Navigation Difficulty: 1 Easy Gradable: automatic

1-5 Copyright © 2018 McGraw-Hill Education. All rights reserved. No reproduction or distribution without the prior written consent of McGraw-Hill Education.


[The following information applies to the questions displayed below.] Foodtek, Inc. Selected financial information ($ millions) 2016 2017 Net sales $296 $364 Cost of goods sold 168 223 Depreciation 51 65 Net income 32 45 Finished goods inventory 34 29 Accounts receivable 47 87 Accounts payable 39 44 Net fixed assets 310 415 Year-end cash balance

$186 $123

29. Please refer to the financial information for Foodtek, Inc. above. During 2017, how much cash (in millions of dollars) did Foodtek collect from sales? A. 364 B. 277 C. 404 D. 324 E. 451 F. None of the options are correct. In 2017, sales were $364 million, but account receivable rose $40 million, indicating that the company only received $324 million in cash. (This ignores possible changes in bad debt reserves.) Letting bop stand for beginning of period, and eop for end of period, the equation is (where AR = accounts receivable) AReop = ARbop + Sales − Collections Collections = Sales − Change in AR $324 = $364 − $40 Accessibility: Keyboard Navigation Difficulty: 2 Medium Gradable: automatic

30. Please refer to the financial information for Foodtek, Inc. above. During 2017, what was the cost of merchandise (in millions of dollars) produced by Foodtek? A. 223 B. 194 C. 252 D. 228 E. 218 F. None of the options are correct. During 2017, the company sold $223 million of merchandise at cost, but finished goods inventory fell $5 million, indicating that the company only produced $218 million of merchandise. Letting bop stand for beginning of period, and eop for end of period, the equation is Inventoryeop = Inventorybop+ Production − Cost of sales Production = Cost of sales + Change in Inventory $218 = $223 − $5 Accessibility: Keyboard Navigation Difficulty: 2 Medium Gradable: automatic

1-6 Copyright © 2018 McGraw-Hill Education. All rights reserved. No reproduction or distribution without the prior written consent of McGraw-Hill Education.


31. Please refer to the financial information for Foodtek, Inc. above. Assuming the company neither sold nor salvaged any assets during the year, what were Foodtek’s capital expenditures (in millions of dollars) during 2017? A. 415 B. 105 C. 310 D. 40 E. 170 F. None of the options are correct. Net fixed assets rose $105 million, depreciation reduced net fixed assets by $65 million, so capital expenditures must have been $170 million (ignoring asset sales or write-offs). Letting bop stand for beginning of period, and eop for end of period, the equation is Net fixed assetseop = Net fixed assetsbop + Capital Expenditures - Depreciation Capital Expenditures = Change in net fixed assets + Depreciation $170 = $105 + $65 Accessibility: Keyboard Navigation Difficulty: 2 Medium Gradable: automatic

32. Please refer to the financial information for Foodtek, Inc. above. Assuming that there were no financing cash flows during 2017 and basing your answer solely on the information provided, what were Foodtek’s cash flows from operations (in millions of dollars) for 2017? A. 45 B. 110 C. 70 D. 80 E. 35 F. None of the options are correct. Cash flow from operations can be calculated directly from items in the table. Start with net income, remove any noncash items (such as depreciation), and add any cash transactions that are not captured by the income statement (such as changes to working capital accounts). CF from operations = Net income - increase in acct. receivable - increase in inventory + increase in acct. payable + depreciation CF from operations = 45 - 40 + 5 + 5 + 65 = $80 million Accessibility: Keyboard Navigation Difficulty: 3 Hard Gradable: automatic

33. Suppose an acquiring firm pays $100 million for a target firm, and the target’s assets have a book value of $70 million and an estimated replacement value of $80 million. What amount would be allocated to the acquiring firm’s goodwill account? A. $0 million B. $20 million C. $30 million D. $70 million E. $80 million F. None of the options are correct. Accessibility: Keyboard Navigation Difficulty: 1 Easy Gradable: automatic

1-7 Copyright © 2018 McGraw-Hill Education. All rights reserved. No reproduction or distribution without the prior written consent of McGraw-Hill Education.


34. JM Case Inc. has a market value of $5 million with 500,000 shares outstanding. The book value of its equity is $1,750,000. What is JM Case’s price per share? A. $3.50 B. $5 C. $10 D. $25 E. $50 F. None of the options are correct. Stock price per share = $5 million/500,000 shares = $10 per share Accessibility: Keyboard Navigation Difficulty: 1 Easy Gradable: automatic

35. JM Case Inc. has a market value of $5 million with 500,000 shares outstanding. The book value of its equity is $1,750,000. What is JM Case’s book value per share? A. $3.50 B. $5 C. $10 D. $25 E. $50 F. None of the options are correct. Book value per share = $1,750,000/500,000 shares = $3.50 per share Accessibility: Keyboard Navigation Difficulty: 1 Easy Gradable: automatic

36. JM Case Inc. has a market value of $5 million with 500,000 shares outstanding. The book value of its equity is $1,750,000. If the company repurchases 20 percent of its shares in the stock market, what will be the book value of equity if all else remains the same? A. $750,000 B. $1,250,000 C. $1,000,000 D. $1,400,000 E. $4,000,000 F. None of the options are correct. JM Case will pay $10 per share for the 100,000 shares (= 0.20 × 500,000) it repurchases. This reduces the book value by $1 million. Assuming all else remains the same, the new book value should be $750,000. Accessibility: Keyboard Navigation Difficulty: 2 Medium Gradable: automatic

1-8 Copyright © 2018 McGraw-Hill Education. All rights reserved. No reproduction or distribution without the prior written consent of McGraw-Hill Education.


37. JM Case Inc. has a market value of $5 million with 500,000 shares outstanding. The book value of its equity is $1,750,000. If the company repurchases 20 percent of its shares in the stock market, there are no taxes or transaction costs, and all else remains the same, what should the market value of the firm be after the repurchase? A. $1,000,000 B. $1,750,000 C. $3,250,000 D. $4,000,000 E. $5,000,000 F. None of the options are correct. Since nothing else has changed, the market value should fall by exactly the amount of the cash paid in the transaction. The new market value will be $4 million. Another way to think about this is to note that repurchase of the shares will reduce cash by $1 million (or increase liabilities by the same amount if financed with debt), and thus the firm is worth $1 million less to the owners after the repurchase, or $4 million. After repurchasing 100,000 shares (= 0.20 × 500,000), 400,000 shares will be outstanding, and the price per share remains $10 ($4 million/400,000). Accessibility: Keyboard Navigation Difficulty: 2 Medium Gradable: automatic

38. ZZZ Corporation’s income statement shows a provision for income taxes of $65 million in 2017. At the end of 2016, ZZZ’s balance sheet reported income taxes payable of $12 million and deferred taxes of $18 million. At the end of 2017, their balance sheet shows income taxes payable of $15 million and deferred taxes of $17 million. What were ZZZ’s taxes paid in 2017? A. $61 million B. $63 million C. $65 million D. $67 million E. $69 million Taxes paid = Provision for income taxes − Increase in taxes payable - Increase in deferred taxes = 65 − 3 + 1 = $63 million Accessibility: Keyboard Navigation Difficulty: 2 Medium Gradable: automatic

39. Which of the following formulas describes the calculation of cash flow from operating activities? A. Net income + Noncash items +/− Changes in current assets and liabilities B. Net income + Capital expenditures − Dividends paid C. Net income − Capital expenditures − Dividends paid D. Net income +/− Changes in current assets and liabilities − Dividends paid E. Net income + Noncash items − Capital expenditures Accessibility: Keyboard Navigation Difficulty: 2 Medium Gradable: automatic

1-9 Copyright © 2018 McGraw-Hill Education. All rights reserved. No reproduction or distribution without the prior written consent of McGraw-Hill Education.


40. The book value of Little Statistic’s total assets is $400,000. Suppose Number Crunching Inc. acquires Little Statistic’s assets for $1 million and finances the purchase by selling $600,000 in new stock, $300,000 in new debt, and reducing cash by $100,000. Describe how the acquisition affects Number Crunching’s balance sheet. First, let us account for Number Crunching’s $1 million expenditure. Cash will fall $100,000, liabilities will rise $300,000, and owners’ equity will rise $600,000. Next, let us account for the assets acquired. The accountants will write up the value of fixed assets and possibly inventory to their estimated replacement value; they will then add the difference between the acquisition price and the replacement value of the assets acquired to a goodwill account appearing in the long-term assets section of Number Crunching’s balance sheet; and lastly, they will consolidate the two companies’ balance sheets by adding like accounts together. Accessibility: Keyboard Navigation Difficulty: 2 Medium Gradable: manual

41. Playdough Products earned net income of $500,000 in 2017. The firm increased its accounts receivable during the year by $220,000. The book value of its assets declined by the year’s depreciation charge, which was $140,000, and the market value of its assets increased by $50,000. Based only on this information, how much cash did Playdough Products generate during the year? Please ignore taxes for this problem. Playdough Products generated $420,000 of cash during the year. The $500,000 net income ignores the fact that accounts receivable rose $220,000, a use of cash. It also treats $140,000 depreciation as an expense, whereas it is a noncash charge. The $50,000 increase in market value of assets adds to the value of the business, but is not a cash flow. Summary: Accounting Income

$ 500,000

Depreciation, a non-cash charge +$ 140,000 Increase in Accounts Receivable−$ 220,000 Cash generated

$ 420,000 Accessibility: Keyboard Navigation Difficulty: 2 Medium Gradable: manual

42. During 2017, Lele Design earned net income of $250,000. The firm neither bought nor sold any capital assets. The book value of its assets declined by the year’s depreciation charge of $200,000. The firm’s operating cash flow for the year was $450,000. The market value of its assets increased by $300,000. Based only on this information, what was Lele Design’s economic income for the year? Why is this figure different from its accounting income? Please ignore taxes for this problem. Lele Design generated economic income equal to $750,000, comprised of $450,000 in operating cash flow ($250,000 net income plus $200,000 depreciation) plus a $300,000 increase in the market value of its assets. The $500,000 difference between economic income and accounting income consists of the $200,000 noncash charge of depreciation, and the $300,000 appreciation in the market value of assets, which accounting income does not include. Accessibility: Keyboard Navigation Difficulty: 2 Medium Gradable: manual

1-10 Copyright © 2018 McGraw-Hill Education. All rights reserved. No reproduction or distribution without the prior written consent of McGraw-Hill Education.


43. Identify the sources and uses of cash for Blackhurst Corporation for 2017 based on the following year-end balance sheets. 20162017 ASSETS Cash

40

30

Accounts receivable

100 150

Inventory

135 125

Net fixed assets

200 240 475 545

LIABILITIES & EQUITY Bank loan

40

100

Long-term debt

200 180

Equity

235 265 475 545 2016 2017SourcesUses

ASSETS Cash

40

30

Accounts receivable

100 150

Inventory

135 125

Net fixed assets

200 240

10 50 10 40

475 545 LIABILITIES & EQUITY Bank loan

40 100

Long-term debt

200 180

Equity

235 265

60 20 30

475 545 110

110 Accessibility: Keyboard Navigation Difficulty: 2 Medium Gradable: manual

1-11 Copyright © 2018 McGraw-Hill Education. All rights reserved. No reproduction or distribution without the prior written consent of McGraw-Hill Education.


Chapter 01 Test Bank Summary Category Accessibility: Keyboard Navigation Difficulty: 1 Easy Difficulty: 2 Medium Difficulty: 3 Hard Gradable: automatic Gradable: manual

# of Questions 43 21 21 1 39 4

1-12 Copyright © 2018 McGraw-Hill Education. All rights reserved. No reproduction or distribution without the prior written consent of McGraw-Hill Education.


Chapter 02 Test Bank 1. An inventory turnover ratio of 10 means that, on average, items are held in inventory for 10 days. FALSE Accessibility: Keyboard Navigation Difficulty: 1 Easy Gradable: automatic

2. All else equal, an increase in a company’s asset turnover will decrease its ROE. FALSE Accessibility: Keyboard Navigation Difficulty: 1 Easy Gradable: automatic

3. A company’s return on assets will always equal or exceed its profit margin. FALSE Accessibility: Keyboard Navigation Difficulty: 2 Medium Gradable: automatic

4. A company’s price-to-earnings ratio is always equal to one minus its earnings yield. FALSE Accessibility: Keyboard Navigation Difficulty: 1 Easy Gradable: automatic

5. Return on assets can be calculated as profit margin times asset turnover. TRUE Accessibility: Keyboard Navigation Difficulty: 1 Easy Gradable: automatic

6. All else equal, a firm would prefer to have a higher gross margin. TRUE Accessibility: Keyboard Navigation Difficulty: 1 Easy Gradable: automatic

7. The times-interest-earned ratio always equals or exceeds the times-burden-covered ratio. TRUE Accessibility: Keyboard Navigation Difficulty: 1 Easy Gradable: automatic

2-1 Copyright © 2018 McGraw-Hill Education. All rights reserved. No reproduction or distribution without the prior written consent of McGraw-Hill Education.


8. If a firm increases its accounts payable period, other things equal, it increases the cash conversion cycle. FALSE Accessibility: Keyboard Navigation Difficulty: 1 Easy Gradable: automatic

9. Across companies, ROA and financial leverage tend to be inversely related. TRUE Accessibility: Keyboard Navigation Difficulty: 2 Medium Gradable: automatic

10. One advantage of ROE is that it is a risk-adjusted measure of performance. FALSE Accessibility: Keyboard Navigation Difficulty: 1 Easy Gradable: automatic

11. The most popular yardstick of financial performance among investors and senior managers is the A. profit margin. B. return on equity. C. return on assets. D. times-burden-covered ratio. E. earnings yield. F. None of the options are correct. Accessibility: Keyboard Navigation Difficulty: 1 Easy Gradable: automatic

12. Which of these ratios, or levers of performance, are the determinants of ROE? I. profit margin II. financial leverage III. times interest earned IV. asset turnover A. I and IV only B. II and IV only C. I, II, and IV only D. I, II, and III only E. I, III, and IV only F. I, II, III, and IV Accessibility: Keyboard Navigation Difficulty: 1 Easy Gradable: automatic

13. Ratios that measure how efficiently a firm manages its assets and operations to generate net income are referred to as _____ ratios.

2-2 Copyright © 2018 McGraw-Hill Education. All rights reserved. No reproduction or distribution without the prior written consent of McGraw-Hill Education.


A. asset turnover and control B. financial leverage C. coverage D. profitability E. None of the options are correct. Accessibility: Keyboard Navigation Difficulty: 1 Easy Gradable: automatic

14. Which of the following ratios are measures of a firm’s liquidity? I. fixed asset turnover ratio II. current ratio III. debt-equity ratio IV. acid test A. I and III only B. II and IV only C. III and IV only D. I, II, and III only E. I, III, and IV only Accessibility: Keyboard Navigation Difficulty: 1 Easy Gradable: automatic

15. Ptarmigan Travelers had sales of $420,000 in 2016 and $480,000 in 2017. The firm’s current asset accounts remained constant. Given this information, which one of the following statements must be true? A. The total asset turnover rate increased. B. The days’ sales in receivables increased. C. The inventory turnover rate increased. D. The fixed asset turnover decreased. E. The collection period decreased. Accessibility: Keyboard Navigation Difficulty: 2 Medium Gradable: automatic

16. In comparison to industry averages, Okra Corp. has a low inventory turnover, a high current ratio, and an average quick ratio. Which of the following would be the most reasonable inference about Okra Corp.? A. Its current liabilities are too low. B. Its cost of goods sold is too low. C. Its cash and securities balance is too low. D. Its inventory level is too high. Accessibility: Keyboard Navigation Difficulty: 1 Easy Gradable: automatic

17. Which one of the following ratios identifies the amount of sales a firm generates for every $1 in assets? A. current ratio

2-3 Copyright © 2018 McGraw-Hill Education. All rights reserved. No reproduction or distribution without the prior written consent of McGraw-Hill Education.


B. debt-to-equity C. retention D. asset turnover E. return on assets Accessibility: Keyboard Navigation Difficulty: 1 Easy Gradable: automatic

18. A times-interest-earned ratio of 3.5 indicates that the firm A. pays 3.5 times its earnings in interest expense. B. has interest expense equal to 3.5% of EBIT. C. has interest expense equal to 3.5% of net income. D. has EBIT equal to 3.5 times its interest expense. Accessibility: Keyboard Navigation Difficulty: 1 Easy Gradable: automatic

19. At the end of 2017, Stacky Corp. had $500,000 in liabilities and a debt-to-assets ratio of 0.5. For 2017, Stacky had an asset turnover of 3.0. What were annual sales for Stacky in 2017? A. $333,333 B. $1,200,000 C. $1,800,000 D. $3,000,000 Liabilities/Assets = 0.5 = $500,000/$1,000,000 So Assets = $1,000,000 Then, Sales/$1,000,000 = 3 So sales = $3,000,000 Accessibility: Keyboard Navigation Difficulty: 2 Medium Gradable: automatic

20. Klamath Corporation has asset turnover of 3.5, a profit margin of 5.2%, and a current ratio of 0.5. What is Klamath Corporation’s return on equity? A. 8.7% B. 9.1% C. 18.2% D. Insufficient information to find ROE Accessibility: Keyboard Navigation Difficulty: 1 Easy Gradable: automatic

21. Assume you are a banker who has loaned money to a firm, but that firm is now facing increased competition and reduced cash flows. Which one of the following ratios would you most closely monitor to evaluate the firm’s ability to repay its loan?

2-4 Copyright © 2018 McGraw-Hill Education. All rights reserved. No reproduction or distribution without the prior written consent of McGraw-Hill Education.


A. current ratio B. debt-to-equity ratio C. times-interest-earned ratio D. times-burden-covered ratio E. None of the options are correct. The times-burden-covered ratio is the best answer, as it indicates how well the firm’s cash flows cover both debt principal and interest payments. The times-interest-earned ratio applies most appropriately when we are confident the firm can roll over existing debt; this is not the case here. Accessibility: Keyboard Navigation Difficulty: 2 Medium Gradable: automatic

22. Breakers Bay Inc. has succeeded in increasing the amount of goods it sells while holding the amount of inventory on hand at a constant level. Assume that both the cost per unit and the selling price per unit also remained constant. All else held constant, how will this accomplishment be reflected in the firm’s financial ratios? A. decrease in the fixed asset turnover rate B. decrease in the financial leverage ratio C. increase in the inventory turnover rate D. increase in the days’ sales in inventory E. decrease in the total asset turnover rate Accessibility: Keyboard Navigation Difficulty: 2 Medium Gradable: automatic

23. Which one of the following statements is correct? A. If the debt-to-assets ratio is greater than 0.50, then the debt-to-equity ratio must be less than 1.0. B. Long-term creditors would prefer the times-interest-earned ratio be 1.4 rather than 1.5. C. The assets-to-equity ratio can be computed as 1 plus the debt-to-equity ratio. D. To realize the best risk and reward profile, financial leverage should be maximized. E. None of the options are correct. Accessibility: Keyboard Navigation Difficulty: 2 Medium Gradable: automatic

24. On a common-size balance sheet, all accounts are expressed as a percentage of A. sales. B. profits. C. equity. D. total assets. E. None of the options are correct. Accessibility: Keyboard Navigation Difficulty: 1 Easy Gradable: automatic

25. Primavera Holdings has a profit margin of 25%, an asset turnover of 0.5, and financial leverage (assets to equity) of 1.5. Primavera has $20 billion in assets, of which half, is in cash and marketable securities. Assume that Primavera earns a 3 percent after-tax return on cash and securities. What would Primavera’s return on equity be if it paid out 90% of its cash and marketable securities as a dividend to shareholders?

2-5 Copyright © 2018 McGraw-Hill Education. All rights reserved. No reproduction or distribution without the prior written consent of McGraw-Hill Education.


A. Negative B. Between 0% and 20% C. Between 20% and 40% D. between 40% and 60% E. Greater than 60% Currently, equity = $13.33 billion (20/13.33 = 1.5), sales = 10 (10/20 = 0.5) and net income = 2.5 (2.5/10 = 25%). Paying a $9 billion dividend would reduce assets to $11 billion and equity to $4.33 billion. Net income would fall by 3% × $9 billion = $0.27 billion, to $2.23 billion. ROE would then be 2.23/4.33 = 51.50% Accessibility: Keyboard Navigation Difficulty: 3 Hard Gradable: automatic

26. Which one of the following statements does NOT describe a problem with using ROE as a performance measure? A. ROE measures return on accounting book value, and this problem is not solved by using market value. B. ROE is a forward-looking, one-period measure, while business decisions span the past and present. C. ROE measures only return, while financial decisions involve balancing risk against return. D. None of these describe problems with ROE. E. All of these describe problems with ROE. Accessibility: Keyboard Navigation Difficulty: 2 Medium Gradable: automatic [The following information applies to the questions displayed below.]

Link, Inc. Selected financial data ($ thousands) 2016 Income statement and related items Sales Cost of goods sold Net income Cash flow from operations Balance sheet items Cash Marketable securities Accounts receivable Inventory Total current assets Accounts payable Accrued liabilities Total current liabilities

2017

$160,835 $ 274,219 141,829 209,628 (91,432) (257,981) (35,831) (12,538) $236,307 $ 164,952 209,670 22,638 12,645 21,655 3,971 40,556 462,593 249,801 17,735 13,962 27,184 76,596 44,919 90,558

27. Please refer to the financial data for Link, Inc. above. The current ratio for Link at the end of 2017 is A. 10.21. B. 2.31. C. 2.76.

2-6 Copyright © 2018 McGraw-Hill Education. All rights reserved. No reproduction or distribution without the prior written consent of McGraw-Hill Education.


D. 10.30. E. None of the options are correct. 249,801/90,558 = 2.76 Accessibility: Keyboard Navigation Difficulty: 1 Easy Gradable: automatic

28. Please refer to the financial data for Link, Inc. above. Which of the following statements best describes how the Link's short-term liquidity changed from 2016 to 2017? A. Link’s short-term liquidity has improved modestly. B. Link’s short-term liquidity has deteriorated very little, but from a low initial base. C. Link’s short-term liquidity has improved considerably, but from a low initial base. D. Link’s short-term liquidity has deteriorated considerably, but from a high initial base. E. None of the options are correct. Accessibility: Keyboard Navigation Difficulty: 2 Medium Gradable: automatic

29. Please refer to the financial data for Link, Inc. above. Assume a 365-day year for your calculations. Link’s collection period in days, based on sales, at the end of 2017 is A. 24.3. B. 219.6. C. 35.7. D. 28.8. E. None of the options are correct. 21,655/(274,219/365) = 28.8 Accessibility: Keyboard Navigation Difficulty: 2 Medium Gradable: automatic

30. Please refer to the financial data for Link, Inc. above. Assume a 365-day year for your calculations. Link’s inventory turnover, based on cost of goods sold, at the end of 2017 is A. 5.2. B. 24.3. C. 28.8. D. 35.7. E. None of the options are correct. 209,628/40,556 = 5.2 Accessibility: Keyboard Navigation Difficulty: 2 Medium Gradable: automatic

31. Please refer to the financial data for Link, Inc. above. Assume a 365-day year for your calculations. Link’s payables period in days, based on cost of goods sold, at the end of 2017 is A. 5.2. B. 24.3. C. 28.8.

2-7 Copyright © 2018 McGraw-Hill Education. All rights reserved. No reproduction or distribution without the prior written consent of McGraw-Hill Education.


D. 35.7. E. None of the options are correct. 13,962/(209,620/365) = 24.3 Accessibility: Keyboard Navigation Difficulty: 2 Medium Gradable: automatic

32. Please refer to the financial data for Link, Inc. above. Assume a 365-day year for your calculations. Link’s days’ sales in cash at the end of 2017 is: A. 24.3 B. 28.8 C. 219.6 D. 249.7 E. None of the options are correct. (164,952 + 22,638)/(274,219/365) = 249.7 Accessibility: Keyboard Navigation Difficulty: 2 Medium Gradable: automatic

33. Please refer to the financial data for Link, Inc. above. Link’s gross margin for 2017 is A. −94%. B. 13%. C. 26%. D. 31%. E. None of the options are correct. (274,219 − 209,628)/274,219 = 23.6% Accessibility: Keyboard Navigation Difficulty: 1 Easy Gradable: automatic

34. Please refer to the financial data for Link, Inc. above. Link’s profit margin for 2017 is A. −94%. B. −57%. C. 13%. D. 31%. E. None of the options are correct. −257,981/274,219 = −94% Accessibility: Keyboard Navigation Difficulty: 1 Easy Gradable: automatic

35. Please refer to the income statement for VGA Associates below. Assuming that cost of goods sold are variable and operating expenses are fixed, what was VGA Associates’ breakeven sales volume in 2017? VGA Associates Income statement for 2017

2-8 Copyright © 2018 McGraw-Hill Education. All rights reserved. No reproduction or distribution without the prior written consent of McGraw-Hill Education.


Sales $200,000 Cost of goods sold 150,000 Gross profit 50,000 Operating expenses 20,000 Operating income 30,000 Interest expense 5,000 Pre-tax income 25,000 Taxes 5,000 Net income $20,000

A. $20,000 B. $80,000 C. $150,000 D. $180,000 E. None of the options are correct. Gross margin = 50,000/200,000 = 25%. Breakeven sales volume = Operating expenses/Gross margin = $20,000/0.25 = $80,000. Accessibility: Keyboard Navigation Difficulty: 2 Medium Gradable: automatic

36. Please refer to the income statement for VGA Associates below. If VGA had a principal repayment of $8,000 due in 2017, what was its times-burden-covered ratio in 2017? VGA Associates Income statement for 2017

2-9 Copyright © 2018 McGraw-Hill Education. All rights reserved. No reproduction or distribution without the prior written consent of McGraw-Hill Education.


Sales Cost of goods sold Gross profit Operating expenses Operating income Interest expense Pre-tax income Taxes Net income

$ 200,000 150,000 50,000 20,000 30,000 5,000 25,000 5,000 $ 20,000

A. 0.67 B. 1.33 C. 2.31 D. 6.00 E. None of the options are correct. EBIT TBC = Int.Exp. +

Prin.Repay. 1−t

TBC =

30,000 8,000 5,000 + 1 − 0.2

= 2

Accessibility: Keyboard Navigation Difficulty: 2 Medium Gradable: automatic

37. Ellsbury Corporation has a goal to reduce its cash conversion cycle. Which of the following actions, holding all else equal, is likely to accomplish this goal? A. Ellsbury changes the credit terms it offers to customers, allowing them to pay in 45 days instead of 30 days. B. Ellsbury increases the efficiency of its production process, reducing by 10% the average time it takes to convert raw materials to finished products. C. Ellsbury starts paying off all outstanding invoices to suppliers twice a month instead of once a month.

2-10 Copyright © 2018 McGraw-Hill Education. All rights reserved. No reproduction or distribution without the prior written consent of McGraw-Hill Education.


D. Ellsbury increases its cash/assets ratio from 12% to 15%. A would increase CCC, because A/R would increase. B would reduce CCC, because inventory would decrease. C would increase CCC, because A/P would decrease. D would not affect CCC. Accessibility: Keyboard Navigation Difficulty: 2 Medium Gradable: automatic

38. What is the length of the cash conversion cycle for a firm with $3 million in inventory, $1.5 million in accounts payable, a collection period of 40 days, and an annual cost of goods sold of $18 million? A. 34.0 days B. 51.2 days C. 70.4 days D. 131.2 days E. None of the options are correct. CCC = Days Inventory Outstanding + Collection Period − Payables Period = Inventory/(COGS/365) + Receivables/(Sales/365) − Payables/(COGS/365) = 3/(18/365) + 40 − 1.5/(18/365) = 60.8 + 40 − 30.4 = 70.4 days Accessibility: Keyboard Navigation Difficulty: 2 Medium Gradable: automatic

39. Answer the questions below based on the following information. The tax rate is 35%, and all dollars are in millions. Assume that the companies have no liabilities other than the debt shown below. Suunto Inc. Runrun Corp. Earnings before interest and taxes Debt (at 10% interest) Equity

$

280

$

294

$ $

140 560

$ $

840 210

2-11 Copyright © 2018 McGraw-Hill Education. All rights reserved. No reproduction or distribution without the prior written consent of McGraw-Hill Education.


a. Calculate each company’s ROE, ROA, and ROIC. b. Why is Runrun’s ROE so much higher than Suunto’s? Does this mean Runrun is a better company? Why or why not? c. Why is Suunto’s ROA higher than Runrun’s? What does this tell you about the two companies? d. How do the two companies’ ROICs compare? What does this suggest about the two companies? a. Net income ROE ROA ROIC

Suunto Inc.

Runrun Corp.

172.9

136.5

31 % 25 % 26 %

65 % 13 % 18 %

b. Runrun’s higher ROE is a natural reflection of its higher financial leverage. It does not mean that Runrun is the better company. c. This is also due to Runrun’s higher leverage. ROA penalizes levered companies by comparing the net income available to equity to the capital provided by owners and creditors. It does not mean that Runrun is a worse company than Suunto. d. ROIC abstracts from differences in leverage to provide a direct comparison of the earning power of the two companies’ assets. On this metric, Suunto is the superior performer. Before drawing any firm conclusions, however, it is important to ask how the business risks faced by the companies compare and whether the observed ratios reflect long-run capabilities or transitory events.

Accessibility: Keyboard Navigation Difficulty: 2 Medium Gradable: manual

[The following information applies to the questions displayed below.] The financial statements for Limited Brands, Inc. follow (fiscal years ending January): Limited Brands, Inc. Balance Sheets ($ Millions) 2007

2006

2005

2-12 Copyright © 2018 McGraw-Hill Education. All rights reserved. No reproduction or distribution without the prior written consent of McGraw-Hill Education.


Total Assets

7,093.00

6,346.00

6,089.00

8

7

0

1,701.00

1,568.00

1,451.00

Total Current Liabilities

1,709.00

1,575.00

1,451.00

Long-Term Debt

1,665.00

1,669.00

1,646.00

Deferred Taxes

173

146

177

Minority Interest

71

33

33

Other Liabilities

520

452

447

Total Liabilities

4,138.00

3,875.00

3,754.00

Total Equity

2,955.00

2,471.00

2,335.00

Total Liabilities & Equity

7,093.00

6,346.00

6,089.00

398

395

407

Liabilities Long-Term Debt Due In One Year Payables and Accrued Expenses

Common Shares Outstanding

Income Statements ($ MILLIONS) 2007

2006

Sales

10,671.00

9,669.00

Cost of Goods Sold

6,342.00

5,920.00

4,329.00

3,749.00

2,837.00

2,502.50

1,492.00

1,246.50

316

299

Gross Profit Selling, General, & Administrative Exp. Operating Income Before Deprec. Depreciation, Depletion, & Amortization Operating Profit

1,176.00

947.5

Interest Expense

102

94

Non-Operating Income/Expense

23

25

Special Items

0

78.5

Pretax Income

1,097.00

957

Total Income Taxes

422

291

Adjusted Available for Common

675

666

Extraordinary Items

1

17

Adjusted Net Income

676

683

Dividends per share $ 0.6 $ 0 40. Please refer to Limited Brands, Inc.’s financial statements above. Use the company’s operating profit as an approximation of its EBIT, and assume a 40% tax rate for your calculations. For the fiscal years ending in January of 2006 and 2007, calculate: a. Debt-to-equity ratio b. Times-interest-earned ratio c. Times burden covered

2-13 Copyright © 2018 McGraw-Hill Education. All rights reserved. No reproduction or distribution without the prior written consent of McGraw-Hill Education.


Fiscal Year Ending Jan. 2007 Jan. 2006 a. Debt-to-equity ratio 1.4 1.57 b. Times-interest-earned ratio 11.53 10.08 c. Times burden covered 10.35 10.08 (Note that principal payment in year t equals current portion of long-term debt in year t -1.) Accessibility: Keyboard Navigation Difficulty: 2 Medium Gradable: manual

41. Please refer to Limited Brands, Inc.’s financial statements above. Use the company’s operating profit as an approximation of its EBIT, and assume a 40% tax rate for your calculations. What percentage decline in earnings before interest and taxes could Limited Brands have sustained in fiscal years ending in January 2006 and 2007 before failing to cover: a. Interest and principal repayment requirements? b. Interest, principal, and common dividend payments? a. For the fiscal year ending January 2006: Interest expense = $94 Principal repayment = $0 (long−term debt due in one year from 2005) EBIT = $947.5, so it could have fallen (947.5 − 94)/947.5 = 90.1% before failing to cover interest and principal. For the fiscal year ending January 2007: Interest expense = $102 Principal repayment = $7 (long−term debt due in one year from 2006) EBIT = $1,176, so it could have fallen (1,176 − 102 − 7/0.6)/1,176 = 90.3% before failing to cover interest and principal. b. For the fiscal year ending January 2006: Interest expense = $94 Principal repayment = $0 (long−term debt due in one year from 2005) Common dividends = Shares outstanding × Dividends per share = 395 × 0.61 = $241.0 EBIT = $947.5, so it could have fallen (947.5 − 94 − 241/0.6)/947.5 = 47.7% before failing to cover interest, principal, and dividends. For the fiscal year ending January 2007: Interest expense = $102 Principal repayment = $7 (long−term debt due in one year from 2006) Common dividends = Shares outstanding × Dividends per share = 398 × 0.60 = $238.8 EBIT = $1,176, so it could have fallen (1,176 − 102 − 245.8/0.6)/1,176 = 56.5% before failing to cover interest, principal, and dividends. Accessibility: Keyboard Navigation Difficulty: 2 Medium Gradable: manual

42. Please refer to Limited Brands, Inc.’s financial statements above. Prepare common-size financial statements for Limited Brands, Inc. for 2006–2007.

BALANCE SHEETS (% of Assets) 2007

2006

2005

2-14 Copyright © 2018 McGraw-Hill Education. All rights reserved. No reproduction or distribution without the prior written consent of McGraw-Hill Education.


TOTAL ASSETS

100

%

100

%

100

%

0.11

%

0.11

%

0

%

23.98

%

24.71

%

23.83

%

Total Current Liabilities

24.09

%

24.82

%

23.83

%

Long-Term Debt

23.47

%

26.3

%

27.03

%

Deferred Taxes

2.44

%

2.3

%

2.91

%

Minority Interest

1

%

0.52

%

0.54

%

Other Liabilities

7.33

%

7.12

%

7.34

%

TOTAL LIABILITIES

58.34

%

61.06

%

61.65

%

41.66

%

38.94

%

38.35

%

100

%

100

%

100

%

LIABILITIES Long-Term Debt Due In One Year Payables and Accrued Expenses

TOTAL EQUITY TOTAL LIABILITIES & EQUITY Common Shares Outstanding

398

395

407

INCOME STATEMENTS (% of Sales) 2007

2006

Sales

100

%

100

%

Cost of Goods Sold

59.4

%

61.23

%

40.6

%

38.77

%

26.6

%

25.88

%

14

%

12.89

%

2.96

%

3.09

%

11

%

9.8

%

Interest Expense

0.96

%

0.97

%

Non-Operating Income/Expense

0.22

%

0.26

%

Special Items

0

%

0.81

%

Pretax Income

10.3

%

9.9

%

Total Income Taxes

3.95

%

3.01

%

Adjusted Available for Common

6.33

%

6.89

%

Extraordinary Items

0.01

%

0.18

%

Adjusted Net Income

6.33

%

7.06

%

Gross Profit Selling, General, & Administrative Exp. Operating Income Before Deprec. Depreciation, Depletion, & Amortization Operating Profit

Accessibility: Keyboard Navigation Difficulty: 2 Medium Gradable: manual

2-15 Copyright © 2018 McGraw-Hill Education. All rights reserved. No reproduction or distribution without the prior written consent of McGraw-Hill Education.


43. Please refer to the financial statements for Roxbury Corporation. Estimate the length of Roxbury’s cash conversion cycle in 2017.

Roxbury Corporation 2016 and 2017, ($ millions) INCOME STATEMENT 2016 Net sales Cost of goods sold GS&A expense

44,790

Accounts receivable

6,66

7,333

6,171

6,788

Inventories

5,236

5,760

EBIT

727

800

Net fixed assets

2,048

2,253

Interest expense Earnings before tax Tax

215

255

Total assets

$

14,901

$

16,392

512

545

Bank loan

$

392

$

547

154

164

Accounts payable

7,419

8,161

382

Long-term debt

2,148

2,551

9,959

11,259

Common stock

1,293

1,293

Retained earnings

3,649

3,840

Total equity Total liabilities & equity

4,942

5,133

$

$

Total liabilities

$

951

2017

40,718

358

$

2016 Cash & securities

$

47,616

2017 52,378

Net income

$

BALANCE SHEET

14,901

$

$

1,046

16,392

CCC = Days Inventory Outstanding + Collection Period − Payables Period = Inventory/(COGS/365) + Receivables/(Sales/365) − Payables/(COGS/365) = 5760/(44,790/365) + 7333/(52,378/365) − 8161/(44,790/365) = 46.9 + 51.1 − 66.5 = 31.5 days Accessibility: Keyboard Navigation Difficulty: 2 Medium Gradable: manual

2-16 Copyright © 2018 McGraw-Hill Education. All rights reserved. No reproduction or distribution without the prior written consent of McGraw-Hill Education.


Chapter 02 Test Bank Summary Category Accessibility: Keyboard Navigation Difficulty: 1 Easy Difficulty: 2 Medium Difficulty: 3 Hard Gradable: automatic Gradable: manual

# of Questions 43 20 22 1 38 5

2-17 Copyright © 2018 McGraw-Hill Education. All rights reserved. No reproduction or distribution without the prior written consent of McGraw-Hill Education.


Chapter 03 Test Bank 1. The percent-of-sales approach to financial forecasting works well for forecasting the income statement but is not useful for forecasting the balance sheet.

FALSE Accessibility: Keyboard Navigation Difficulty: 1 Easy Gradable: automatic

2. An advantage of the percent-of-sales approach to financial forecasting is that effective forecasts can be prepared without consulting historical financial statements. FALSE Accessibility: Keyboard Navigation Difficulty: 1 Easy Gradable: automatic

3. The forecast for retained earnings on the 2019 balance sheet can be determined as 2018 retained earnings plus projected 2019 after-tax earnings less projected 2019 dividends. TRUE Accessibility: Keyboard Navigation Difficulty: 2 Medium Gradable: automatic

4. An annual financial forecast for 2017 showing no external funding required assures a company that no cash shortfalls are likely to occur during 2017. FALSE Accessibility: Keyboard Navigation Difficulty: 1 Easy Gradable: automatic

5. All else equal, increasing the assumed payables period in a financial forecast will decrease external funding required. TRUE Accessibility: Keyboard Navigation Difficulty: 1 Easy Gradable: automatic

6. All else equal, increasing the assumed collection period in a financial forecast will decrease external funding required. FALSE Accessibility: Keyboard Navigation Difficulty: 1 Easy Gradable: automatic

7. Given the same assumptions, cash flow forecasts and pro forma projections will yield the same need for external funding. TRUE Accessibility: Keyboard Navigation Difficulty: 1 Easy Gradable: automatic

3-1 Copyright © 2018 McGraw-Hill Education. All rights reserved. No reproduction or distribution without the prior written consent of McGraw-Hill Education.


8. A drawback of forecasting using spreadsheets is that typical spreadsheet programs are not equipped to deal with the circularity involving interest expense and debt. FALSE Accessibility: Keyboard Navigation Difficulty: 1 Easy Gradable: automatic

9. Scenario analysis involves changing one input to a financial forecast, whereas sensitivity analysis involves changing multiple inputs. FALSE Accessibility: Keyboard Navigation Difficulty: 1 Easy Gradable: automatic

10. Cash flow forecasts are less informative than pro forma financial statements. TRUE Accessibility: Keyboard Navigation Difficulty: 1 Easy Gradable: automatic

11. You are estimating your company’s external financing needs for the next year. At the end of next year, you expect that owners’ equity will be $80 million, total assets will amount to $170 million, and total liabilities will be $70 million. How much will your firm need to borrow, or otherwise acquire, from outside sources during the next year? A. $20 million B. $70 million C. $150 million D. $160 million E. $180 million F. None of the options are correct. Accessibility: Keyboard Navigation Difficulty: 1 Easy Gradable: automatic

12.You are estimating your company’s external financing needs for the next year. Your first-pass pro forma financial statements showed a large financing deficit for next year. Which of the following changes to your company’s operating plan would reduce the financing deficit if incorporated in revised pro forma financial statements? A. Increase the sales growth rate B. Increase cost of goods sold as a percentage of sales C. Reduce the collection period D. Increase the dividend payout ratio E. None of the options are correct. Accessibility: Keyboard Navigation Difficulty: 1 Easy Gradable: automatic

3-2 Copyright © 2018 McGraw-Hill Education. All rights reserved. No reproduction or distribution without the prior written consent of McGraw-Hill Education.


13. To estimate Missed Places, Inc.’s (MP) external financing needs, the CFO needs to figure out how much equity her firm will have at the end of next year. At the end of the most recent fiscal year, MP’s retained earnings were $158,000. The Controller has estimated that over the next year, gross profits will be $360,700, earnings after tax will total $23,400, and MP will pay $12,400 in dividends. What are the estimated retained earnings at the end of next year? A. $169,000 B. $170,400 C. $181,400 D. $506,300 E. $518,700 F. None of the options are correct. 158,000 + 23,400 − 12,400 = $169,000 Accessibility: Keyboard Navigation Difficulty: 1 Easy Gradable: automatic

14. The most common approach to developing pro forma financial statements is called the

A. cash budget method. B. financial planning method. C. seasonality approach. D. percent-of-sales method. E. market-oriented approach. F. None of the options are correct. Accessibility: Keyboard Navigation Difficulty: 1 Easy Gradable: automatic

15. Which of the following are viable techniques to cope with the uncertainty inherent in realistic financial projections? I. Simulation II. Ad hoc adjustments III. Scenario analysis IV. Sensitivity analysis A. II and IV only B. III and IV only C. II, III, and IV only D. I, II, and III only E. I, III, and IV only F. I, II, III, and IV Accessibility: Keyboard Navigation Difficulty: 2 Medium Gradable: automatic

16. Which one of the following statements is correct concerning the cash balance of a firm? A. Most firms attempt to maintain a zero cash balance at all times. B. The cumulative cash surplus shown on a cash budget is equal to the ending cash balance plus the minimum desired cash balance. C. Most firms attempt to maximize the cash balance at all times. D. A cumulative cash deficit on a cash budget indicates the need to acquire additional funds. E. The ending cash balance must equal the minimum desired cash balance. Accessibility: Keyboard Navigation Difficulty: 1 Easy Gradable: automatic

3-3 Copyright © 2018 McGraw-Hill Education. All rights reserved. No reproduction or distribution without the prior written consent of McGraw-Hill Education.


17. Assume each month has 30 days and AmDocs has a 60-day accounts receivable period. During the second calendar quarter of the year (April, May, and June), AmDocs will collect payment for the sales it made during which of the months listed below? A. October, November, and December B. November, December, and January C. December, January, and February D. January, February, and March E. February, March, and April Accessibility: Keyboard Navigation Difficulty: 1 Easy Gradable: automatic

18. The Limited collects 25 percent of sales in the month of sale, 60 percent of sales in the month following the month of sale, and 15 percent of sales in the second month following the month of sale. During the month of April, the firm will collect

A. 60 percent of February sales. B. 15 percent of April sales. C. 60 percent of March sales. D. 15 percent of March sales. E. 25 percent of February sales. Accessibility: Keyboard Navigation Difficulty: 1 Easy Gradable: automatic

19. Steve has estimated the cash inflows and outflows for his sporting goods store for next year. The report that he has prepared summarizing these cash flows is called a

A. pro forma income statement. B. sales projection. C. cash budget. D. receivables analysis. E. credit analysis. F. None of the options are correct. Accessibility: Keyboard Navigation Difficulty: 1 Easy Gradable: automatic

20. You are developing a financial plan for a corporation. Which of the following questions will be considered as you develop this plan? I. How much will our sales grow? II. Will additional fixed assets be required? III. Will dividends be paid to shareholders? IV. How much new debt must be obtained? A. I and IV only B. II and III only C. I, III, and IV only D. II, III, and IV only E. I, II, III, and IV Accessibility: Keyboard Navigation Difficulty: 1 Easy Gradable: automatic

3-4 Copyright © 2018 McGraw-Hill Education. All rights reserved. No reproduction or distribution without the prior written consent of McGraw-Hill Education.


21. Ruff Wear expects sales of $560, $650, $670, and $610 for the months of May through August, respectively. The firm collects 20 percent of sales in the month of sale, 70 percent in the month following the month of sale, and 8 percent in the second month following the month of sale. The remaining 2 percent of sales is never collected. How much money does the firm expect to collect in the month of August? A. $621 B. $628 C. $633 D. $639 E. $643 August collections = 0.20($610) + 0.70($670) + 0.08($650) = $643 Accessibility: Keyboard Navigation Difficulty: 1 Easy Gradable: automatic

22. On May 1, Vaya Corp. had a beginning cash balance of $175. Vaya’s sales for April were $430, and May sales were $480. During May, the firm had cash expenses of $110 and made payments on accounts payable of $290. Vaya’s accounts receivable period is 30 days. What is the firm’s beginning cash balance on June 1?

A. $145 B. $155 C. $205 D. $215 E. $265 Cash balance = $175 − $110 − $290 + $430 = $205 Accessibility: Keyboard Navigation Difficulty: 1 Easy Gradable: automatic

23. You are preparing pro forma financial statements for 2017 using the percent-of-sales method. Sales were $100,000 in 2016 and are projected to be $120,000 in 2017. Net income was $5,000 in 2016 and is projected to be $6,000 in 2017. Equity was $45,000 at year-end 2015 and $50,000 at year-end 2016. Assuming that this company never issues new equity, never repurchases equity, and never changes its dividend payout ratio, what would be projected for equity at year-end 2017? A. $55,000 B. $56,000 C. $60,000 D. Insufficient information is provided to project equity in 2017. All of net income was added to equity in 2016, so all of net income will be added to equity in 2017. $50,000 + $6,000 = $56,000. Accessibility: Keyboard Navigation Difficulty: 2 Medium Gradable: automatic

3-5 Copyright © 2018 McGraw-Hill Education. All rights reserved. No reproduction or distribution without the prior written consent of McGraw-Hill Education.


[The following information applies to the questions displayed below.]

Oscar's Incredible Eatery ($ thousands) Income Statement for the year ending Dec. 31, 2017 Net sales 17,300 Cost of goods sold 10,600 Depreciation 3,250 Earnings before interest and taxes 3,450 Interest expense 680 Earnings before tax 2,770 Tax 940 Earnings after tax 1,830 Dividends

450

Oscar's Incredible Eatery ($ thousands) Balance Sheet as of Dec. 31, 2017 Cash 350 Accounts payable 1,920 Accounts receivable940 Long-term debt 3,500 Inventory 2,360 Common stock 7,500 Total current assets 3,650 Retained earnings 1,580 Net fixed assets 10,850 Total assets 14,500Total liab. & equity14,500

24. Please refer to Oscar’s financial statements above. What was Oscar’s increase in retained earnings during 2017? A. $450 B. $1,380 C. $1,830 D. $2,280 E. None of the options are correct. 1,830 − 450 = 1,380 Accessibility: Keyboard Navigation Difficulty: 1 Easy Gradable: automatic

25. Please refer to Oscar’s financial statements above. Sales are projected to increase by 3 percent next year. The profit margin and the dividend payout ratio are projected to remain constant. What is the projected addition to retained earnings for next year? A. $1,309.19 B. $1,421.40 C. $1,884.90 D. $2,667.78 E. $3,001.40 F. None of the options are correct. Projected addition to retained earnings = $1,380 × (1 + 0.03) = $1,421.40 Accessibility: Keyboard Navigation Difficulty: 2 Medium Gradable: automatic

3-6 Copyright © 2018 McGraw-Hill Education. All rights reserved. No reproduction or distribution without the prior written consent of McGraw-Hill Education.


26. Please refer to Oscar’s financial statements above. All of Oscar’s costs and current asset accounts vary directly with sales. Sales are projected to increase by 10 percent. What is the pro forma accounts receivable balance for next year? A. $949 B. $1,034 C. $1,113 D. $1,730 E. $2,670 F. None of the options are correct. Pro forma accounts receivable = $940 × (1 + 0.10) = $1,034 Accessibility: Keyboard Navigation Difficulty: 1 Easy Gradable: automatic

27. Please refer to Oscar’s financial statements above. Assume a constant profit margin and dividend payout ratio, and further assume all of Oscar’s assets and current liabilities vary directly with sales. Assume long-term debt and common stock remain unchanged. Sales are projected to increase by 10 percent. What is Oscar’s external financing need for next year? A. -$410 B. -$260 C. $235 D. $1,320 E. $7,240 F. None of the options are correct. Projected total assets = $14,500 × 1.10 = $15,950 Projected total liabilities = $1,920 × 1.10 + $3,500 = $5,612 Projected total equity = $7,500 + ($1,580 + $1,380 × 1.1) = $10,598 External financing needed = $15,950 − ($5,612 + $10,598) = −$260 Accessibility: Keyboard Navigation Difficulty: 2 Medium Gradable: automatic

28. Please refer to Oscar’s financial statements above. Assume a constant debt-equity ratio, net profit margin, and dividend payout ratio, and further assume all of Oscar’s expenses, assets, and current liabilities vary directly with sales. What is the pro forma net fixed asset value for next year if sales are projected to increase by 7.5 percent?

A. $10,857.50 B. $10,931.38 C. $11,663.75 D. $15,587.50 E. $18,987.50 F. None of the options are correct. Pro forma net fixed assets = $10,850 × (1 + 0.075) = $11,663.75 Accessibility: Keyboard Navigation Difficulty: 2 Medium Gradable: automatic

3-7 Copyright © 2018 McGraw-Hill Education. All rights reserved. No reproduction or distribution without the prior written consent of McGraw-Hill Education.


[The following information applies to the questions displayed below.]

Financial Statements for Royal Corporation Actual 2016 and Pro Forma 2017 ($ millions) Income Statement

Balance Sheet

Net sales Cost of goods sold Other expenses Depreciation expense EBIT

2016 $ 47,616 40,718 5,171 1,000 727

2017 $ 52,378 Cash & securities 44,790 Accounts receivable 5,688 Inventories 1,100 Net fixed assets 800 Total assets

Interest expense Earnings before tax Tax Net income

215 512 154 $ 359

215 585 176 $ 409

Dividends paid Add. to retained earnings

90 $ 269

102

Bank loan (short-term) Accounts payable Long-term debt Total liabilities Shareholders' equity Total liabilities & equity

2016 $ 951 5,666 4,236 4,048 14,901

2017 $ 1,046 6,233 4,660

$ 392 $ 431 7,419 8,161 2,148 9,959 4,942 $ 14,901

29. In the above financial statements, Royal Corporation has prepared (incomplete) pro forma financial statements for 2017 based on actual financial statements for 2016. Royal Corp. used the percent-of-sales method, assuming a sales growth rate of 10% for 2017. If capital expenditures are planned to be $1,615 in 2017, then what would be the appropriate projection for net fixed assets in 2017?

A. $4,453 B. $4,563 C. $4,663 D. $5,663 4,048 + 1,615 − 1,100 = $4,563 Accessibility: Keyboard Navigation Difficulty: 1 Easy Gradable: automatic

30. Please refer to the pro forma financial statements for Royal Corporation above. If Royal Corporation plans to issue $100 in new equity in 2017, what should be the projection for shareholders’ equity for 2017? A. $5,349 B. $5,436 C. $5,451 D. $5,536 4,942 + 307 + 100 = $5,349 Accessibility: Keyboard Navigation Difficulty: 1 Easy Gradable: automatic

3-8 Copyright © 2018 McGraw-Hill Education. All rights reserved. No reproduction or distribution without the prior written consent of McGraw-Hill Education.


31. Please refer to the pro forma financial statements for Royal Corporation above. Assume that net fixed assets are projected to be $5,000 for 2017 and that shareholders’ equity is projected to be $5,500 for 2017. If long-term debt is the plug figure, what should be the projection for long-term debt for Royal Corporation in 2017?

A. $2,206 B. $2,363 C. $2,455 D. $2,847 Total assets would be 1,046 + 6,233 + 4,660 + 5,000 = $16,939 Total liabilities and equity, without long-term debt, would be 431 + 8,161 + 5,500 = $14,092 Long-term debt must make up the difference = 16,939 − 14,092 = $2,847 Accessibility: Keyboard Navigation Difficulty: 2 Medium Gradable: automatic

[The following information applies to the questions displayed below.] Selected assumptions for 2018 Sales growth rate Cost of goods sold/Sales Dividends/Net income

9

%

62

%

40

%

Income Statement

Sales

$

Balance Sheet

Actual

Forecast

Actual

Forecast

2017

2018

2017

2018

1,000

Cash

$

100

Cost of goods sold

600

Accounts receivable

200

Operating expense Depreciation expense EBIT

200

Inventory

500

100

Total Current Assets

800

100

Net PP&E

1,000

Interest expense

35

Total Assets

1,800

Pre-tax income

65

Accounts payable

300

Tax

26

Bank loan Total Current Liabilities Long-Term Debt

100

Shareholders' Equity Total Liabilities & Equity

1,000

Net Income

$

39

400 400

$

1,800

32. Please refer to the spreadsheet above. Selected assumptions are given for preparing pro forma financial statements for 2018. Which of the following formulas would correctly give the forecast for sales in cell C8? A. = B8 × B2 B. = B8 + B8 × B2 C. = (1 + B8) × B2 D. = (1/B2) × B8 E. None of the options are correct.

3-9 Copyright © 2018 McGraw-Hill Education. All rights reserved. No reproduction or distribution without the prior written consent of McGraw-Hill Education.


Accessibility: Keyboard Navigation Difficulty: 1 Easy Gradable: automatic

33. Please refer to the spreadsheet above. Selected assumptions are given for preparing pro forma financial statements for 2018. When the pro formas are completed, which of the following formulas would correctly give the forecast for cost of goods sold in cell C9? A. = B9 × B3 B. = B9 + B9 × B3 C. = B8 × B3 D. = B9 × B2 E. None of the options are correct. Accessibility: Keyboard Navigation Difficulty: 1 Easy Gradable: automatic

34. Please refer to the spreadsheet above. Selected assumptions are given for preparing pro forma financial statements for 2018. Assume that no new equity will be issued in 2018. When the pro formas are completed, which of the following formulas would correctly give the forecast for shareholders’ equity in cell G19? A. = F19 × B2 B. = F19 × (1 + B2) C. = F19 + (1 − B4) × C16 D. = F19 + B4 × C16 E. None of the options are correct. Accessibility: Keyboard Navigation Difficulty: 2 Medium Gradable: automatic

35. Which of the following statements is correct if a firm’s pro forma financial statements project net income of $12,000 and external financing required of $5,000? A. Total assets cannot grow by more than $10,000. B. Dividends cannot exceed $10,000. C. Retained earnings cannot grow by more than $12,000. D. Long-term debt cannot grow by more than $5,000. Accessibility: Keyboard Navigation Difficulty: 2 Medium Gradable: automatic

36. Pro forma financial statements, by definition, are predictions of a company’s financial statements at a future point in time. So, why is it important to analyze the historical performance of the company before constructing pro forma financial statements? Historical analysis helps decide for which financial statement items a percent-of-sales forecast might be appropriate. For example, a stable trend in the collection period would tell you that, unless you expect changes in the management of the accounts receivable, future collection periods should continue along this trend. Accessibility: Keyboard Navigation Difficulty: 2 Medium Gradable: manual

3-10 Copyright © 2018 McGraw-Hill Education. All rights reserved. No reproduction or distribution without the prior written consent of McGraw-Hill Education.


37. Edna’s Laundry Services just completed pro forma statements using the percentage-of-sales approach. The pro forma shows a projected external financing need of -$5,500. Interpret this figure. What are the firm’s options in this case? A negative value implies that the company has excess cash above its desired minimum. With a negative external financing need, the firm has a surplus of funds that it can use to reduce current liabilities, reduce long-term debt, buy back common stock, or increase dividends. If acceptable opportunities exist, the firm might also use the extra funds to purchase fixed assets, thereby increasing its potential growth, should that action be warranted. Accessibility: Keyboard Navigation Difficulty: 2 Medium Gradable: manual

38. Preston Fencing Company’s sales, half of which are for cash and the other half sold on credit, over the past three months were: a. Estimate Preston’s cash receipts in October if the company’s collection period is 30 days. b. Estimate Preston’s cash receipts in October if the company’s collection period is 45 days. c. What would be Preston’s accounts receivable balance at the end of October if the company’s collection period is 30 days? 45 days? a. If the collection period is 30 days, October cash receipts from September sales will equal half of September sales or $60,000. In addition, the company will receive cash from half of October sales, which were for cash of $40,000. The total is $100,000. b. With a 45–day collection period, cash collected during the first half of October is from credit sales made from the middle until the end of August; and collections during the second half of October are from credit sales made from the beginning until the middle of September. Therefore, cash receipts from credit sales (which are half of total sales) are from the period mid-August through mid–September, or (70,000/2 + 120,000/2)/2 = $47,500. Adding sales for cash of $40,000 in October, the total is $87,500. c. If the collection period is 30 days, then the October accounts receivable balance should be the last 30 days" worth of credit sales. October credit sales were $40,000, thus the accounts receivable balance would be $40,000. If the collection period is 45 days, then the balance would be October"s credit sales and half of September"s credit sales, or 40,000 + 60,000/2 = $70,000. Accessibility: Keyboard Navigation Difficulty: 2 Medium Gradable: manual

39. Suppose your colleague constructed a pro forma balance sheet and a cash budget for your company for the same time period, and the external financing required from the pro forma forecast exceeded the cash deficit estimated on the cash budget. How would you interpret this result? This would tell you that your colleague had erred in constructing one or both of the forecasts. Using the same assumptions and avoiding accounting and arithmetic errors, estimated external financing required should equal estimated cash surplus or deficit for the same date. Accessibility: Keyboard Navigation Difficulty: 1 Easy Gradable: manual

3-11 Copyright © 2018 McGraw-Hill Education. All rights reserved. No reproduction or distribution without the prior written consent of McGraw-Hill Education.


40. Complete the following pro forma financial statements for XYZ Corporation. Use the percent-of-sales method and use long-term debt as the plug figure (balancing item). Assume the following: 20% sales growth, capital expenditures of $200 in 2018, no equity issues or repurchases in 2018, no sale or disposal of fixed assets in 2018, and a 50% dividend payout ratio. Round figures to the nearest whole dollar. XYZ Corporation Financial Statements Actual 2017 and Pro Forma 2018 Income Statement

Balance Sheet 2017 2018 2017 2018 sales $1,000 Current assets $ 300 COGS 700 Net fixed assets 500 Operating expense 100 Total assets 800 Depreciation expense 100 130 EBIT 100 Current liabilities 400 Interest expense 25 20 Long-term debt 200 Pre-tax income 75 Equity 200 Tax 25 Total liabilities & equity$ 800 Net income $50 XYZ Corporation Financial Statements Actual 2017 and Pro Forma 2018 Income Statement

Balance Sheet 2017 2018 2017 2018 sales $1,000$1,200Current assets $ 300 $360 COGS 700 840 Net fixed assets 500 570 Operating expense 100 120 Total assets 800 930 Depreciation expense 100 130 EBIT 100 110 Current liabilities 400 480 Interest expense 25 20 Long-term debt 200 220 Pre-tax income 75 90 Equity 200 230 Tax 25 30 Total liabilities & equity$ 800 $930 Net income $50 $60 Notes: Net fixed assets = 500 + 200 − 130 = 570 Equity = 200 + 0.5 × 60 = 230 Accessibility: Keyboard Navigation Difficulty: 2 Medium Gradable: manual

3-12 Copyright © 2018 McGraw-Hill Education. All rights reserved. No reproduction or distribution without the prior written consent of McGraw-Hill Education.


Chapter 03 Test Bank Summary Category Accessibility: Keyboard Navigation Difficulty: 1 Easy Difficulty: 2 Medium Gradable: automatic Gradable: manual

# of Questions 40 27 13 35 5

3-13 Copyright © 2018 McGraw-Hill Education. All rights reserved. No reproduction or distribution without the prior written consent of McGraw-Hill Education.


Chapter 04 Test Bank 1. The sustainable growth rate is defined as the maximum rate at which company sales can increase. FALSE Accessibility: Keyboard Navigation Difficulty: 1 Easy Gradable: automatic

2. The sustainable growth rate is the only growth rate in sales that is consistent with stable values of the profit margin, retention rate, asset turnover, and leverage (assets/equitybop). TRUE Accessibility: Keyboard Navigation Difficulty: 1 Easy Gradable: automatic

3. A company experiencing balanced growth does not generate cash surpluses or cash deficits. TRUE Accessibility: Keyboard Navigation Difficulty: 1 Easy Gradable: automatic

4. If a company seeks to maximize firm value, it should never grow at a rate above its sustainable growth rate. FALSE Accessibility: Keyboard Navigation Difficulty: 1 Easy Gradable: automatic

5. The only way a company can grow at a rate above its current sustainable growth rate is by increasing leverage. FALSE Accessibility: Keyboard Navigation Difficulty: 2 Medium Gradable: automatic

6. In recent years, U.S. companies as a whole have repurchased more equity than they have issued. TRUE Accessibility: Keyboard Navigation Difficulty: 1 Easy Gradable: automatic

7. Share repurchases usually decrease earnings per share. FALSE Accessibility: Keyboard Navigation Difficulty: 1 Easy Gradable: automatic

4-1 Copyright © 2018 McGraw-Hill Education. All rights reserved. No reproduction or distribution without the prior written consent of McGraw-Hill Education.


8. Issue costs of equity are high relative to those of debt. TRUE Accessibility: Keyboard Navigation Difficulty: 2 Medium Gradable: automatic

9. One way to manage an actual growth rate above the sustainable growth rate is to decrease prices. FALSE Accessibility: Keyboard Navigation Difficulty: 1 Easy Gradable: automatic

10. One way to manage an actual growth rate below the sustainable growth rate is to repurchase shares. TRUE Accessibility: Keyboard Navigation Difficulty: 1 Easy Gradable: automatic

11. Which one of the following will increase the sustainable rate of growth a corporation can achieve? A. avoidance of external equity financing B. increase in corporate tax rates C. reduction in the retention ratio D. decrease in the dividend payout ratio E. decrease in sales given a positive profit margin F. None of the options are correct. Accessibility: Keyboard Navigation Difficulty: 1 Easy Gradable: automatic

12. Which of these ratios are the determinants of a firm’s sustainable growth rate? I. Assets-to-equity ratio II. Profit margin III. Retention ratio IV. Asset turnover ratio A. I and III only B. II and III only C. II, III, and IV only D. I, II, and III only E. I, II, III, and IV F. None of the options are correct Accessibility: Keyboard Navigation Difficulty: 1 Easy Gradable: automatic

4-2 Copyright © 2018 McGraw-Hill Education. All rights reserved. No reproduction or distribution without the prior written consent of McGraw-Hill Education.


13. The retention ratio is A. equal to net income divided by the change in total equity. B. the percentage of net income available to the firm to fund future growth. C. equal to one minus the asset turnover ratio. D. the change in retained earnings divided by the dividends paid. E. the dollar increase in net income divided by the dollar increase in sales. F. None of the options are correct. Accessibility: Keyboard Navigation Difficulty: 1 Easy Gradable: automatic

14. Which of the following statements is true? A. Rapid growth spurs increases in market share and profits and thus, is always a blessing. B. Firms that grow rapidly very rarely encounter financial problems. C. The cash flows generated in a given time period are equal to the profits reported. D. Profits provide assurance that cash flow will be sufficient to maintain solvency. E. Due to required cash investments in current assets, fast-growing and profitable companies can literally "grow broke". F. None of the options are correct. Accessibility: Keyboard Navigation Difficulty: 1 Easy Gradable: automatic

15. Which one of the following correctly defines the retention ratio? A. one plus the dividend payout ratio B. additions to retained earnings divided by net income C. additions to retained earnings divided by dividends paid D. net income minus additions to retained earnings E. net income minus cash dividends F. None of the options are correct. Accessibility: Keyboard Navigation Difficulty: 1 Easy Gradable: automatic

16. Which one of the following policies most directly affects the projection of the retained earnings balance to be used on a pro forma statement? A. net working capital policy B. capital structure policy C. dividend policy D. capital budgeting policy E. capacity utilization policy F. None of the options are correct. Accessibility: Keyboard Navigation Difficulty: 1 Easy Gradable: automatic

4-3 Copyright © 2018 McGraw-Hill Education. All rights reserved. No reproduction or distribution without the prior written consent of McGraw-Hill Education.


17. Which of the following questions are appropriate to address upon conducting sustainable growth analysis and the financial planning process? I. Should the firm merge with a competitor? II. Should additional equity be sold? III. Should a particular division be sold? IV. Should a new product be introduced? A. I, II, and III only B. I, II, and IV only C. I, III, and IV only D. II, III, and IV only E. I, II, III, and IV F. None of the options are correct Accessibility: Keyboard Navigation Difficulty: 2 Medium Gradable: automatic

18. The sustainable growth rate of a firm is best described as the A. minimum growth rate achievable, assuming a 100 percent retention ratio. B. minimum growth rate achievable if the firm maintains a constant equity multiplier. C. maximum growth rate achievable, excluding external financing of any kind. D. maximum growth rate achievable, excluding any external equity financing while maintaining a constant debt-equity ratio. E. maximum growth rate achievable with unlimited debt financing. F. None of the options are correct. Accessibility: Keyboard Navigation Difficulty: 1 Easy Gradable: automatic

19. The sustainable growth rate A. assumes there is no external financing of any kind. B. assumes no additional long-term debt is available. C. assumes the debt-equity ratio is constant. D. assumes the debt-equity ratio is 1.0. E. assumes all income is retained by the firm. F. None of the options are correct. Accessibility: Keyboard Navigation Difficulty: 1 Easy Gradable: automatic

20. Which of the following can affect a firm's sustainable rate of growth? I. Asset turnover ratio II. Profit margin III. Dividend policy IV. Financial leverage A. III only B. I and III only C. II, III, and IV only D. I, II, and IV only E. I, II, III, and IV F. None of the options are correct Accessibility: Keyboard Navigation Difficulty: 2 Medium Gradable: automatic

4-4 Copyright © 2018 McGraw-Hill Education. All rights reserved. No reproduction or distribution without the prior written consent of McGraw-Hill Education.


21. Gujarat Corporation doubled its shareholders’ equity during the year 2017. Gujarat did not issue any new equity, repurchase any equity, or pay out any dividends during the year. What is Gujarat’s sustainable growth rate for 2017? A. 50% B. 100% C. 150% D. 200% If equity doubled, then g* = change in equity/equitybop = 100%. For example, if equitybop was 25, the change in equity must also be 25 in order to double equity. Accessibility: Keyboard Navigation Difficulty: 3 Hard Gradable: automatic

22. Hayesville Corporation had net income of $5 million this year on net sales of $125 million per year. At the beginning of this year, its debt-to-equity ratio was 1.5 and it held $75 million in total liabilities. It paid out $2 million in dividends for the year. What is Hayesville Corporation’s sustainable growth rate? A. 3% B. 4% C. 5% D. 6% ROEbop × Retention ratio = (5/50) × 0.6 = 6% Accessibility: Keyboard Navigation Difficulty: 2 Medium Gradable: automatic

23. Milano Corporation has experienced growth of 20% for each of the last 5 years. Over this 5-year period, Milano’s return on equity has never exceeded 15%, its profit margin has held steady at 5%, and its total asset turnover has not changed. Over the 5-year period, Milano paid no dividends and issued no new equity. Based on this information, which of the following can you most likely infer about Milano’s performance over the past 5 years? A. Milano’s leverage has decreased. B. Milano’s leverage has remained constant. C. Milano’s leverage has increased. D. None of the options are correct. Note first that g > g* because g = 20% and g*<15%. With g > g* one of PRAT must increase. P has held steady at 5%, R has remained at 100%, A has not changed. Thus T (leverage) must have increased. Accessibility: Keyboard Navigation Difficulty: 3 Hard Gradable: automatic

24. Which of the following would increase a company’s need for external finance, all else equal? A. An increase in the dividend payout ratio B. A decrease in sales growth C. An increase in profit margin D. A decrease in the collection period Accessibility: Keyboard Navigation Difficulty: 2 Medium Gradable: automatic

4-5 Copyright © 2018 McGraw-Hill Education. All rights reserved. No reproduction or distribution without the prior written consent of McGraw-Hill Education.


25. You constructed a pro forma balance sheet for next year and found that external financing required was negative (i.e., the company projected a financing surplus). Which of the following options, all else equal, would NOT correct the projected imbalance? A. A stock repurchase B. A decrease in accounts payable C. An increase in cash and marketable securities D. An increase in the retention ratio Accessibility: Keyboard Navigation Difficulty: 2 Medium Gradable: automatic

26. The sustainable growth rate A. is the highest growth rate attainable for a firm that pays no dividends. B. is the highest growth rate attainable for a firm without issuing new stock. C. can never be greater than the return on equity. D. can be increased by decreasing leverage. Accessibility: Keyboard Navigation Difficulty: 2 Medium Gradable: automatic

27. Wax Music expects sales of $437,500 next year. The profit margin is 4.8 percent, and the firm has a 30 percent dividend payout ratio. What is the projected increase in retained earnings? A. $14,700 B. $17,500 C. $18,300 D. $20,600 E. $21,000 F. None of the options are correct. Change in retained earnings = $437,500 × 0.048 × (1 − 0.30) = $14,700 Accessibility: Keyboard Navigation Difficulty: 1 Easy Gradable: automatic

28. Komatsu has a 4.5 percent profit margin and a 15 percent dividend payout ratio. The asset turnover ratio is 1.6, and the assets-to-equity ratio (using beginning-of-period equity) is 1.77. What is Komatsu’s sustainable rate of growth? A. 1.91% B. 6.12% C. 10.83% D. 11.26% E. 12.74% F. None of the options are correct. Sustainable growth = PRAT = 0.045 × (1 − 0.15) × 1.6 × 1.77 = 10.83% Accessibility: Keyboard Navigation Difficulty: 1 Easy Gradable: automatic

4-6 Copyright © 2018 McGraw-Hill Education. All rights reserved. No reproduction or distribution without the prior written consent of McGraw-Hill Education.


29. A firm has a retention ratio of 40 percent and a sustainable growth rate of 6.2 percent. Its asset turnover ratio is 0.85, and its assets-to-equity ratio (using beginning-of-period equity) is 1.80. What is its profit margin? A. 3.79% B. 5.69% C. 6.75% D. 10.13% E. 18.24% 0.062 = PRAT = profit margin × 0.40 × 0.85 × 1.80 profit margin= 0.062/(0.40 × 0.85 × 1.80) = 10.13% Accessibility: Keyboard Navigation Difficulty: 2 Medium Gradable: automatic

30. Westcomb, Inc. had equity of $150,000 at the beginning of the year. At the end of the year, the company had total assets of $195,000. During the year, the company sold no new equity. Net income for the year was $72,000, and dividends were $44,640. What is Westcomb’s sustainable growth rate? A. 15.32 percent B. 15.79 percent C. 17.78 percent D. 18.01 percent E. 18.24 percent Change in Equity = Retained earnings = $72,000 − $44,640 = $27,360 Sustainable growth rate = g* = Change in Equity/Equitybop = $27,360/$150,000 = 18.24% Alternative: g* = R × ROEbop = 72,000 − 44,640)/72,000 × 72,000/150,000 = 0.38 × 0.48 = 0.1824 Accessibility: Keyboard Navigation Difficulty: 2 Medium Gradable: automatic [The following information applies to the questions displayed below.]

Boss Stores, Inc. Selected financial information ($ millions) 2014 Sales $ 287.31 Net income 11.22 Total assets 268.58 Equity 180.63 Dividends 0.00

2015 $ 339.19 16.48 275.30 191.90 5.21

2016 $ 411.78 19.70 318.43 211.03 0.58

2017 $ 446.84 12.23 451.32 222.57 0.69

31. Please refer to the selected financial information for Boss Stores above. What is the retention ratio for 2016? A. 0.32 B. 0.68 C. 0.97 D. 1.00 E. None of the options are correct. 1 − (0.58/19.70) = 0.97 Accessibility: Keyboard Navigation Difficulty: 1 Easy Gradable: automatic

4-7 Copyright © 2018 McGraw-Hill Education. All rights reserved. No reproduction or distribution without the prior written consent of McGraw-Hill Education.


32. Please refer to the selected financial information for Boss Stores above. What is the actual sales growth rate for 2016? A. −17.6% B. −7.9% C. 8.51% D. 21.4% E. None of the options are correct. (411.78 − 339.19)/339.19 = 0.214 Accessibility: Keyboard Navigation Difficulty: 1 Easy Gradable: automatic 33. Please refer to the selected financial information for Boss Stores above. What is the sustainable growth rate for

2016? A. −17.6% B. −7.9% C. 9.97% D. 10.27% E. 12.23% F. 21.40% Sustainable growth = g* = Change in Equity/Equitybop = (211.03 − 191.90)/191.90 = 9.97% Accessibility: Keyboard Navigation Difficulty: 2 Medium Gradable: automatic

34. Please refer to the selected financial information for Boss Stores above. What is the difference between Boss’s sustainable growth rate and its actual growth rate for 2017? A. −11.40% B. −7.09% C. −3.04% D. 5.47% E. 13.98% F. 21.40% Sustainable growth = g* = PRAT = 0.0274 × 0.94 × 0.99 × 2.14 = 5.47% Alternatively, g* = Change in Equity/Equitybop = (222.57 − 211.03)/211.03 = 5.47% Actual growth = g = Change in Sales/Salesbop = (446.84 − 411.78)/411.78 = 8.51% g* − g = 5.47% − 8.51% = − 3.04% Accessibility: Keyboard Navigation Difficulty: 3 Hard Gradable: automatic

4-8 Copyright © 2018 McGraw-Hill Education. All rights reserved. No reproduction or distribution without the prior written consent of McGraw-Hill Education.


35. Which of the following actions would help a firm’s growth problem if its actual sales growth exceeds its sustainable rate of growth? I. Increase prices II. Decrease financial leverage III. Decrease dividends IV. Prune away less-profitable products

A. I and II only B. I and III only C. I, II, and IV only D. I, III, and IV only E. I, II, III, and IV F. None of the options are correct Accessibility: Keyboard Navigation Difficulty: 2 Medium Gradable: automatic

36. Which of the following is NOT a reason for why U.S. corporations haven’t issued more equity in recent years? A. Managers usually believe that their stock is overvalued. B. Companies in the aggregate had sufficient funds through profits and new debt. C. Equity is relatively expensive to issue. D. Managers try to avoid dilution of earnings per share. E. Managers perceive the stock market to be an unreliable funding source. Accessibility: Keyboard Navigation Difficulty: 1 Easy Gradable: automatic

37. Why do financial managers need to understand the implications of the sustainable rate of growth? Working capital, fixed assets, and external financing must coordinate with and be able to support a firm’s sales growth. If, for example, a projected increase in sales requires external financing when no such financing is available, then the firm cannot grow at the desired rate. Understanding the implications of the sustainable growth rate helps managers understand the need to manage growth so that the firm does not attempt to outgrow its resources. Accessibility: Keyboard Navigation Difficulty: 2 Medium Gradable: manual [The following information applies to the questions displayed below.]

Law Specialists, Inc. Selected financial information Profit margin (%) Retention ratio (%) Asset turnover (X) Financial leverage (X)

2013 6.89 100.00 2.78 1.69

2014 4.94 100.00 2.47 1.38

2015 0.11 100.00 2.03 1.16

2016 0.32 100.00 2.00 1.32

2017 5.25 78.27 2.23 1.52

Growth rate in sales (%)

22.89

- 10.05

- 28.76

3.55

26.19

4-9 Copyright © 2018 McGraw-Hill Education. All rights reserved. No reproduction or distribution without the prior written consent of McGraw-Hill Education.


38. Use Law Specialists’s selected financial information above to answer the following questions: A. Calculate Law Specialists’s sustainable growth rate in each year. b. Comparing the company’s sustainable growth rate with its actual growth rate in sales, what growth problems did the company face over this period? A. Law Specialists’s sustainable growth rates (these are the product of the first four rows of financial information):

Sustainable growth rate (%)

2013

2014

2015

2016

2017

32.37

16.84

0.26

0.84

13.93

b. From 2010 through 2012, Law Specialists’s actual growth (or decline) in sales was well below its sustainable growth rate. In 2013 and 2014, the company’s growth problems reversed, with actual growth modestly exceeding sustainable growth in 2013, and doubling sustainable growth in 2014. Accessibility: Keyboard Navigation Difficulty: 2 Medium Gradable: manual

39. Please refer to the selected financial information for Law Specialists, Inc. above. Law Specialists paid its first dividends in 2017. As an analyst, assess the company’s decision to pay dividends. Dividends are typically the hallmark of a maturing industry. The growth rates between 2014 and 2016 indicate slowing growth for the staffing services business. In this context, the company’s decision makes sense. However, in 2017 the company resumed rapid growth. Persistent growth rates above sustainable growth could jeopardize its ability to pay dividends on an ongoing basis. Accessibility: Keyboard Navigation Difficulty: 2 Medium Gradable: manual

[The following information applies to the questions displayed below.] Hard Knock Doors Selected financial information ($ thousands) 2011 Sales Net income Total assets Equity Dividen ds

$

2012

2013

491.62

706.52

-

43.27

26.31

28.58

34.84

25.76

-

477.06

648.42

664.26

697.16

982.63

-

346.32

372.63

400.41

433.6

457.14

-

-

-

0.8

1.65

2.22

477.84

$

2014 $

792.01

2015 $

876.52

2017 $

1,088.46

4-10 Copyright © 2018 McGraw-Hill Education. All rights reserved. No reproduction or distribution without the prior written consent of McGraw-Hill Education.


40. Please refer to the selected financial information for Hard Knock Doors above. Calculate the actual and sustainable growth rates for Hard Knock Doors for each year, 2013–2016. Hard Knock Doors 2013

2014 %

3.61

2015 %

3.97

2016

Profit margin

3.72

%

2.37

Retention ratio

1

0.97

0.95

0.91

Asset turnover

1.09

1.19

1.26

1.11

Financial leverage Sustainable growth (g* = PRAT) OR g* = ΔEquity/Equitybop Actual growth

1.87

1.78

1.74

2.27

%

7.6

%

7.46

%

8.29

%

5.43

%

7.6

%

7.46

%

8.29

%

5.43

%

43.71

%

12.1

%

10.67

%

24.18

% Accessibility: Keyboard Navigation Difficulty: 2 Medium Gradable: manual

41. Please refer to the selected financial information for Hard Knock Doors above. Do you think Hard Knock Doors is having a problem financing its growth? If so, what is it doing to address the problem? Hard Knock’s actual growth has exceeded its sustainable growth rate every year. The company has financed its rapid growth by increasing its asset turnover, and in 2016, its financial leverage. Accessibility: Keyboard Navigation Difficulty: 2 Medium Gradable: manual

42. Please refer to the selected financial information for Hard Knock Doors above. Is the increase in dividends between 2013 and 2016 a good idea for Hard Knock Doors? The initiation and increase in dividend payments, although modest, have exacerbated its sustainable growth problems by decreasing the retention ratio. Accessibility: Keyboard Navigation Difficulty: 2 Medium Gradable: manual

4-11 Copyright © 2018 McGraw-Hill Education. All rights reserved. No reproduction or distribution without the prior written consent of McGraw-Hill Education.


Chapter 04 Test Bank Summary Category

Accessibility: Keyboard Navigation Difficulty: 1 Easy Difficulty: 2 Medium Difficulty: 3 Hard Gradable: automatic Gradable: manual

# of Questions

42 21 18 3 36 6

4-12 Copyright © 2018 McGraw-Hill Education. All rights reserved. No reproduction or distribution without the prior written consent of McGraw-Hill Education.


Chapter 05 Test Bank 1. In the steps a company takes to prepare for an IPO, the “road show” precedes the “bake-off”. FALSE Accessibility: Keyboard Navigation Difficulty: 2 Medium Gradable: automatic

2. The only reason why the price would fall on a corporate bond is if market interest rates increase. FALSE Accessibility: Keyboard Navigation Difficulty: 1 Easy Gradable: automatic

3. After issue, the market price of a fixed-rate bond can differ substantially from its par value. TRUE Accessibility: Keyboard Navigation Difficulty: 1 Easy Gradable: automatic

4. Bond investors should be more concerned with real returns than with nominal returns. TRUE Accessibility: Keyboard Navigation Difficulty: 1 Easy Gradable: automatic

5. Investment–grade bonds are usually defined as bonds with ratings of BBB– or higher. TRUE Accessibility: Keyboard Navigation Difficulty: 1 Easy Gradable: automatic

6. Private equity firms comprise a relatively insignificant portion of the American economy. FALSE Accessibility: Keyboard Navigation Difficulty: 1 Easy Gradable: automatic

7. Shelf registration is possible for both debt and equity issues. TRUE Accessibility: Keyboard Navigation Difficulty: 1 Easy Gradable: automatic

5-1 Copyright © 2018 McGraw-Hill Education. All rights reserved. No reproduction or distribution without the prior written consent of McGraw-Hill Education.


8. In a strong-form efficient market, insider trading is not profitable. TRUE Accessibility: Keyboard Navigation Difficulty: 2 Medium Gradable: automatic

9. Principal is exchanged in interest rate swaps but not in currency swaps. FALSE Accessibility: Keyboard Navigation Difficulty: 2 Medium Gradable: automatic

10. Valuing a call option requires an accurate estimate of the future value of the underlying asset. FALSE Accessibility: Keyboard Navigation Difficulty: 2 Medium Gradable: automatic

11. Which one of the following statements is false? A. Financial executives must design financial securities to meet the needs of the firm and its investors. B. Financial instruments are subject to full disclosure requirements. C. The design of financial instruments is greatly constrained by law and regulation. D. Financial instruments are claims against a company’s cash flows and assets. E. None of the options are correct. Accessibility: Keyboard Navigation Difficulty: 1 Easy Gradable: automatic

12. Which of the following securities has a purely fixed claim against a firm’s cash flows? A. bonds B. options C. common stock D. None of the options are correct. Accessibility: Keyboard Navigation Difficulty: 1 Easy Gradable: automatic

13. Which of the following securities has a purely residual claim against a firm’s cash flows? A. preferred stock B. callable bonds C. common stock D. non-callable bonds E. None of the options are correct. Accessibility: Keyboard Navigation Difficulty: 1 Easy Gradable: automatic

5-2 Copyright © 2018 McGraw-Hill Education. All rights reserved. No reproduction or distribution without the prior written consent of McGraw-Hill Education.


14. Mike just purchased a bond which pays $40 every six months in interest. The $40 interest payment is also called the A. coupon. B. par value. C. discount. D. call premium. E. yield. F. None of the options are correct. Accessibility: Keyboard Navigation Difficulty: 1 Easy Gradable: automatic

15. A $1,000 par value bond with a fixed 10% rate of interest pays coupons semiannually. What amount will the bondholder receive on the bond’s maturity date? A. $50 B. $100 C. $500 D. $1,000 E. $1,050 F. $1,100 Accessibility: Keyboard Navigation Difficulty: 1 Easy Gradable: automatic

16. Zack owns a bond that will pay him $35 each year in interest plus a $1,000 principal payment at maturity. The $1,000 principal payment is called the A. coupon. B. par value. C. discount. D. yield. E. call premium. F. None of the options are correct. Accessibility: Keyboard Navigation Difficulty: 1 Easy Gradable: automatic

17. Which one of the following statements is true? A. Debt instruments offer residual claims to future cash payouts. B. Bonds with call provisions will have lower coupon rates than otherwise identical bonds. C. Bondholders enjoy a direct voice in company decisions. D. Bonds are low-risk investments that do well in inflationary periods. E. Preferred shareholders are the first investors to be repaid in bankruptcy liquidation. F. None of the options are correct.

Accessibility: Keyboard Navigation Difficulty: 2 Medium Gradable: automatic

5-3 Copyright © 2018 McGraw-Hill Education. All rights reserved. No reproduction or distribution without the prior written consent of McGraw-Hill Education.


18. Which one of the following accurately orders the rate of return on financial securities from highest to lowest over most of recorded market history (the 1928-2016 period)? A. Short-term government bills, long-term corporate bonds, long-term government bonds, common stocks B. Long-term corporate bonds, long-term government bonds, common stocks, short-term government bills C. Common stocks, long-term government bonds, long-term corporate bonds, short-term government bills D. Common stocks, long-term corporate bonds, long-term government bonds, short-term government bills E. Long-term corporate bonds, common stocks, short-term government bills, long-term government bonds F. None of the options are correct. Accessibility: Keyboard Navigation Difficulty: 1 Easy Gradable: automatic

19. Which one of the following statements is true? A. Equity securities offer fixed claims on future cash payouts. B. Unlike bondholders, for their returns, shareholders rely entirely on price appreciation. C. In theory, common shareholders exercise very little control over company decisions. D. Historically, common shareholders have earned a risk premium as compensation for risk borne in excess of government bonds. E. Preferred shareholders are the first investors to be repaid in bankruptcy liquidation. F. None of the options are correct. Accessibility: Keyboard Navigation Difficulty: 2 Medium Gradable: automatic

20. You bought a yen-denominated corporate bond at the beginning of the year for ¥100,000. The bond paid 3 percent annual interest and was trading for ¥110,000 at year-end. What holding period return, measured in yen, did you earn on the bond? A. 3% B. 7% C. 10% D. 13% E. 30% F. None of the options are correct. The holding period return in yen was (3% × 100,000 + 10,000)/100,000 = 13%. Accessibility: Keyboard Navigation Difficulty: 2 Medium Gradable: automatic

5-4 Copyright © 2018 McGraw-Hill Education. All rights reserved. No reproduction or distribution without the prior written consent of McGraw-Hill Education.


21. You bought a yen–denominated corporate bond at the beginning of the year for ¥100,000. The bond paid 3 percent annual interest and was trading for ¥110,000 at year-end. The exchange rate was $1 = ¥100 at the beginning of the year and $1 = ¥122 at year-end. What holding period return, measured in U.S. dollars, did you earn on the bond? A. −18.03% B. −7.38% C. −5.03% D. 3.0% E. 10.0% F. None of the options are correct. You paid $1,000 for the bond (¥100,000/100). At the end of the year, you had interest income and a yen bond worth a total of $926.23 (¥113,000/122). Your dollar return was ($926.23 − $1,000)/$1,000 = −7.38 percent. Accessibility: Keyboard Navigation Difficulty: 3 Hard Gradable: automatic

22. You bought a yen-denominated corporate bond at the beginning of the year for ¥100,000. The bond paid 3 percent annual interest and was trading for ¥110,000 at year-end. The exchange rate was $1 = ¥100 at the beginning of the year and $1 = ¥97 at year-end. What holding period return, measured in U.S. dollars, did you earn on the bond? A. 3.09% B. 6.09% C. 13% D. 16.49% E. 30% F. None of the options are correct. You paid $1,000 for the bond (¥100,000/100). At the end of the year, you had interest income and a yen bond worth a total of $1,164.95 (¥113,000/97). The U.S. dollar holding period return would be ($1,164.95 − $1,000)/$1,000 = 16.49%.

Accessibility: Keyboard Navigation Difficulty: 3 Hard Gradable: automatic

23. What is the holding period return for the year on a bond with a par value of $1,000 and a coupon rate of 8.5% if its price at the beginning of the year was $1,215 and its price at the end of the year was $1,020? Assume interest is paid annually. A. −11.00% B. −10.78% C. −9.05% D. 10.50% (85 + 1,020 − 1,215)/1,215 = −0.0905 Accessibility: Keyboard Navigation Difficulty: 1 Easy Gradable: automatic

5-5 Copyright © 2018 McGraw-Hill Education. All rights reserved. No reproduction or distribution without the prior written consent of McGraw-Hill Education.


24. Which of the following statements regarding junk bonds is true? A. Junk bonds typically offer lower yields to maturity than investment-grade bonds. B. Junk bonds have higher priority in bankruptcy than preferred stock. C. Junk bonds offer no coupon payments to investors. D. Junk bonds are typically defined as bonds with default probabilities of 25% or higher. Accessibility: Keyboard Navigation Difficulty: 3 Hard Gradable: automatic

25. Which of the following statements regarding preferred stock is true? A. Holders of preferred stock have the same voting rights as common stockholders. B. Preferred stock dividend payments are a deductible expense for corporate tax purposes. C. Almost all public corporations are at least partly financed with preferred stock. D. None of the options are correct. Accessibility: Keyboard Navigation Difficulty: 2 Medium Gradable: automatic

26. What would be the carried interest (at 20%) on a private equity portfolio with an initial value of $500 million that was subsequently liquidated for $750 million? A. $50 million B. $100 million C. $150 million D. $250 million Accessibility: Keyboard Navigation Difficulty: 1 Easy Gradable: automatic

27. Which of the following statements are true? I. Underwriters help private companies access public stock markets through IPOs. II. Shelf registrations and private placements are examples of seasoned security issues. III. Issue costs for debt are typically greater than issue costs for equity. IV. Bearer bonds make it easier for investors to avoid paying taxes on interest income.

A. I and II only B. I and III only C. I, II, and IV only D. I, III, and IV only E. I, II, III, and IV F. None of the options are correct. Accessibility: Keyboard Navigation Difficulty: 2 Medium Gradable: automatic

5-6 Copyright © 2018 McGraw-Hill Education. All rights reserved. No reproduction or distribution without the prior written consent of McGraw-Hill Education.


28. Carbon8 Corporation wants to raise $120 million in a seasoned equity offering, net of all fees. Carbon8 stock currently sells for $28.00 per share. The underwriters will require a fee of $1.25 per share, and indicate that the issue must be underpriced by 7.5%. In addition to the underwriter’s fee, the firm will incur $785,000 in legal, administrative, and other costs. How many shares must Carbon8 sell in order to raise the desired amount of capital? A. 4.3 million B. 4.5 million C. 4.6 million D. 4.9 million The price will be set at 7.5% below the current price, or at 0.925 × 28.00 = $25.90. The underwriters will take $1.25 per share, leaving $24.65/share for Carbon8. Carbon8 needs to earn $120,785,000 at $24.65/share, so it must sell 120.785mil/24.65 = 4.9 million shares. Accessibility: Keyboard Navigation Difficulty: 2 Medium Gradable: automatic

29. At the end of 2016, Crane Industries, Inc.’s stock price was $30.75. A year later, it was $34.88. Per share dividends over the year were $0.55, while earnings per share were $1.33. What rate of return did the common stockholders earn in fiscal year 2017? A. 1.79% B. 4.33% C. 13.43% D. 15.22% E. 17.76% F. None of the options are correct. Rate of return = (34.88 + 0.55 − 30.75)/30.75 = 15.22% Accessibility: Keyboard Navigation Difficulty: 2 Medium Gradable: automatic

30. At the end of 2016, Crane Industries, Inc.’s stock price was $30.75. A year later, it was $34.88. Per share dividends over the year were $0.55, while earnings per share were $1.33. What was the dividend yield in fiscal year 2017? A. 1.79% B. 4.33% C. 13.43% D. 15.22% E. 17.76% F. None of the options are correct. Dividend yield = 0.55/30.75 = 1.79% Accessibility: Keyboard Navigation Difficulty: 1 Easy Gradable: automatic

5-7 Copyright © 2018 McGraw-Hill Education. All rights reserved. No reproduction or distribution without the prior written consent of McGraw-Hill Education.


31. At the end of 2016, Crane Industries, Inc.’s stock price was $30.75. A year later, it was $34.88. Per share dividends over the year were $0.55, while earnings per share were $1.33. What was the percentage change in the share price in fiscal year 2017? A. 1.79% B. 4.33% C. 13.43% D. 15.22% E. 17.76% F. None of the options are correct. Percentage change in share price = (34.88 − 30.75)/30.75 = 4.13/30.75 = 13.43%. Accessibility: Keyboard Navigation Difficulty: 1 Easy Gradable: automatic

32. Which of the following would allow a corporation to issue a bond at a lower coupon rate, all else equal? A. The addition of a call provision to the bond B. The removal of protective covenants from the bond C. A deterioration in the corporation’s credit quality D. An increase in the expected inflation rate E. None of the options are correct. Accessibility: Keyboard Navigation Difficulty: 2 Medium Gradable: automatic

33. Which of the following statements related to market efficiency tend to be supported by current evidence? I. Markets tend to respond quickly to new information. II. It is difficult for the typical investor to earn above-average returns without taking above-average risks. III. Short-run prices are difficult to predict accurately based on public information. IV. Markets are most likely strong-form efficient. A. I and III only B. II and IV only C. I and IV only D. I, III, and IV only E. I, II, and III only F. None of the options are correct Accessibility: Keyboard Navigation Difficulty: 2 Medium Gradable: automatic

34. Individuals who continually monitor the financial markets seeking mispriced securities A. earn excess profits over the long term. B. make the markets increasingly more efficient. C. are never able to find a security that is temporarily mispriced. D. are overwhelmingly successful in earning abnormal profits. E. are always quite successful using only historical price information as their basis of evaluation. F. None of the options are correct. Accessibility: Keyboard Navigation Difficulty: 2 Medium Gradable: automatic

5-8 Copyright © 2018 McGraw-Hill Education. All rights reserved. No reproduction or distribution without the prior written consent of McGraw-Hill Education.


35. Which of the following are the most likely reasons for why a stock price might not react at all on the day that new information related to the stock issuer is released? I. Insiders knew the information prior to the announcement. II. Investors need time to digest the information prior to reacting. III. The information has no bearing on the value of the firm. IV. The information was anticipated. A. I and II only B. I and III only C. II and III only D. II and IV only E. III and IV only F. None of the options are correct Accessibility: Keyboard Navigation Difficulty: 2 Medium Gradable: automatic

36. In March, with the spot price of wheat at $5.75 per bushel, Hollywood Bakery longs 100 July wheat futures contracts (5,000 bushels each) on the CBOE at a futures price of $5.90 per bushel. In June, Hollywood Bakery closes out its futures contracts when the futures price is $5.80 per bushel. What is Hollywood Bakery’s gain (or loss) on the futures contracts? A. A gain of $50,000 B. A gain of $25,000 C. A loss of $25,000 D. A loss of $50,000 E. None of the options are correct. Gain on futures = ($5.80 − $5.90) × 500,000 bushels = −$50,000 Accessibility: Keyboard Navigation Difficulty: 2 Medium Gradable: automatic

37. Which of the following variables does NOT affect the value of a stock option? A. The predicted future price of the underlying stock B. The current price of the underlying stock C. The option’s time to maturity D. The option’s strike price E. The interest rate Accessibility: Keyboard Navigation Difficulty: 2 Medium Gradable: automatic

38. What type of financial instrument is depicted in the position diagram shown below? A. Forward sale B. Forward purchase C. Call option D. Put option Accessibility: Keyboard Navigation Difficulty: 1 Easy Gradable: automatic

5-9 Copyright © 2018 McGraw-Hill Education. All rights reserved. No reproduction or distribution without the prior written consent of McGraw-Hill Education.


39. The price of a call option tends to be lower when which of the following is higher (all else equal)? A. The expected volatility of the underlying stock B. The price of the underlying stock C. The time to maturity D. The strike price E. None of the options are correct. Accessibility: Keyboard Navigation Difficulty: 2 Medium Gradable: automatic

40. Which of the following factors, when increased, will tend to cause the value of a put to decrease (all else equal)? A. The expected volatility of the underlying stock B. The price of the underlying stock C. The time to maturity D. The strike price E. None of the options are correct. Accessibility: Keyboard Navigation Difficulty: 2 Medium Gradable: automatic

41. Suppose you purchase a call option on XYZ stock when the stock price is $81. The option premium is $3, and the strike price is $85. What is your net profit on the call option if the stock price is $89 at maturity? A. −$7 B. −$3 C. $1 D. $4 E. $5 The gain on the call is 89 − 85 = $4; less the option premium of $3 gives a net profit of $1. Accessibility: Keyboard Navigation Difficulty: 1 Easy Gradable: automatic

42. Suppose you purchase a put option on XYZ stock when the stock price is $40. The option premium is $2, and the strike price is $39. What is your net profit on the put option if the stock price is $41 at maturity? A. −$2 B. −$1 C. $0 D. $1 E. $2 The put option is out of the money at maturity, so the net profit is −$2. Accessibility: Keyboard Navigation Difficulty: 2 Medium Gradable: automatic

5-10 Copyright © 2018 McGraw-Hill Education. All rights reserved. No reproduction or distribution without the prior written consent of McGraw-Hill Education.


43. Principal amounts are usually exchanged A. in currency swaps. B. in interest rate swaps. C. in both currency swaps and interest rate swaps. D. in neither currency swaps nor interest rate swaps. Accessibility: Keyboard Navigation Difficulty: 2 Medium Gradable: automatic

44. Chapter 5 presents evidence that the average annual rate of return on common stocks over many years has exceeded the return on government bonds in the United States, while returns on common stocks have also exhibited more volatility than returns on U.S. government bonds. Suppose that last year, the realized rate of return on government bonds exceeded the return on common stocks. Your colleague suggests that "last year shows us that investors are now willing to settle for lower returns on stocks than on bonds." How would you interpret this result?

The fact that government bonds earned a higher rate of return than common stocks in one year is not evidence that investors are suddenly willing to settle for lower returns on stocks than bonds. It means that investors’ expectations were not met or, alternatively, that investors were surprised. To take on additional risk, risk-averse investors require additional expected return. But expected returns are not the same as realized returns. Because stocks and bonds are risky, their returns will fluctuate from year to year, and bonds will earn higher returns than stocks in some years. But the expected returns of common stocks should always be higher than the expected returns of government bonds. Accessibility: Keyboard Navigation Difficulty: 3 Hard Gradable: manual

45. Himmel Corp. wants to raise $100 million in a new stock issue. Its investment banker indicates that the sale of new stock will require 12 percent underpricing and a 7 percent spread. a. Assuming Himmel’s stock price does not change from its current price of $50 per share, how many shares must the company sell and at what price to the public? b. How much money will the investment banking syndicate earn on the sale? c. Is the 12 percent underpricing a cash flow? Is it a cost? If so, to whom? a.Stock Price

$50.00

− 12% underpricing

6.00

Issue price

44.00

− 7% spread

3.08

Net to company

$40.92

Desired revenue (millions) $100 Number of shares (millions)$2.444 b. Investment bankers’ revenue = $3.08 × 2.444 million = $7.528 million. c. Underpricing is not a cash flow. It is, however, an opportunity cost to current owners because it means that more shares must be sold to raise $100 million and each share will represent a smaller ownership interest in the company. Accessibility: Keyboard Navigation Difficulty: 2 Medium Gradable: manual

5-11 Copyright © 2018 McGraw-Hill Education. All rights reserved. No reproduction or distribution without the prior written consent of McGraw-Hill Education.


46. If the stock market in the United States is efficient, how do you explain the fact that some people make very high returns? Would it be more difficult to reconcile very high returns with efficient markets if the same people made extraordinary returns year after year? Earning high returns in an efficient market is like winning at roulette. In any random process, there will be winners and losers, and some winners might win big. Earning consistently high returns over time is also possible in an efficient market, just like a gambler on a lucky streak might win repeatedly at roulette. The relevant question is whether the very high returns or the length of the winning streak is inconsistent with blind luck or not.

Accessibility: Keyboard Navigation Difficulty: 2 Medium Gradable: manual

47. You believe interest rates will soon fall.

a. Would you rather own a three-year, 6 percent coupon, fixed-rate bond or an equivalent-risk, three-year, floating-rate bond currently paying 6 percent interest? b. Would your answer to (a) change if you were contemplating issuing a bond rather than owning one? If so, how? c. Would your answer to (a) change if, as an investor, you believed interest rates would soon rise? If so, why?

A. I would rather own a fixed-rate bond because the interest income I receive from a floating-rate bond will fall as interest rates decline. Equivalently, the market value of the fixed-rate bond will rise as rates fall, but that of the floating-rate bond will not. (This presumes the fixed-rate bond is not callable.) b. My answer would change. I would rather issue a floating-rate bond now because future interest payments will fall as rates decline. c. My answer would change. As an investor, I would want to hold a floating-rate bond because interest income will rise as interest rates rise. Equivalently, the price of the fixed-income bond will fall as rates rise, while that of the floating-rate bond will not.

Accessibility: Keyboard Navigation Difficulty: 2 Medium Gradable: manual

48. Houston Corp., an American company, has a payment of ¥500 million due to Osaka Corp. one year from today. At the prevailing spot rate of 100 ¥/$, this would cost Houston $5 million, but Houston faces the risk that the ¥/$ rate will fall in the coming year, so that it will end up paying a higher amount in dollar terms. To hedge this risk, Houston has two possible strategies. The first strategy is to buy ¥500 million forward today at a one-year forward rate of 98 ¥/$. The second strategy is to pay a premium of $100,000 for a one-year call option on ¥500 million at an exchange rate of 0.96 ¥/$. a. Suppose that in one year, the spot exchange rate is 95 ¥/$. What would be Houston’s net dollar cost for the payable under each strategy? b. Suppose that in one year, the spot exchange rate is 105 ¥/$. What would be Houston’s net dollar cost for the payable under each strategy? c. Which hedging strategy would you recommend to Houston Corp., if any? Why? a. In the first strategy, the net cost in dollars is ¥500 million/98 = $5.102 million. In the second strategy, Houston Corp. exercises its option to exchange at 96 ¥/$ for a cost of ¥500 million/96 = $5.208 million. Including the cost of the option, the net cost is $5.308 million. b. In the first strategy, the net cost in dollars is again ¥500 million/98 = $5.102 million. In the second strategy, Houston Corp. allows its option to expire and exchanges at the spot rate of 105 ¥/$ for a cost of ¥500 million/105 = $4.762 million. Including the cost of the option, the net cost is $4.862 million.

5-12 Copyright © 2018 McGraw-Hill Education. All rights reserved. No reproduction or distribution without the prior written consent of McGraw-Hill Education.


c. The first strategy allows Houston to lock in an exact cost without paying a premium up front. The second strategy requires a premium payment, but Houston retains the possibility of having a much lower net dollar cost if the dollar strengthens enough relative to the yen. The correct strategy for Houston depends on its risk tolerance, its expectations for exchange rate movements, and its cash availability. Accessibility: Keyboard Navigation Difficulty: 3 Hard Gradable: manual

49. ABC Corp. has an outstanding debt of $50 million on which it pays a 4 percent fixed interest rate annually. ABC just made its annual interest payment and has three years remaining until maturity. ABC wants to swap its fixed rate payments for floating rate payments. A bank offers ABC a three-year interest rate swap with annual payments in which ABC will pay LIBOR, currently at 4.2 percent, and receive a 3.8 percent fixed rate on $50 million notional principal. Suppose that LIBOR turns out to be 4.3 percent in one year, 4.4 percent in two years, and 4.5 percent in three years. Including interest payments on ABC’s outstanding debt and payments on the swap, what will be ABC’s net interest payments for the next three years? The table below shows ABC’s net interest payments for years 1 through 3. Negative signs indicate cash flows paid by ABC and positive signs indicate cash flows received by ABC, all in dollars.

YearLIBOR Outstanding Fixed leg of Floating legNet debt swap of swap payment payment 0

4.2%

1

4.3% -2,000,000

1,900,000 -2,100,000

-2,200,000

2

4.4% -2,000,000

1,900,000 -2,150,000

-2,250,000

3

-2,000,000

1,900,000 -2,200,000

-2,300,000 Accessibility: Keyboard Navigation Difficulty: 3 Hard Gradable: manual

5-13 Copyright © 2018 McGraw-Hill Education. All rights reserved. No reproduction or distribution without the prior written consent of McGraw-Hill Education.


Chapter 05 Test Bank Summary Category Accessibility: Keyboard Navigation Difficulty: 1 Easy Difficulty: 2 Medium Difficulty: 3 Hard Gradable: automatic Gradable: manual

# of Questions 49 19 24 6 43 6

5-14 Copyright © 2018 McGraw-Hill Education. All rights reserved. No reproduction or distribution without the prior written consent of McGraw-Hill Education.


1. The evidence indicates that, on average, a company’s stock price declines when it announces a new issue of equity. TRUE Accessibility: Keyboard Navigation Difficulty: 1 Easy Gradable: automatic

2. Debt financing results in lower after-tax earnings relative to equity financing. TRUE Accessibility: Keyboard Navigation Difficulty: 2 Medium Gradable: automatic

3. The interest tax shield reduces a firm’s taxes by the amount of interest on its debt. FALSE Accessibility: Keyboard Navigation Difficulty: 2 Medium Gradable: automatic

4. If the return on invested capital is greater than the after-tax interest rate, then a higher debt-to-equity ratio increases return on equity. TRUE Accessibility: Keyboard Navigation Difficulty: 2 Medium Gradable: automatic

5. The M&M irrelevance proposition assures financial managers that their choice between equity and debt financing will ultimately have no impact on firm value. FALSE Accessibility: Keyboard Navigation Difficulty: 1 Easy Gradable: automatic

6. In some instances, additional debt financing can encourage managers to act more in the interests of owners. TRUE Accessibility: Keyboard Navigation Difficulty: 1 Easy Gradable: automatic

7. If the maturity of a company’s liabilities is less than that of its assets, the company incurs a refinancing risk. TRUE Accessibility: Keyboard Navigation Difficulty: 1 Easy Gradable: automatic

6-1 Copyright © 2018 McGraw-Hill Education. All rights reserved. No reproduction or distribution without the prior written consent of McGraw-Hill Education.


8. A company incurs costs of financial distress only after declaring bankruptcy. FALSE Accessibility: Keyboard Navigation Difficulty: 1 Easy Gradable: automatic

9. When a company is in financial distress, its shareholders may have an incentive to undertake excessively risky investments. TRUE Accessibility: Keyboard Navigation Difficulty: 1 Easy Gradable: automatic

10. Inflation benefits borrowers only if the inflation is unexpected. TRUE Accessibility: Keyboard Navigation Difficulty: 1 Easy Gradable: automatic

11. Financial leverage I. increases expected ROE but does not affect its variability. II. increases breakeven sales, like operating leverage, but increases the rate of earnings per share growth once breakeven is achieved. III. is a fundamental financial variable affecting sustainable growth. IV. increases expected return and risk to owners. A. I and II only B. I and III only C. II and IV only D. II, III, and IV only E. I, II, III, and IV F. None of the options are correct. Accessibility: Keyboard Navigation Difficulty: 1 Easy Gradable: automatic

12. The best financing choice is the one that A. sets the debt-to-assets ratio equal to 1. B. trades off the tax disadvantage of debt against the signaling effects of equity. C. maximizes expected cash flows. D. ignores the false comfort of financial flexibility. E. results in the lowest possible financial distress costs. Accessibility: Keyboard Navigation Difficulty: 1 Easy Gradable: automatic

6-2 Copyright © 2018 McGraw-Hill Education. All rights reserved. No reproduction or distribution without the prior written consent of McGraw-Hill Education.


13. Homemade leverage is A. the incurrence of debt by a corporation in order to pay dividends to shareholders. B. the exclusive use of debt to fund a corporate expansion project. C. the borrowing or lending of money by individual shareholders as a means of adjusting their level of financial leverage. D. best defined as an increase in a firm’s debt-equity ratio. E. the term used to describe the capital structure of a levered firm. F. None of the options are correct. Accessibility: Keyboard Navigation Difficulty: 1 Easy Gradable: automatic

14. The basic lesson of the M&M theory is that the value of a firm is dependent upon A. the firm’s capital structure. B. the total cash flow of the firm. C. minimizing the marketed claims. D. the amount of marketed claims to that firm. E. the size of the stockholders’ claims. F. None of the options are correct. Accessibility: Keyboard Navigation Difficulty: 1 Easy Gradable: automatic

15. The term "financial distress costs" includes which of the following? I. Direct bankruptcy costs II. Indirect bankruptcy costs III. Direct costs related to being financially distressed but not bankrupt IV. Indirect costs related to being financially distressed but not bankrupt A. I only B. III only C. I and II only D. III and IV only E. I, II, III, and IV F. None of the options are correct. Accessibility: Keyboard Navigation Difficulty: 1 Easy Gradable: automatic

16. Which of the following is/are helpful for evaluating the effect of leverage on a company’s risk and potential returns? I. Estimated pro forma coverage ratios II. The recognition that financing decisions do not affect firm or shareholder value III. A range of earnings chart and proximity of expected EBIT to the breakeven value IV. A conservative debt policy that obviates the need to evaluate risk A. I only B. III only C. I and III only D. II and III only E. IV only F. None of the options are correct. Accessibility: Keyboard Navigation Difficulty: 1 Easy Gradable: automatic

6-3 Copyright © 2018 McGraw-Hill Education. All rights reserved. No reproduction or distribution without the prior written consent of McGraw-Hill Education.


17. In general, the capital structures used by non-financial U.S. firms A. typically result in debt-to-asset ratios between 60 and 80 percent. B. tend to converge to the same proportions of debt and equity. C. tend to be those that maximize the use of the firm’s available tax shelters. D. vary significantly across industries. E. None of the options are correct. Accessibility: Keyboard Navigation Difficulty: 1 Easy Gradable: automatic

18. Which of the following factors favor the issuance of debt in the financing decision? I. Market signaling II. Distress costs III. Tax benefits IV. Financial flexibility A. I and II only B. I and III only C. II and IV only D. I, II, and III only E. I, II, and IV only F. None of the options are correct. Accessibility: Keyboard Navigation Difficulty: 2 Medium Gradable: automatic

19. Which of the following factors favor the issuance of equity in the financing decision? I. Market signaling II. Distress costs III. Management incentives IV. Financial flexibility A. I and II only B. I and III only C. II and IV only D. II, III, and IV only E. I, II, and IV only F. None of the options are correct. Accessibility: Keyboard Navigation Difficulty: 2 Medium Gradable: automatic

6-4 Copyright © 2018 McGraw-Hill Education. All rights reserved. No reproduction or distribution without the prior written consent of McGraw-Hill Education.


20. Which of the following factors favor the issuance of debt in the financing decision? I. Market signaling II. Distress costs III. Management incentives IV. Financial flexibility A. I and II only B. I and III only C. II and IV only D. I, II, and III only E. I, II, and IV only F. None of the options are correct. Accessibility: Keyboard Navigation Difficulty: 2 Medium Gradable: automatic

21. Which of the following is NOT likely to be a prudent financing policy for a rapidly growing business? A. Adopt a modest dividend payout policy that enables the company to finance most of its growth internally. B. Borrow funds rather than limit growth, thereby limiting growth only as a last resort. C. Maintain a conservative leverage ratio to ensure continuous access to financial markets. D. If external financing is necessary, use debt to the point it does not affect financial flexibility. E. None of the options are correct. Accessibility: Keyboard Navigation Difficulty: 1 Easy Gradable: automatic

22. According to the pecking order theory proposed by Stewart Myers of MIT, which of the following are correct? I. For financing needs, firms prefer to first tap internal sources, such as retained profits and excess cash. II. There is an inverse relationship between a firm’s profit level and its debt level. III. Firms prefer to issue new equity rather than source external debt. IV. A firm’s capital structure is dictated by its need for external financing. A. I and III only B. II and IV only C. I, III, and IV only D. I, II, and IV only E. I, II, III, and IV F. None of the options are correct. Accessibility: Keyboard Navigation Difficulty: 2 Medium Gradable: automatic

23. Which of the following is NOT an implication of the pecking order theory of capital structure? A. On average, a firm’s stock price drops when it announces an equity issue. B. Firms may want to maintain a reserve of cash or unused borrowing capacity. C. More-profitable firms (all else equal) should have higher debt ratios. D. Firms may fail to undertake positive-NPV projects if they would have to be financed with a new issue of equity. Accessibility: Keyboard Navigation Difficulty: 2 Medium Gradable: automatic

6-5 Copyright © 2018 McGraw-Hill Education. All rights reserved. No reproduction or distribution without the prior written consent of McGraw-Hill Education.


24. Salinas Corporation has net income of $15 million per year on net sales of $90 million per year. It currently has no long-term debt but is considering a debt issue of $20 million. The interest rate on the debt would be 7%. Salinas Corp. currently faces an effective tax rate of 40%. What would be the annual interest tax shield to Salinas Corp. if it goes through with the debt issuance? A. $560,000 B. $1,400,000 C. $8,000,000 D. $20,000,000 Interest tax shield = interest rate × amount of debt × tax rate = 0.07 × 20,000,000 × 0.40 = $560,000 Accessibility: Keyboard Navigation Difficulty: 2 Medium Gradable: automatic

25. Which of the following statements regarding interest tax shields is correct? A. Taxes are reduced by the amount of a firm’s interest-bearing debt. B. Taxable income is reduced by the amount of a firm’s interest-bearing debt. C. Taxes are reduced by the amount of the interest on a firm’s debt. D. Taxable income is reduced by the amount of the interest on a firm’s debt. Accessibility: Keyboard Navigation Difficulty: 2 Medium Gradable: automatic

26. Which of the following would NOT be considered a cost of financial distress? A. Lack of interest tax shields B. Bankruptcy costs C. Excessive risk-taking by shareholders D. Loss of customers or suppliers Accessibility: Keyboard Navigation Difficulty: 2 Medium Gradable: automatic

27. When considering the impact of distress costs on capital structure, which of the following facts should lead ABC Corporation to set a higher target debt ratio than XYZ Corporation (all else equal)? A. ABC’s cash flows from operations are less volatile than XYZ’s. B. ABC is a computer software firm, and XYZ is an electric utility. C. ABC operates in a more competitive industry than XYZ. D. ABC’s assets have lower resale values than XYZ’s assets. Accessibility: Keyboard Navigation Difficulty: 2 Medium Gradable: automatic

28. According to the pecking order theory of capital structure, why do firms avoid issuing equity? A. Because fees associated with issuing new equity are so high B. Because they want to avoid dilution of earnings per share C. Because they don’t want to commit to paying dividends on the new equity D. Because equity issuance signals that managers believe their stock is overvalued, which causes the price of the stock to fall Accessibility: Keyboard Navigation Difficulty: 2 Medium Gradable: automatic

6-6 Copyright © 2018 McGraw-Hill Education. All rights reserved. No reproduction or distribution without the prior written consent of McGraw-Hill Education.


29. Under the simplifying assumptions of Modigliani and Miller, an increase in a firm’s financial leverage will A. increase the variability in earnings per share. B. reduce the operating risk of the firm. C. increase the value of the firm. D. decrease the value of the firm Accessibility: Keyboard Navigation Difficulty: 1 Easy Gradable: automatic

[The following information applies to the questions displayed below.] Expected net income after tax next year before new financing $ 40 million Sinking-fund payments due next year on existing debt $ 14 million Interest due next year on existing debt $ 15 million Company tax rate 36 % Common stock price, per share 20 Common shares outstanding 18 million 30. Please refer to the financial information for Squamish Equipment above. For next year, calculate Squamish’s times-burdencovered ratio if Squamish sells 2 million new shares at $20 a share. A. 1.03 B. 1.38 C. 1.60 D. 1.89 E. 2.10 F. None of the options are correct. EBIT = 40/(1 − 0.36) + 15 = $77.5 Times burden covered = 77.5/[15 + 14/(1 − 0.36)] = 2.10 times Accessibility: Keyboard Navigation Difficulty: 2 Medium Gradable: automatic

31. Please refer to the financial information for Squamish Equipment above. For next year, calculate Squamish’s earnings per share if Squamish sells 2 million new shares at $20 a share. A. 1.28 B. 1.39 C. 2.00 D. 2.22 E. 4.00 F. None of the options are correct. EPS = 40/(18 + 2) = $2.00 Accessibility: Keyboard Navigation Difficulty: 2 Medium Gradable: automatic

6-7 Copyright © 2018 McGraw-Hill Education. All rights reserved. No reproduction or distribution without the prior written consent of McGraw-Hill Education.


32. Please refer to the financial information for Squamish Equipment above. Calculate Squamish’s times-interest-earned ratio for next year assuming the firm raises $40 million of new debt at an interest rate of 7 percent. A. 2.00 B. 3.09 C. 3.66 D. 4.35 E. None of the options are correct. EBIT = 40/(1 − 0.36) + 15 = $77.5 Interest = 15 + 0.07(40) = $17.8 Times interest earned = 77.5/17.8 = 4.35 times Accessibility: Keyboard Navigation Difficulty: 2 Medium Gradable: automatic

33. Please refer to the financial information for Squamish Equipment above. Calculate Squamish’s times-burden-covered ratio for the next year assuming the firm raises $40 million of new debt at an interest rate of 7 percent, and that annual sinking fund payments on the new debt will equal $8 million. A. 1.01 B. 1.08 C. 1.38 D. 1.49 E. 1.95 F. None of the options are correct. EBIT = 40/(1 − 0.36) + 15 = $77.5 Interest = $15 + 0.07(40) = $17.8 Burden of interest and sinking fund before tax = 17.8 + (14 + 8)/(1 − 0.36) = $52.175 Times burden covered = 77.5/52.175 = 1.49 times Accessibility: Keyboard Navigation Difficulty: 3 Hard Gradable: automatic

34. Please refer to the financial information for Squamish Equipment above. Calculate Squamish’s earnings per share next year assuming Squamish raises $40 million of new debt at an interest rate of 7 percent. A. 1.28 B. 2.00 C. 2.12 D. 2.22 E. 3.06 F. None of the options are correct. EBIT = 40/(1 − 0.36) + 15 = $77.5 Interest = $15 + 0.07(40) = $17.8 EPS = (77.5 − 17.8)(1 - 0.36)/18 = $2.12 Accessibility: Keyboard Navigation Difficulty: 2 Medium Gradable: automatic

6-8 Copyright © 2018 McGraw-Hill Education. All rights reserved. No reproduction or distribution without the prior written consent of McGraw-Hill Education.


35. JKL Corporation has a projected times-interest-earned ratio of 4.0 for next year. What percentage could EBIT decline next year before JKL’s times-interest-earned ratio would fall below 1.0? A. 3% B. 30% C. 75% D. 90% E. 300% F. Insufficient information is provided. % EBIT can fall = (4.0 − 1)/4.0 = 0.75 Accessibility: Keyboard Navigation Difficulty: 2 Medium Gradable: automatic

36. The interest tax shield has no value when a firm has: I. no taxable income. II. debt-equity ratio of 1. III. zero debt. V. no leverage. A. I and III only B. II and IV only C. I, III, and IV only D. II, III, and IV only E. I, II, and IV only F. None of the options are correct. Accessibility: Keyboard Navigation Difficulty: 1 Easy Gradable: automatic

37. "A firm can’t use interest tax shields unless it has (taxable) income to shield." What does this statement imply for capital structure? Explain briefly, comparing the following two examples: a start-up biotech firm and an electric utility company. The corporate tax shield of debt is only valuable to a company that has taxable income to shield. This implies that companies that do not have a lot of taxable income (i.e. startups), or companies that have highly variable taxable income (e.g., younger tech or biotech firms) should not have as much debt, because the benefits of the corporate tax shield are less. The statement also implies that if a company has a lot of debt, it may exhaust its taxable income. At this stage, the marginal benefit of additional debt is zero, and the probability of financial distress grows. The (typical) utility’s more stable cash flows and profitability imply greater tax benefits, and lower costs of financial distress. (Note that the current tax code allows firms to carry forward and carry back their income/losses, for purposes of computing taxes. The argument given above still holds, with "taxable income" incorporating carryforwards or carrybacks.) Accessibility: Keyboard Navigation Difficulty: 2 Medium Gradable: automatic

6-9 Copyright © 2018 McGraw-Hill Education. All rights reserved. No reproduction or distribution without the prior written consent of McGraw-Hill Education.


[The following information applies to the questions displayed below.] Nile Holdings Selected financial information as of Dec. 31, 2017 Last year's EBIT (2014)

$

175

million

Expected EBIT (2015)

$

189.8

million

Current portion of existing long-term debt, due 2015

$

34

million

Interest due in 2015 on existing debt

$

36

million

35

%

Tax rate Common stock price per share

$

Common shares outstanding Dividends per share

50 20

$

million

2

38. Please refer to the financial information for Nile Holdings above. Nile must decide how to finance a $100 million investment. Assume Nile raises $100 million of new debt at the end of 2017 at an interest rate of 7%. a. Calculate the firm’s pro forma 2018 times-interest-earned (TIE) ratio. b. Calculate the percentage EBIT can fall (below expected EBIT) before interest coverage dips below 1.0.

a. Interest expense = $36 + 0.07($100) = $43 Pro forma TIE = EBIT/Int. Exp. = 189.8/43 = 4.41 b. EBIT can fall (189.8 − 43)/189.8 = 77.34% before interest coverage dips below 1.0. Accessibility: Keyboard Navigation Difficulty: 2 Medium Gradable: manual

39. Please refer to the financial information for Nile Holdings above. Nile must decide how to finance a $100 million investment. Assume Nile raises $100 million of new debt at the end of 2017 at an interest rate of 7%.

a. Assuming Nile must make a $20 million payment on the new debt next year, calculate the firm’s times-burdencovered ratio and times-common-covered ratio (i.e., the number of times EBIT could cover interest, principal payments, and dividends). b. As Nile’s banker, would you be comfortable loaning the company this new debt? Briefly explain why, or for what reasons you’d be comfortable or uncomfortable. a. Interest expense = $36 + 0.07($100) = $43 Upcoming payments on debt = existing + new = 34 + 20 = 54 Times burden covered = EBIT/(interest + debt pmts./(1 − t)) = 189.8/(43 + 54/(1 − 0.35)) = 1.51 Times common covered = EBIT/(interest + (debt pmts. + dividends)/(1 − t)) = 189.8/(43 + (54 + 40)/(1 − 0.35)) = 1.01 b. Debt (both principal and interest) coverage is relatively strong at 1.5 times, but if the company continues to pay its dividend, its expected ability to pay its pro forma fixed financing charges (or burden) including dividends is right at the edge. Although EBIT can fall 33.6% ((189.8 − 126.08)/189.8 = 33.6%) before TBC falls below 1, Nile has a quite low TCC (1.01) including the burden. There is risk here, and a prudent lender will require covenants that restrict dividend payouts to certain situations, if at all. Covenants (restrictions set in the debt contract) impose additional costs on the firm; this fact should be considered by management as they weigh the costs and benefits of new debt. Nile’s banker will compare Nile’s coverage ratios to the industry’s average of these ratios. If the company has strong and relatively stable CFs, and the ratios are above or within industry averages, the banker will be more comfortable with the added debt. Accessibility: Keyboard Navigation Difficulty: 3 Hard Gradable: manual

6-10 Copyright © 2018 McGraw-Hill Education. All rights reserved. No reproduction or distribution without the prior written consent of McGraw-Hill Education.


40. Please refer to the financial information for Nile Holdings above. Nile must decide how to finance a $100 million investment. Calculate next year’s earnings per share assuming Nile raises $100 million of new debt.

EBIT

189.80

Interest expense

43.00

Pre-tax income

146.80

Tax

51.38

Net income

95.42

Shares outstanding

20.00

EPS

4.77

Accessibility: Keyboard Navigation Difficulty: 2 Medium Gradable: manual

41. Please refer to the financial information for Nile Holdings above. Nile must decide how to finance a $100 million investment. Calculate next year’s times-burden-covered ratio and earnings per share if Nile sells 2 million new shares at $50 a share instead of raising new debt. Times burden covered = EBIT/(interest + debt pmts.//(1 − t)) = 189.8/(36 + 34/(1 − 0.35)) = 2.15 EBIT

189.80

Interest expense

36.00

Pre−tax income

153.80

Tax

53.83

Net income

99.97

Shares outstanding

22.00

EPS

4.54 Accessibility: Keyboard Navigation Difficulty: 2 Medium Gradable: manual

6-11 Copyright © 2018 McGraw-Hill Education. All rights reserved. No reproduction or distribution without the prior written consent of McGraw-Hill Education.


42. Please refer to the financial information for Nile Holdings above. Nile must decide how to finance a $100 million investment. Suppose Nile expects $4.52 in EPS next year if it does not go through with the investment and associated financing. As a shareholder, to satisfy its funding needs for the investment opportunity, do you prefer the company issue $100 million in new debt at an interest rate of 7% and a payment of $20 million due on the debt next year or issue 2 million shares of equity at a target price of $50? Show supporting calculations and provide arguments and potential counter-arguments for your recommendation. First compare Nile’s EPS and coverage ratios under the two scenarios. Debt Issuance: EBIT

189.80

Interest expense

43.00

Pre−tax income

146.80

Tax

51.38

Net income

95.42

Shares outstanding

20.00

EPS

4.77

Upcoming payments on debt = existing + new = 34 + 20 = 54 Times burden covered = EBIT/(interest + debt pmts./(1 − t)) = 189.8/(43 + 54/(1 − 0.35)) = 1.51 Times common covered = EBIT/(interest + (debt pmts. + dividends)/(1 − t)) = 189.8/(43 + (54 + 40)/(1 − 0.35)) = 1.01 Equity issuance: EBIT

189.80

Interest expense

36.00

Pre−tax income

153.80

Tax

53.83

Net income

99.97

Shares outstanding

22.00

EPS

4.54

Times burden covered = EBIT/(interest + debt pmts./(1 − t)) = 189.8/(36 + 34/(1 − 0.35)) = 2.15 Times common covered = EBIT/(interest + (debt pmts. + dividends)/(1 − t)) = 189.8/(36 + (34 + 44)/(1 − 0.35)) = 1.22 The EPS calculations above indicate that, while both debt and equity issuances are expected to increase EPS compared to no issuance and subsequent investment, the debt issuance results in the highest EPS, although the higher EPS is accompanied by a higher level or risk. Depending on the volatility of the company’s EBIT, the issuance of debt presents risk to the shareholders. If the company issues debt and its EBIT falls below $126 million [$43 + 54/(1 − 0.35)], then the company could default on its borrowings. On the other hand, if financing needs were met with equity, EBIT could fall to $88 million [$36 + 34/(1 − 0.35)] and Nile could still make the interest and loan payments. Defaulting on its loans would cause a catastrophic decrease in Nile’s stock price. Raising equity poses much less risk of financial distress in this case. The issuance of debt also presents the risk that the company will be forced to miss (or reduce) future dividend payments, causing the stock price to fall. As indicated in Chapter 6, debt can increase shareholders’ returns in good times, but decrease them in bad times; i.e., the increase in expected return comes at the cost of higher risk. Management must weigh these trade−offs. Other considerations: ● The issuance of equity has a negative signaling effect. Evidence indicates that the stock market on average reacts negatively to announcements of secondary, or seasoned public stock offerings (SEOs). Debt avoids this result. ● In fact, debt issuance often has positive signaling and managerial incentive effects. ● Equity issuance retains more future financing flexibility. Accessibility: Keyboard Navigation Difficulty: 3 Hard Gradable: manual

6-12 Copyright © 2018 McGraw-Hill Education. All rights reserved. No reproduction or distribution without the prior written consent of McGraw-Hill Education.


43. Can a company incur costs of financial distress without ever going bankrupt? Explain. What is the nature of these costs? Yes. As discussed in the text, there are costs of financial distress even before the company actually defaults on the debt. These costs arise as the company, customers, and suppliers become aware that there is a possibility the company may default soon. These costs may include managers’ time and effort, disruption of supplies and customer service, drastic sales decreases for sellers of durable goods, and delay of research or capital improvement projects. The firm’s competitors can also sense a firm in trouble, and could engage in predatory pricing to weaken or eliminate the firm as a competitor. Moreover, detrimental conflicts of interest among owners, creditors, and managers can also arise when a company gets into financial difficulty. Accessibility: Keyboard Navigation Difficulty: 2 Medium Gradable: manual

44. Kahuku Corporation has 100 million shares outstanding trading at $20 per share. The company announces its intention to raise $150 million by selling new shares. a. What do market signaling studies suggest will happen to Kahuku’s stock price on the announcement date? Why? b. How large a gain or loss in aggregate dollar terms do market signaling studies suggest existing Kahuku shareholders will experience on the announcement date? c. What percentage of the value of Kahuku’s existing equity prior to the announcement is this expected gain or loss? d. At what price should Kahuku expect its existing shares to sell immediately after the announcement? a. Market signaling studies suggest that the price of existing Kahuku shares will fall. One explanation for the decline is that managers know more about their company than outsiders do and that the announcement of an equity sale signals they are worried about the company’s prospects. Alternatively, they believe the company’s stock is overvalued at the current price. b. Expected loss = 30% of issue size = 0.3 × ($150 million) = $45 million. c. 45/($20 × 100) = 2.25 percent d. Price per share = $20 × (1 − 0.025) = $19.50. Accessibility: Keyboard Navigation Difficulty: 2 Medium Gradable: manual

45. An all-equity business has 200 million shares outstanding selling for $30 a share. Management believes that interest rates are unreasonably low and decides to execute a leveraged recapitalization (a recap). It will raise $750 million in debt and repurchase 25 million shares. a. What is the market value of the firm prior to the recap? What is the market value of equity? b. Assuming the irrelevance proposition holds, what is the market value of the firm after the recap? What is the market value of equity? c. Do equity shareholders appear to have gained or lost as a result of the recap? Please explain. a. Because there is no debt outstanding, Firm value = Equity value = 200 million × $30 = $6 billion. b. Firm value is unchanged at $6 billion. Debt outstanding is $0.75 billion, so equity value is $5.25 billion. (Equity value = firm value − debt value.) c. Equity investors neither gain nor lose from the recap. Before the recap, equity value is $6 billion for 200 million shares, or $30 per share. After the recap, equity value is $5.25 billion for 175 million remaining shares, or $30 per share. (Equity investors that sold shares in the repurchase also received $30 cash for each share.) Accessibility: Keyboard Navigation Difficulty: 2 Medium Gradable: manual

6-13 Copyright © 2018 McGraw-Hill Education. All rights reserved. No reproduction or distribution without the prior written consent of McGraw-Hill Education.


Chapter 06 Test Bank Summary Category

Accessibility: Keyboard Navigation Difficulty: 1 Easy Difficulty: 2 Medium Difficulty: 3 Hard Gradable: automatic Gradable: manual

# of Questions

45 17 25 3 36 9

6-14 Copyright © 2018 McGraw-Hill Education. All rights reserved. No reproduction or distribution without the prior written consent of McGraw-Hill Education.


Chapter 07 Test Bank 1. The accounting rate of return is deficient as a figure of merit because it is insensitive to the timing of cash flows. TRUE Accessibility: Keyboard Navigation Difficulty: 1 Easy Gradable: automatic

2. The payback period measures the amount of time the company must wait to recoup its initial investment. TRUE Accessibility: Keyboard Navigation Difficulty: 1 Easy Gradable: automatic

3. Inflation is one reason why a dollar today is worth more than a dollar in the future. TRUE Accessibility: Keyboard Navigation Difficulty: 1 Easy Gradable: automatic

4. The IRR and NPV always yield the same investment recommendations. FALSE Accessibility: Keyboard Navigation Difficulty: 1 Easy Gradable: automatic

5. When evaluating investments under capital rationing that are independent and can be acquired fractionally, ranking by the BCR is the appropriate technique. TRUE Accessibility: Keyboard Navigation Difficulty: 2 Medium Gradable: automatic

6. When conducting a discounted cash flow analysis of a project, it is important to always include a careful estimate of financing costs in the project’s cash flows. FALSE Accessibility: Keyboard Navigation Difficulty: 1 Easy Gradable: automatic

7. The IRR is the discount rate at which an investment’s NPV equals its initial cost. FALSE Accessibility: Keyboard Navigation Difficulty: 1 Easy Gradable: automatic

7-1 Copyright © 2018 McGraw-Hill Education. All rights reserved. No reproduction or distribution without the prior written consent of McGraw-Hill Education.


8. The present value of a perpetuity can be calculated as the annual cash flow divided by the discount rate. TRUE Accessibility: Keyboard Navigation Difficulty: 1 Easy Gradable: automatic

9. As a noncash expense, depreciation is irrelevant in the determination of a project’s cash flows. FALSE Accessibility: Keyboard Navigation Difficulty: 2 Medium Gradable: automatic

10. Sunk costs should be included in the cash flows for valuing a project only if they are directly attributable to that specific project. FALSE Accessibility: Keyboard Navigation Difficulty: 1 Easy Gradable: automatic

11. Which of the following is NOT an important step in the financial evaluation of an investment opportunity? A. Calculate a figure of merit for the investment. B. Estimate the accounting rate of return for the investment. C. Estimate the relevant cash flows. D. Compare the figure of merit to an acceptance criterion. E. All of the above are important steps. Accessibility: Keyboard Navigation Difficulty: 1 Easy Gradable: automatic

12. Which of the following figures of merit might not use all possible cash flows in its calculations? I. Payback period II. Internal rate of return III. Net present value (NPV) IV. Benefit-cost ratio A. III only B. I & III only C. II & III only D. I only E. III & IV only F. I, II, III, and IV Accessibility: Keyboard Navigation Difficulty: 2 Medium Gradable: automatic

7-2 Copyright © 2018 McGraw-Hill Education. All rights reserved. No reproduction or distribution without the prior written consent of McGraw-Hill Education.


13. Which of the following figures of merit does not directly take into consideration the time value of money? I. Payback period II. Internal rate of return III. Net present value (NPV) IV. Accounting rate of return A. IV only B. I & III only C. II & III only D. I & II only E. I & IV only F. I, II, III, and IV Accessibility: Keyboard Navigation Difficulty: 2 Medium Gradable: automatic

14. Ian is going to receive $20,000 six years from now. Sunny is going to receive $20,000 nine years from now. Which one of the following statements is correct if both Ian and Sunny apply a 7-percent discount rate to these amounts? A. The present values of Ian and Sunny’s monies are equal. B. In future dollars, Sunny’s money is worth more than Ian's money. C. In today’s dollars, Ian’s money is worth more than Sunny’s. D. Twenty years from now, the value of Ian’s money will be equal to the value of Sunny’s money. E. Sunny’s money is worth more than Ian’s money given the 7-percent discount rate. F. None of the options are correct. Accessibility: Keyboard Navigation Difficulty: 2 Medium Gradable: automatic

15. Which of the following is NOT a reason why a dollar today is worth more than a dollar in the future? A. Inflation reduces the purchasing power of future dollars. B. The value of a dollar in the future will be compounded more than the value of a dollar today. C. There is more uncertainty of receiving dollars further into the future. D. A dollar today can be productively invested in the time before receiving a dollar in the future. Accessibility: Keyboard Navigation Difficulty: 1 Easy Gradable: automatic

7-3 Copyright © 2018 McGraw-Hill Education. All rights reserved. No reproduction or distribution without the prior written consent of McGraw-Hill Education.


16. You plan to buy a new Mercedes four years from now. Today, a comparable car costs $82,500. You expect the price of the car to increase by an average of 4.8 percent per year over the next four years. How much will your dream car cost by the time you are ready to buy it? A. $98,340.00 B. $98,666.67 C. $99,517.41 D. $99,818.02 E. $100,023.16 F. None of the options are correct. Future value = $82,500 × (1 + 0.048)4 = $99,517.41

Input:

4

4.8

−82,500

0

?

n

i

PV

PMT

FV

Output:

99,517.41

In Excel: = FV(0.048, 4, 0, −82500) = 99,517.41 Accessibility: Keyboard Navigation Difficulty: 1 Easy Gradable: automatic

17. Your grandmother invested a lump sum 26 years ago at 4.25-percent interest. Today, she gave you the proceeds of that investment which totaled $51,480.79. How much did she originally invest? A. $15,929.47 B. $16,500.00 C. $17,444.86 D. $17,500.00 E. $17,999.45 F. None of the options are correct. Present value = $51,480.79 ÷ (1 + 0.0425)26 = $17,444.86 Input: Output:

26

4.25

n

i

?

0

51,480.79

PV

PMT

FV

−17,444.86

In Excel: = PV(0.0425, 26, 0, 51480.79) = -17,444.86 Accessibility: Keyboard Navigation Difficulty: 1 Easy Gradable: automatic

7-4 Copyright © 2018 McGraw-Hill Education. All rights reserved. No reproduction or distribution without the prior written consent of McGraw-Hill Education.


18. Naomi plans on saving $3,000 a year and expects to earn an annual rate of 10.25 percent. How much will she have in her account at the end of 45 years? A. $1,806,429 B. $1,838,369 C. $2,211,407 D. $2,333,572 E. $2,508,316 F. None of the options are correct.

Input:

45

10.25

0

−3,000

n

i

PV

PMT

Output:

? FV 2,333,572

In Excel:

= FV(0.1025, 45, −3000) = 2,333,572 Accessibility: Keyboard Navigation Difficulty: 1 Easy Gradable: automatic

19. You are the beneficiary of a life insurance policy. The insurance company informs you that you have two options for receiving the insurance proceeds. You can receive a lump sum of $200,000 today or receive payments of $1,400 a month for 20 years. You can earn a 6-percent annual rate on your money, compounded monthly. Which option should you take and why? A. You should accept the monthly payments because they are worth $209,414 to you. B. You should accept the $200,000 lump sum because the monthly payments are only worth $16,057 to you today. C. You should accept the monthly payments because they are worth $336,000 to you. D. You should accept the $200,000 lump sum because the monthly payments are only worth $189,311 to you today. E. You should accept the $200,000 lump sum because the monthly payments are only worth $195,413 to you today. F. None of the options are correct. The number of monthly periods = 20 × 12 = 240 The monthly interest rate = 6%/12 = 0.5% Input:

240

0.5

?

1,400

0

n

i

PV

PMT

FV

Output:

−195,413

In Excel:

= PV(0.005, 240, 1400) = −195,413 Accessibility: Keyboard Navigation Difficulty: 1 Easy Gradable: automatic

7-5 Copyright © 2018 McGraw-Hill Education. All rights reserved. No reproduction or distribution without the prior written consent of McGraw-Hill Education.


20. Your brother will borrow $17,800 to buy a car. The terms of the loan call for monthly payments for 5 years at an 8.6-percent annual interest rate, compounded monthly. What is the amount of each payment? A. $287.71 B. $296.67 C. $301.12 D. $342.76 E. $366.05 F. None of the options are correct. The number of monthly periods = 5 × 12 = 60 The monthly interest rate = 8.6%/12 = 0.71667% Input:

60

0.7167

17,800

?

0

n

i

PV

PMT

FV

−366.05

Output: In Excel:

= PMT(0.0071667, 60, 17800) = −366.05 Accessibility: Keyboard Navigation Difficulty: 1 Easy Gradable: automatic

21. EAC Nutrition offers a 9.5-percent coupon bond with annual payments maturing 11 years from today. Your required return is 11.2 percent. What price are you willing to pay for this bond if the face (or par) value is $1,000? A. $895.43 B. $896.67 C. $941.20 D. $946.18 E. $953.30 F. None of the options are correct. Price = present value of coupons and face value Coupon payment = 0.095 × 1000 = $95 per year Input: Output:

11

11.2

?

95

1,000

n

i

PV

PMT

FV

−895.43

In Excel:

= PV(0.112, 11, 95, 1000) = −895.43 Accessibility: Keyboard Navigation Difficulty: 1 Easy Gradable: automatic

7-6 Copyright © 2018 McGraw-Hill Education. All rights reserved. No reproduction or distribution without the prior written consent of McGraw-Hill Education.


22. A project will produce after-tax operating cash inflows of $3,200 a year for 5 years. The after-tax salvage value of the project is expected to be $2,500 in year 5. The project’s initial cost is $9,500. What is the net present value of this project if the required rate of return is 16 percent? A. −$311.02 B. $2,168.02 C. $4,650.11 D. $9,188.98 E. $21,168.02 F. None of the options are correct. Solve for the PV of the cash inflows, and then subtract the initial investment: Input: Output:

5

16

?

3,200

2,500

n

i

PV

PMT

FV

−11,668.02

In Excel:

= PV(0.16, 5, 3200, 2500) = −11,668.02 NPV = 11668.02 − 9,500 = $2,168.02 Accessibility: Keyboard Navigation Difficulty: 2 Medium Gradable: automatic

23. Which of the following should be included in the cash flow projections for a new product? I. Money already spent for research and development of the new product II. Capital expenditures for equipment to produce the new product III. Increase in working capital needed to finance sales of the new product IV. Interest expense on the loan used to finance the new product launch

A. II and III only B. II and IV only C. I, II, and III only D. II, III, and IV only E. I, II, III, and IV F. None of the options are correct. Accessibility: Keyboard Navigation Difficulty: 2 Medium Gradable: automatic

7-7 Copyright © 2018 McGraw-Hill Education. All rights reserved. No reproduction or distribution without the prior written consent of McGraw-Hill Education.


24. Pro forma free cash flows for a proposed project should I. exclude the cost of employing existing assets that could be sold anyway. II. exclude interest expense. III. include the depreciation tax shield related to the project. IV. exclude any required increase in operating current assets.

A. I and II only B. II and III only C. II and IV only D. I, III, and IV only E. I, II, III, and IV F. None of the options are correct. Accessibility: Keyboard Navigation Difficulty: 2 Medium Gradable: automatic

25. Which of the following statements related to the internal rate of return (IRR) are correct? I. The IRR is the discount rate at which an investment’s NPV equals zero. II. An investment should be undertaken if the discount rate exceeds the IRR. III. The IRR tends to be used more than net present value simply because its results are easier to comprehend. IV. The IRR is the best tool available for deciding between mutually exclusive investments.

A. I and II only B. I and III only C. II and III only D. I, II, and IV only E. I, II, III, and IV F. None of the options are correct. Accessibility: Keyboard Navigation Difficulty: 2 Medium Gradable: automatic

26. You plan to pay $50 for a share of preferred stock that pays a $2.40 dividend per year forever. What annual rate of return will you realize? A. 0.48% B. 2.40% C. 4.80% D. 5.10% E. 20.83% F. None of the options are correct. r = A/P = $2.40/$50 = 4.80% Accessibility: Keyboard Navigation Difficulty: 1 Easy Gradable: automatic

7-8 Copyright © 2018 McGraw-Hill Education. All rights reserved. No reproduction or distribution without the prior written consent of McGraw-Hill Education.


27. Sol’s Sporting Goods is expanding and, as a result, expects additional operating cash flows of $26,000 a year for 4 years. This expansion requires $39,000 in new fixed assets. These assets will be worthless at the end of the project. In addition, the project requires an additional $3,000 of net working capital throughout the life of the project; Sol expects to recover this amount at the end of the project. What is the net present value of this expansion project at a 16-percent required rate of return? A. $18,477.29 B. $21,033.33 C. $28,288.70 D. $29,416.08 E. $32,409.57 F. None of the options are correct. The initial investment consists of the fixed assets and incremental working capital: $39,000 + $3000 = $42,000. The working capital amount is recovered at the end of year 4. Solve for the PV of the cash inflows, and then subtract the initial investment: Input:

4

16

n

i

Output:

?

26,000

3,000

PV

PMT

FV

−74,409.57

In Excel:

= PV(0.16, 5, 3200, 2500) = −74,409.57 NPV = 74,409.57 − 42,000 = $32,409.57 Accessibility: Keyboard Navigation Difficulty: 2 Medium Gradable: automatic

28. What is the benefit-cost ratio for an investment with the following cash flows at a 14.5-percent required return? Year 0 1 2 3

Cash Flow $(46,500) $ 12,200 $ 38,400 $ 11,300

A. 0.94 B. 0.98 C. 1.02 D. 1.06 E. 1.11 F. None of the options are correct. PVinflows = (12,200/1.145) + (38,400/1.1452) + (11,300/1.1453) = $47,472.78 BCR = $47,472.78/$46,500 = 1.02 Accessibility: Keyboard Navigation Difficulty: 1 Easy Gradable: automatic

7-9 Copyright © 2018 McGraw-Hill Education. All rights reserved. No reproduction or distribution without the prior written consent of McGraw-Hill Education.


29. When making a capital budgeting decision, which of the following is/are NOT relevant? I. The size of a cash flow II. The risk of a cash flow III. The accounting earnings from a cash flow IV. The timing of a cash flow A. I only B. II only C. III only D. II and III only E. III and IV only F. They are all relevant. Accessibility: Keyboard Navigation Difficulty: 1 Easy Gradable: automatic

30. Giant Corp. is considering a project that requires a $1,500 initial cost for a new machine that will be depreciated straight line to a salvage value of 0 on a 5-year schedule. The project will require a one-time increase in the level of net working capital of $300. The project will generate an additional $1,600 in revenues and $700 in operating expenses each year. The project will end at the end of year 2, at which time the machinery is expected to be sold for $800. Giant’s tax rate is 50%. In a discounted cash flow analysis of this project, what would be the projected Year 0 free cash flow? A. −$1,200 B. −$1,500 C. −$1,800 D. −$2,100 Accessibility: Keyboard Navigation Difficulty: 1 Easy Gradable: automatic

[The following information applies to the questions displayed below.]

0 Revenue Expense Depreciation Pre-tax profit Taxes After-tax profit Plant and equipment Increased working capital

$1,500 300

Free cash

−$1,800

flow

1 $ 1,600 700 300 600 300 300

2 $1,600 700 300 600 300 300 −850 −300

$ 600

$1,750

7-10 Copyright © 2018 McGraw-Hill Education. All rights reserved. No reproduction or distribution without the prior written consent of McGraw-Hill Education.


31. In a discounted cash flow analysis of Giant Corp.’s project described in the problem above, what would be the projected Year 1 free cash flow? A. $300 B. $600 C. $750 D. $900

0 Revenue Expense Depreciation Pre-tax profit Taxes After-tax profit Plant and equipment Increased working capital Free cash flow

$1,500 300 −$1,800

1 $1,600 700 300 600 300 300

$ 600

2 $1,600 700 300 600 300 300 −850 −300 $1,750 Accessibility: Keyboard Navigation Difficulty: 2 Medium Gradable: automatic

32. In a discounted cash flow analysis of Giant Corp.’s project described in the problem above, what would be the projected Year 2 free cash flow? A. $1,300 B. $1,450 C. $1,700 D. $1,750 0 Revenue Expense Depreciation Pre-tax profit Taxes After-tax profit Plant and equipment Increased working capital Free cash flow

$1,500 300 −$1,800

1 $1,600 700 300 600 300 300

$ 600

2 $1,600 700 300 600 300 300 −850 −300 $1,750

(Note that the salvage value from the equipment includes the $800 resale value plus $50 tax savings from the $100 loss on the equipment relative to book value.) Accessibility: Keyboard Navigation Difficulty: 3 Hard Gradable: automatic

7-11 Copyright © 2018 McGraw-Hill Education. All rights reserved. No reproduction or distribution without the prior written consent of McGraw-Hill Education.


33. A divisional manager submitted a project proposal to the chief financial officer, complete with a calculated NPV for the project. The chief financial officer studied the proposal and pointed out that the divisional manager had failed to account for a one-time increase in net working capital of $60,000 that will be required over the life of the seven-year project. Assuming the full value of net working capital will be recovered at the end of the project, how will the project’s NPV change after making the chief financial officer’s adjustment? Assume a discount rate of 9%. A. The NPV will decrease by $16,411. B. The NPV will decrease by $32,822. C. The NPV will decrease by $60,000. D. The NPV will not be affected. E. None of the options are correct. In Year 0 there is a $60,000 outflow. In Year 5 there is a $60,000 inflow, which has a present value of 60,000/1.097 = $32,822. The decrease in NPV is 60,000 − 32,822 = $27,178. Accessibility: Keyboard Navigation Difficulty: 2 Medium Gradable: automatic

34. You are to receive an annuity of $1,000 per year for 10 years. You will receive the first payment two years from today. At a discount rate of 10%, what is the present value of this annuity? A. $5,078.15 B. $5,585.97 C. $6,144.57 D. $6,759.03 Input:

10

10

?

1,000

0

n

i

PV

PMT

FV

Output:

−6,144.57

In Excel:

= PV(0.1, 10, 1000) = −6,144.57 The PV = $6,144.57. But the first payment is received in two years, not one year, so discount the PV by one more year: 6,144.57/1.1 = $5,585.97 Accessibility: Keyboard Navigation Difficulty: 3 Hard Gradable: automatic

7-12 Copyright © 2018 McGraw-Hill Education. All rights reserved. No reproduction or distribution without the prior written consent of McGraw-Hill Education.


35. TTT Corporation reported earnings per share of $2.52 in 2012 and $3.15 in 2017. At what compound annual rate did earnings per share grow over this period? A. 3.79% B. 4.00% C. 4.56% D. 5.00% E. None of the options are correct. Input:

5

?

−2.52

0

3.15

n

i

PV

PMT

FV

Output:

4.56

In Excel:

= RATE(5, 0, −2.52, 3.15) = 4.56% Accessibility: Keyboard Navigation Difficulty: 2 Medium Gradable: automatic

36. What is the difference in the value of a $5,000 annual perpetuity and an annuity of $5,000 for 100 years? Assume that the discount rate is 8% and that cash flows are received at the end of the year. A. $28 B. $656 C. $1,656 D. $5,000 The present value of the perpetuity is 5,000/0.08 = $62,500. For the annuity: Input: Output:

100

8

?

5,000

0

n

i

PV

PMT

FV

−62,472

In Excel:

= PV(0.08, 100, 5000) = −62,472 The difference = 62,500 − 62,472 = $28 Accessibility: Keyboard Navigation Difficulty: 2 Medium Gradable: automatic

7-13 Copyright © 2018 McGraw-Hill Education. All rights reserved. No reproduction or distribution without the prior written consent of McGraw-Hill Education.


37. Given the spreadsheet below, what value would Excel return if you entered the following formula? = NPV(B2,B5:D5)

Discount rate Cash flows

9% $−250

$500

$500

A. $577.57 B. $629.56 C. $750.00 D. #VALUE! −250/1.09 + 500/1.092 + 500/1.093 = $577.57 Accessibility: Keyboard Navigation Difficulty: 3 Hard Gradable: automatic

38. An investment costing $100,000 promises an after-tax cash flow of $36,000 per year for 6 years. a. Find the investment’s accounting rate of return and its payback period. b. Find the investment’s net present value at a 15-percent discount rate. c. Find the investment’s benefit-cost ratio (profitability index) at a 15-percent discount rate. d. Find the investment’s internal rate of return. e. Assuming the required rate of return on the investment is 15 percent, which of the above figures of merit indicate the investment is attractive? Which indicate it is unattractive? a. The accounting rate of return = 36/100 = 36% The payback period = 100/36 = 2.78 years b. NPV = −100,000 + 136,241 = $36,241 Input:

6

15

?

36,000

0

n

i

PV

PMT

FV

−136,241

Output: In Excel:

= RATE(6, 36000, −100000) = 27.70% In Excel: = PV(0.15, 6, 36000) = −136,241 c. BCR, or PI = 136,241/100,000 = 1.362 d. IRR = 27.70% Input: Output:

6

?

−100,000

36,000

0

n

i

PV

PMT

FV

27.7

7-14 Copyright © 2018 McGraw-Hill Education. All rights reserved. No reproduction or distribution without the prior written consent of McGraw-Hill Education.


In Excel: = RATE(6, 36000, −100000) = 27.70% e. The NPV, PI, and IRR all indicate unambiguously that the investment is worthwhile. The accounting rate of return suggests the same, but is not a reliable indicator of investment value, nor is it comparable to the 15 percent required rate of return on the investment. Because there is no reliable way to transform a 15 percent required rate of return into a maximum acceptable payback period, it is not clear whether a 2.78 year payback period is short enough. Accessibility: Keyboard Navigation Difficulty: 2 Medium Gradable: manual

39. At $1,000 par value, a 10-percent coupon bond matures in 20 years. If the price of the bond is $1,196.80, what is the yield to maturity on the bond? Assume interest is paid annually. Input:

20

?

−1,196.80

100

1,000

n

i

PV

PMT

FV

Output:

8

In Excel:

= RATE(20, 100, −1196.8, 1000) = 8.00% Accessibility: Keyboard Navigation Difficulty: 1 Easy Gradable: manual

40. Ten years ago, you invested $1,000 for 10 shares of Providien, Inc. common stock. You sold the shares recently for $2,000. While you owned the stock, it paid $10.08 per share in annual dividends. What was your rate of return on Providien stock? Total dividends per year = 10 shares × $10.08 per share = $100.80. Rate of return = 15.00%, as shown below. Input:

10 n

? i

Output:

−1,000 PV 15.00

100.80 PMT

2,000 FV

In Excel:

= RATE(10, 100.8, −1000, 2000) = 15.00% Accessibility: Keyboard Navigation Difficulty: 2 Medium Gradable: manual

7-15 Copyright © 2018 McGraw-Hill Education. All rights reserved. No reproduction or distribution without the prior written consent of McGraw-Hill Education.


41. Consider the following investment opportunity.

Initial cost Annual revenues Annual operating costs, excluding depreciation Expected life (years) Salvage value after taxes Annual depreciation for tax purposes Tax rate

$ 850,000 $500,000 $200,000 20 $ 40,000 $ 25,000 34%

Assume the annual figures are unchanged for the expected life of the investment. What is the rate of return on this investment? Assuming the investor wants to earn at least 12 percent, is this investment an attractive one? The annual cash flows for the investment are:

Revenues Operating costs Depreciation Profit before tax Taxes Profit after tax Depreciation

$500,000 200,000 25,000 275,000 93,500 181,500 25,000

Cash flow after tax

206,500

And the rate of return = 23.96% as shown below. Input: Output:

20

?

−850,000

206,500

0

n

i

PV

PMT

FV

23.96

In Excel:

= RATE(20,206500,−850000,,,0.2) = 23.96% The investment is clearly attractive at a hurdle rate of 12%. (Note that Excel requires a guess to be entered in order to solve for the IRR in this case.) Accessibility: Keyboard Navigation Difficulty: 2 Medium Gradable: manual

7-16 Copyright © 2018 McGraw-Hill Education. All rights reserved. No reproduction or distribution without the prior written consent of McGraw-Hill Education.


42. A company is considering two alternative methods of producing a new product. The relevant data concerning the alternatives are presented below.

Alternative I $ $ $ $

Initial investment Annual receipts Annual disbursements Annual depreciation Expected life (years) Salvage value

64,000 50,000 20,000 16,000 4 0

$

Alternative II $120,000 $ 60,000 $ 12,000 $ 20,000 6 $ 0

At the end of the useful life of whatever equipment is chosen, the product will be discontinued. The company’s tax rate is 50 percent, and its cost of capital is 10 percent. a. Calculate the net present value of each alternative. b. Calculate the benefit-cost ratio for each alternative. c. Calculate the internal rate of return for each alternative. d. If the company is not under capital rationing, which alternative should be chosen? Why? a. The annual cash flows for each alternative are as follows:

Alternative I $ 50,000 $ 20,000 $ 16,000 $ 14,000 $ 7,000 $ 7,000 $ 16,000 $ 23,000

Receipts Disbursements Depreciation Income before tax Taxes Income after tax Depreciation Cash flow after tax

Alternative II $ 60,000 $ 12,000 $ 20,000 $ 28,000 $ 14,000 $ 14,000 $ 20,000 $ 34,000

Alternative I PV: Input: Output:

4

10

?

23,000

0

n

i

PV

PMT

FV

−72,906.91

In Excel: = PV(0.1, 6, 34000) = −148,078.86 NPVI = −64,000 + 72,907 = $8,907 NPVII = −120,000 + 148,079 = $28,079

7-17 Copyright © 2018 McGraw-Hill Education. All rights reserved. No reproduction or distribution without the prior written consent of McGraw-Hill Education.


b. BCRI = 72,907 ÷ 64,000 = 1.14 BCRII = 148,079 ÷ 120,000 = 1.23 c. IRRI = 16.3% as shown below: Input:

4

?

−64,000

23,000

0

n

i

PV

PMT

FV

Output:

16.3

In Excel: = RATE(4, 23000, −64000) = 16.3% IRRII = 17.6% as shown below: Input: Output:

6

?

−120,000

34,000

0

n

i

PV

PMT

FV

17.6

In Excel: = RATE(6, 34000, −120000) = 17.6% d. The company should choose Alternative II because II has the higher NPV and NPV is a direct measure of increase in shareholder wealth. Accessibility: Keyboard Navigation Difficulty: 3 Hard Gradable: manual

43. Given the following information about a possible average-risk, new product investment, calculate the investment’s net present value.

Initial cost Expected life (years) Salvage value Annual depreciation Incremental annual sales Incremental annual production costs Incremental annual selling and administrative costs Tax rate Expected inventory turnover (Prod. cost/End. inv.) Expected collection period (days) Cost of capital Borrowing rate Target debt-to-equity ratio

$ $ $ $ $ $

200,000 10 0 20,000 200,000 110,000 20,000 50% 4 45 8% 7% 130%

7-18 Copyright © 2018 McGraw-Hill Education. All rights reserved. No reproduction or distribution without the prior written consent of McGraw-Hill Education.


Inventory = Production cost/Inventory turnover = 110,000/4 = $27,500 Accounts receivable = 45(200,000/365) = $25,000 Working capital required = 27,500 + 25,000 = $52,500 Total Year 0 cost = 200,000 + 52,500 = $252,500

Annual after-tax cash flow:

Revenue Production costs Selling and administrative expense Depreciation Profit before tax Tax Profit after tax Depreciation Cash flow after tax

Input:

10 n

Output:

8 i

? PV −326,271

$

$

45,000 PMT

200,000 110,000 20,000 20,000 50,000 25,000 25,000 20,000 45,000

52,500 FV

In Excel:

= PV(0.08, 10, 45000, 52500) = −326,271 NPV = −252,500 + 326,271 = $73,771 Accessibility: Keyboard Navigation Difficulty: 3 Hard Gradable: manual

7-19 Copyright © 2018 McGraw-Hill Education. All rights reserved. No reproduction or distribution without the prior written consent of McGraw-Hill Education.


44. Your brother, age 40, is the regional manager at an office supply company. He thinks he might want to leave his job to go back to school for an MBA. He expects that his current job, if he were to stay at it, would pay him a real income stream of $75,000 per year until retirement at age 65. If he goes back to school, he would forego two years of income, but his real income after graduation would be $115,000 per year until retirement at age 65. He has been accepted to an MBA program that costs a real $22,000 per year. If his real opportunity cost is 8 percent, would leaving his job to get an MBA be a smart financial decision? To begin, calculate the present value of the real income stream from his current position. This results in a present value $800,608. Input:

25

8

?

75,000

0

n

i

PV

PMT

FV

−800,608

Output: In Excel: = PV(0.08,25,75000) = −800,608

For school, the present value of your brother’s real income stream is the present value of the 23-year real income from the new job, discounted back 2 years at the real discount rate, minus the present value of the cost of the MBA program. The value of the MBA real income stream, discounted back 2 years, is 1,192,672/1.08 2 = $1,022,524. Input:

23

8

n

i

?

115,000

0

PV

PMT

FV

−1,192,672

Output:

In Excel: = PV(0.08,23,115000) = −1,192,672 The present value of the cost of the MBA program is $39,232. Input: Output:

2

8

?

22,000

0

n

i

PV

PMT

FV

−39,232

In Excel: = PV(0.08,2,22000) = −39,232 The net present value of your brother’s MBA real income stream is the difference, or 1,022,524 − 39,232 = $983,292. This is greater than the value of his current income stream, so the correct financial decision is to return to school for an MBA. Accessibility: Keyboard Navigation Difficulty: 3 Hard Gradable: manual

7-20 Copyright © 2018 McGraw-Hill Education. All rights reserved. No reproduction or distribution without the prior written consent of McGraw-Hill Education.


Chapter 07 Test Bank Summary Category

Accessibility: Keyboard Navigation Difficulty: 1 Easy Difficulty: 2 Medium Difficulty: 3 Hard Gradable: automatic Gradable: manual

# of Questions

44 21 17 6 37 7

7-21 Copyright © 2018 McGraw-Hill Education. All rights reserved. No reproduction or distribution without the prior written consent of McGraw-Hill Education.


Chapter 08Test Bank 1. An average-risk project that has an NPV of zero when its cash flows are discounted at the weighted-average cost of capital will provide sufficient returns to satisfy both stockholders and bondholders. TRUE Accessibility: Keyboard Navigation Difficulty: 2 Medium Gradable: automatic

2. All else equal, if two competing firms in industry X are valuing the same plant in industry Y for a potential acquisition, the firm with the more volatile stock should arrive at a lower valuation for the plant. FALSE Accessibility: Keyboard Navigation Difficulty: 3 Hard Gradable: automatic

3. A beta greater than 1 is indicative of an above-average level of diversifiable (unsystematic) risk. FALSE Accessibility: Keyboard Navigation Difficulty: 1 Easy Gradable: automatic

4. Failing to include real options in a project valuation could cause the NPV of the project to be overestimated. FALSE Accessibility: Keyboard Navigation Difficulty: 1 Easy Gradable: automatic

5. The adjusted present value (APV) method of valuation is superior to the standard WACC method of valuation because the WACC method makes no adjustment for interest tax shields. FALSE Accessibility: Keyboard Navigation Difficulty: 2 Medium Gradable: automatic

6. In reality, the cost of equity is always less than the cost of debt because firms are not obligated to pay out cash to shareholders. FALSE Accessibility: Keyboard Navigation Difficulty: 2 Medium Gradable: automatic

7. Asset betas measure financial risk and business risk. FALSE Accessibility: Keyboard Navigation Difficulty: 2 Medium Gradable: automatic

8-1 Copyright © 2018 McGraw-Hill Education. All rights reserved. No reproduction or distribution without the prior written consent of McGraw-Hill Education.


8. When projected cash flows are in nominal dollars, they should be discounted with a nominal discount rate. TRUE Accessibility: Keyboard Navigation Difficulty: 1 Easy Gradable: automatic

9. A firm’s WACC is the appropriate discount rate to value a project undertaken by the firm only if the project has the same risk as the firm’s existing assets. TRUE Accessibility: Keyboard Navigation Difficulty: 1 Easy Gradable: automatic

10. If a firm’s cost of debt is lower than its cost of equity, shifting the firm’s financing toward more debt will always reduce the firm’s WACC. FALSE Accessibility: Keyboard Navigation Difficulty: 1 Easy Gradable: automatic

11. Total risk is measured by _____, and systematic risk is measured by ____. A. beta; alpha B. beta; standard deviation C. WACC; beta D. standard deviation; beta E. standard deviation; variance F. None of the options are correct. Accessibility: Keyboard Navigation Difficulty: 1 Easy Gradable: automatic

12. When investment returns are less than perfectly positively correlated, the resulting diversification effect means that A. making an investment in two or three large stocks will eliminate all of the unsystematic risk. B. making an investment in three companies all within the same industry will greatly reduce the systematic risk. C. spreading an investment across five diverse companies will not lower the total risk. D. spreading an investment across many diverse assets will eliminate all of the systematic risk. E. spreading an investment across many diverse assets will eliminate some of the total risk. F. None of the options are correct. Accessibility: Keyboard Navigation Difficulty: 1 Easy Gradable: automatic

13. Unsystematic risk A. can be effectively eliminated by portfolio diversification. B. is compensated for by the risk premium. C. is measured by beta. D. is measured by standard deviation. E. is related to the overall economy. F. None of the options are correct. Accessibility: Keyboard Navigation Difficulty: 1 Easy Gradable: automatic

8-2 Copyright © 2018 McGraw-Hill Education. All rights reserved. No reproduction or distribution without the prior written consent of McGraw-Hill Education.


14. Which of the following are examples of diversifiable risk? I. An earthquake damages Oakland, California. II. The federal government imposes an additional $1,000 fee on all business entities. III. Employment taxes increase nationally. IV. Toymakers are required to improve their safety standards. A. I and III only B. II and IV only C. II and III only D. I and IV only E. I, III, and IV only F. None of the options are correct. Accessibility: Keyboard Navigation Difficulty: 1 Easy Gradable: automatic

15. Which of the following statements are correct concerning diversifiable, or unsystematic, risks? I. Diversifiable risks can be largely eliminated by investing in 50 unrelated securities. II. There is no reward for accepting diversifiable risks. III. Diversifiable risks are generally associated with an individual firm or industry. IV. Beta measures diversifiable risk. A. I and III only B. II and IV only C. I and IV only D. I, II, and III only E. I, II, III, and IV F. None of the options are correct. Accessibility: Keyboard Navigation Difficulty: 1 Easy Gradable: automatic

16. Which of the following statements concerning risk are correct? I. Systematic risk is measured by beta. II. The risk premium increases as unsystematic risk increases. III. Systematic risk is the only part of total risk that should affect asset prices and returns. IV. Diversifiable risks are market risks you cannot avoid. A. I and III only B. II and IV only C. I and II only D. III and IV only E. I, II, and III only F. None of the options are correct. Accessibility: Keyboard Navigation Difficulty: 1 Easy Gradable: automatic

17. Which one of the following is an example of systematic risk? A. The Federal Reserve unexpectedly announces an increase in target interest rates. B. A flood washes away a firm’s warehouse. C. A city imposes an additional one-percent sales tax on all products. D. A toymaker has to recall its top-selling toy. E. Corn prices increase due to increased demand for alternative fuels. F. None of the options are correct. Accessibility: Keyboard Navigation Difficulty: 1 Easy Gradable: automatic

8-3 Copyright © 2018 McGraw-Hill Education. All rights reserved. No reproduction or distribution without the prior written consent of McGraw-Hill Education.


18. The excess return earned by a risky asset, for example, with a beta of 1.4, over that earned by a risk-free asset is referred to as a A. market risk premium. B. risk premium. C. systematic return. D. total return. E. real rate of return. F. None of the options are correct. Accessibility: Keyboard Navigation Difficulty: 1 Easy Gradable: automatic

19. The dividend growth model can be used to compute the cost of equity for a firm in which of the following situations? I. Firms that have a 100-percent retention ratio II. Firms that pay an unchanging dividend III. Firms that pay a constantly increasing dividend IV. Firms that pay an erratically growing dividend A. I and II only B. I and IV only C. II and III only D. I, II, and III only E. I, III, and IV only F. None of the options are correct. Accessibility: Keyboard Navigation Difficulty: 1 Easy Gradable: automatic

20. The cost of equity for a firm A. tends to remain static for firms with increasing levels of risk. B. increases as the unsystematic risk of the firm increases. C. can be estimated from the capital asset pricing model or the dividend growth model. D. equals the risk-free rate plus the market risk premium. E. equals the firm’s pre-tax weighted-average cost of capital. F. None of the options are correct. Accessibility: Keyboard Navigation Difficulty: 1 Easy Gradable: automatic

21. The pre-tax cost of debt A. is based on the current yield to maturity of the firm’s outstanding bonds. B. is equal to the coupon rate on the latest bonds issued by a firm. C. is equivalent to the average current yield on all of a firm’s outstanding bonds. D. is based on the original yield to maturity on the latest bonds issued by a firm. E. has to be estimated as it cannot be directly observed in the market. F. None of the options are correct. Accessibility: Keyboard Navigation Difficulty: 1 Easy Gradable: automatic

8-4 Copyright © 2018 McGraw-Hill Education. All rights reserved. No reproduction or distribution without the prior written consent of McGraw-Hill Education.


22. The after-tax cost of debt generally increases when I. a firm’s bond rating improves. II. the market-required rate of interest for the company’s bonds increases. III. tax rates decrease. IV. bond prices rise. A. I and III only B. II and III only C. I, II, and III only D. II, III, and IV only E. I, II, III, and IV F. None of the options are correct. Accessibility: Keyboard Navigation Difficulty: 1 Easy Gradable: automatic [The following information applies to the questions displayed below.]

FM Foods, Inc. Facts and assumptions as of Dec. 31, 2017 Yield to maturity on long-term government bonds Yield to maturity on company long-term bonds Coupon rate on company long-term bonds Historical excess return on common stocks Company equity beta Stock price Number of shares outstanding (millions) Book value of equity (millions) Book value of interest-bearing debt (millions)

4.4% 6.3% 7.0% 6.5% 1.20 $40.00 240 $5,240 $1,250

Tax rate

35.0%

23. Please refer to the information for FM Foods above. Estimate FM’s after-tax cost of equity capital. A. 4.50% B. 6.92% C. 7.93% D. 12.20% E. 17.48% F. None of the options are correct. E = gov’t borrowing rate + equity beta × market risk premium = 0.044 + 1.2 × 0.065 = 0.122

K

Accessibility: Keyboard Navigation Difficulty: 1 Easy Gradable: automatic

8-5 Copyright © 2018 McGraw-Hill Education. All rights reserved. No reproduction or distribution without the prior written consent of McGraw-Hill Education.


24. Please refer to the information for FM Foods above. Estimate FM’s after-tax cost of debt capital. A. 2.21% B. 4.10% C. 4.55% D. 6.30% E. 7.00% F. None of the options are correct. The correct approach is to use the YTM on the firm’s bonds for the before-tax cost. Thus, after-tax cost = 6.3% × (1 − 0.35) = 4.10%. Accessibility: Keyboard Navigation Difficulty: 1 Easy Gradable: automatic

25. Please refer to the information for FM Foods above. Estimate the appropriate weight of equity to be used when calculating FM’s weighted-average cost of capital. A. 11.5% B. 19.3% C. 80.7% D. 88.5% E. 100.0% F. None of the options are correct. Market value of equity = $40 × 240 million = $9,600 million. Weight of equity = 9,600/(9,600 + 1,250) = 0.8848 or 88.5%. Accessibility: Keyboard Navigation Difficulty: 1 Easy Gradable: automatic

26. Please refer to the information for FM Foods above. Estimate the appropriate weight of debt to be used when calculating FM’s weighted-average cost of capital. A. 11.5% B. 19.3% C. 80.7% D. 88.5% E. 100.0% F. None of the options are correct. Market value of equity = $40 × 240 million = $9,600 million. Weight of debt = 1,250/(9,600 + 1,250) = 0.1152 or 11.5%. Accessibility: Keyboard Navigation Difficulty: 1 Easy Gradable: automatic

8-6 Copyright © 2018 McGraw-Hill Education. All rights reserved. No reproduction or distribution without the prior written consent of McGraw-Hill Education.


27. Please refer to the information for FM Foods above. Estimate FM’s weighted-average cost of capital. A. 6.46% B. 6.58% C. 11.27% D. 11.32% E. 11.52% F. None of the options are correct. D KW =

(1 − t)KD

E

+ KE D+E

D+E

KW = (1 − 0.35) 0.063 (0.115) + 0.122 (0.885) KW = 0.1127 Accessibility: Keyboard Navigation Difficulty: 2 Medium Gradable: automatic

28. Please refer to the information for FM Foods above. FM is contemplating an average-risk investment costing $100 million and promising an annual after-tax cash flow of $15 million in perpetuity. Which of the following statements is/are correct? I. FM should reject the project because the IRR is greater than the firm’s WACC. II. FM should accept the project because the IRR is greater than the firm’s WACC. III. FM should accept the project because the NPV is greater than zero. IV. FM should reject the project because the NPV is less than zero. A. I only B. II only C. IV only D. I and IV only E. II and III only F. None of the options are correct. The IRR of the investment is 15/100 = 15%. FM’s WACC of 11.27% is shown in the calculations below. The NPV of the investment at the WACC = −100 + 15/0.1127 = $33.1 million. D

KW = (1 − t)KD

D+E

E

+ KE

D+E

KW = (1 − 0.35) 0.063 (0.115) + 0.122 (0.885) KW = 0.1127 Accessibility: Keyboard Navigation Difficulty: 3 Hard Gradable: automatic

8-7 Copyright © 2018 McGraw-Hill Education. All rights reserved. No reproduction or distribution without the prior written consent of McGraw-Hill Education.


29. Celebrity Auto Parts, Inc. Facts and assumptions as of Dec. 31, 2017 EBIT for 2017 (millions) Company equity beta Stock price Number of shares outstanding (millions) Book value of equity (millions) Book value of interest-bearing debt (millions) WACC

$ $ $ $

Tax rate

420 1.20 30 200 3,500 1,500 9% 25%

Please refer to the information for Celebrity Auto Parts above. What was Celebrity’s EVA in 2017? A. −$360 million B. −$135 million C. −$30 million D. $765 million E. None of the options are correct. EVA = EBIT(1 − t) − KWC = 420(1 − 25%) − 9%(3,500 + 1,500) = −$135 Accessibility: Keyboard Navigation Difficulty: 2 Medium Gradable: automatic

30. Which of the following statements are correct? I. Using the same risk-adjusted discount rate to discount all future cash flows adjusts for the fact that the more distant cash flows are often more risky than cash flows occurring sooner. II. If you can borrow all of the money you need for a project at 5%, the cost of capital for this project is 5%. III. The best way to obtain the cost of debt capital for a firm is to use the coupon rates on its bonds. IV. A firm’s weighted-average cost of capital is NOT the correct discount rate to use for all projects undertaken by the firm. A. I and III only B. II and IV only C. I and II only D. I and IV only E. I, II, and III only F. None of the options are correct. Accessibility: Keyboard Navigation Difficulty: 2 Medium Gradable: automatic

31. The capital structure weights used in computing the weighted-average cost of capital A. are based on the book values of total debt and total equity. B. are based on the market value of the firm’s debt and equity securities. C. are computed using the book value of the long-term debt and the book value of equity. D. remain constant over time unless the firm issues new securities. E. are restricted to the firm’s debt and common stock. F. None of the options are correct. Accessibility: Keyboard Navigation Difficulty: 1 Easy Gradable: automatic

8-8 Copyright © 2018 McGraw-Hill Education. All rights reserved. No reproduction or distribution without the prior written consent of McGraw-Hill Education.


32. The discount rate assigned to an individual project should be based on A. the firm’s weighted-average cost of capital. B. the actual sources of funding used for the project. C. an average of the firm’s overall cost of capital for the past five years. D. the current risk level of the overall firm. E. the risks associated with the use of the funds required by the project. F. None of the options are correct. Accessibility: Keyboard Navigation Difficulty: 1 Easy Gradable: automatic

33. The weighted-average cost of capital for a firm is the A. discount rate which the firm should apply to all of the projects it undertakes. B. rate of return a firm must earn on its existing assets to maintain the current value of its stock. C. coupon rate the firm should expect to pay on its next bond issue. D. minimum discount rate the firm should require on any new project. E. rate of return shareholders should expect to earn on their investment in this firm. F. None of the options are correct. Accessibility: Keyboard Navigation Difficulty: 1 Easy Gradable: automatic

34. Honest Abe’s is a chain of furniture retail stores. Integral Designs is a furniture maker and a supplier to Honest Abe’s. Honest Abe’s has a beta of 1.38 as compared to Integral Designs' beta of 1.12. Both firms carry no debt, i.e., are 100% equity financed. The risk-free rate of return is 3.5 percent and the market risk premium is 8 percent. What discount rate should Honest Abe's use if it considers a project that involves the manufacturing of furniture? A. 12.46% B. 12.92% C. 13.50% D. 14.08% E. 14.54% F. None of the options are correct. KE = gov’t borrowing rate + equity beta × market risk premium = 0.035 + 1.12(0.08) = 0.1246 or 12.46% Accessibility: Keyboard Navigation Difficulty: 1 Easy Gradable: automatic

35. A firm is considering an average-risk project with an IRR of 6%. The firm’s cost of debt ( KD) is 5%, its cost of equity (KE) is 12%, and its tax rate (t) is 20%. The target debt ratio (D/(D+E)) for the project, in market values, is 0.5. The firm should A. accept the project only if it can be completely financed with equity. B. accept the project only if it can be completely financed with debt. C. accept the project regardless of the financing method. D. reject the project regardless of the financing method. D

E

KW = (1 − t)KDD + E+ KED + E KW = (1 − 0.2)0.05(0.5) + 0.12(0.5) KW = 0.08 = 8%

The project should be rejected because the IRR of 6% does not meet the hurdle of 8%. Accessibility: Keyboard Navigation Difficulty: 2 Medium Gradable: automatic

8-9 Copyright © 2018 McGraw-Hill Education. All rights reserved. No reproduction or distribution without the prior written consent of McGraw-Hill Education.


36. Unitron Corp. is considering project Z, which costs $50 million and offers an annual after-tax cash flow of $7.5 million in perpetuity. The project is in an industry that has greater market risk than Unitron’s typical projects. Unitron’s company weightedaverage cost of capital, based on its typical projects, is 15%. Should Unitron Corp. accept project Z? A. Yes, because the NPV of the project is positive. B. Yes, because a zero-NPV project is marginally acceptable. C. No, because a zero-NPV project is a waste of resources. D. No, because the NPV of the project is negative. Use the equation for a perpetuity to solve for the IRR: 7.5/IRR = 50 IRR = 15% Since the IRR is 15%, it would have an NPV of zero at a WACC of 15%. However, this project is riskier than the firm’s average projects, so the WACC would be higher than 15%, which would make the NPV negative. Accessibility: Keyboard Navigation Difficulty: 2 Medium Gradable: automatic

37. Company X has 2 million shares of common stock outstanding at a book value of $2 per share. The stock trades for $3.00 per share. It also has $2 million in face value of debt that trades at 90% of par. What is the appropriate debt ratio ( D/(D+E)) to use for calculating Company X’s weighted-average cost of capital? A. 23.1% B. 25.0% C. 31.0% D. 33.3% D = 0.9 × $2 million = $1.8 million E = $3 ×2 million = $6 million D/(D+E) = 1.8/(1.8 + 6) = 0.231 Accessibility: Keyboard Navigation Difficulty: 1 Easy Gradable: automatic

38. JKL Corporation, a company devoted primarily to paper products, is estimating the cost of equity appropriate for a vegetable processing plant it is planning to build. JKL Corp. has an equity beta of 1.0 and a debt ratio ( D/(D+E)) of 0.3. A comparable (vegetable processing) firm has an equity beta of 0.8 and a debt ratio of 0.2. Assume a risk-free rate of 5% and a market risk premium of 8%. What cost of equity should JKL use in this situation? A. 7.7% B. 11.4% C. 12.3% D. 13.0% Unlever the comparable firm’s beta: βA = (E/V)βE = (0.8)0.8 = 0.64 Then relever at JKL’s debt ratio: βE = βA/(E/V) = 0.64/(0.7) = 0.91 Cost of Equity: KE = 5% + 0.91(8%) = 12.3% Accessibility: Keyboard Navigation Difficulty: 2 Medium Gradable: automatic

8-10 Copyright © 2018 McGraw-Hill Education. All rights reserved. No reproduction or distribution without the prior written consent of McGraw-Hill Education.


39. Florida Corp. is calculating the appropriate rate for discounting cash flows on a project valued using the APV method. Florida’s target debt ratio (D/(D+E)) in market value terms is 50%, and the yield-to-maturity on its outstanding debt is 6%. A comparable firm has an equity beta of 1.4 and a debt ratio ( D/(D+E)) of 40%. Assume a risk-free rate of 5% and a market risk premium of 8%. Florida’s tax rate is 40%. What discount rate should Florida use? A. 7.66% B. 11.02% C. 11.72% D. 18.44% βA = (E/V)βE = (0.6)1.4 = 0.84 KE = 5% + 0.84(8%) = 11.72% Accessibility: Keyboard Navigation Difficulty: 2 Medium Gradable: automatic

40. Key facts and assumptions concerning Costco Company, at December 31, 2011, appear below. Facts and Assumptions Yield to maturity on long-term government bonds Yield to maturity on company long-term bonds Coupon rate on company long-term bonds Historical excess return on common stocks Company equity beta Stock price Number of shares outstanding (millions) Book value of equity (millions) Book value of interest-bearing debt (millions) Tax rate

$ $11,585 $

3.28% 4.62% 5.50% 6.10% 0.80 75.08 449.5 2,524 35.00%

Use the above information to answer the following questions. a. Estimate Costco’s cost of equity capital. b. Estimate Costco’s weighted-average cost of capital. a. KE = gov’t borrowing rate + equity beta × market risk premium KE = 3.28 + 0.80 × 6.10 = 8.16% b. Market value of equity = 449.5 × 75.08 = $33,748 Weight for debt = 2,524/(2,524 + 33,748) = 0.070 D KW = (1 − t)KD

D+E

E

+ KE

D+E

KW = (1 − 0.35)0.0462(0.07) + 0.0816(0.93) KW = 0.078 = 7.8% Accessibility: Keyboard Navigation Difficulty: 2 Medium Gradable: manual

8-11 Copyright © 2018 McGraw-Hill Education. All rights reserved. No reproduction or distribution without the prior written consent of McGraw-Hill Education.


41. Explain the difference between systematic and unsystematic risk and why one of these types of risks is rewarded with a risk premium while the other type is not. Unsystematic, or diversifiable, risk affects a limited number of securities and can be eliminated by investing in securities from various industries and geographic regions. Unsystematic risk is not rewarded because it can be eliminated by investors. Systematic risk is risk that affects most, or all, securities and cannot be diversified away. Since systematic risk cannot be eliminated by investors it is rewarded with a risk premium. Systematic risk is measured by beta. Accessibility: Keyboard Navigation Difficulty: 2 Medium Gradable: manual

42. Suppose that your company’s weighted-average cost of capital is 9 percent. Your company is planning to undertake a project with an internal rate of return of 12%, but you believe that this project is not a good investment for the firm. What logical arguments might you use to convince your boss to forego the project despite its high rate of return? Is it possible that making investments with expected returns higher than your company’s cost of capital will destroy value? If so, how? If the investment is above the company’s average risk, the company’s cost of capital is not an appropriate benchmark. Equivalently, you might argue that the high risk of the investment places it below the security market line. Such investments destroy value because they promise returns that, while greater than the company’s WACC, are still below those available on similar-risk investments available. Other possible arguments include that the investment is not consistent with the strategic plan or that the cash flow estimates are too optimistic. Accessibility: Keyboard Navigation Difficulty: 2 Medium Gradable: manual

43. The standard deviation of returns on Wildcat Oil Drilling is very high. Does this necessarily imply that Wildcat Oil Drilling is a high-risk investment when investors hold diversified portfolios? Explain why or why not. The equation for beta in Chapter 8 shows that the nondiversifiable risk of an asset is the product of its standard deviation of returns and the correlation of those returns with those on a well-diversified portfolio. Wildcat Oil Drilling may have a high standard deviation of returns, but if those returns are poorly correlated with those on a well-diversified portfolio, as is likely the case, nondiversifiable risk may be low. In other words, if investors can diversify away most of Wildcat’s risk, then it is not truly a highrisk investment. Accessibility: Keyboard Navigation Difficulty: 2 Medium Gradable: manual

44. What is the present value of a cash flow stream of $10,000 per year annually for 11 years that then grows at 2 percent per year forever? Assume the appropriate discount rate is 12 percent. Divide the cash flows into two periods: An 11-year annuity of $10,000, and a growing perpetuity beginning in the 11 th year. The value of the 11-year annuity is $59,377, as shown below: Input: Output:

11

12

?

10,000

0

n

i

PV

PMT

FV

−59,377

In Excel: = PV(0.12,11,10000) = −59,377 The value of the growing perpetuity (using the formula provided in Chapter 8 for a growing dividend into perpetuity) at time 11 is $10,000(1 + 0.02)/(0.12 − 0.02) = $102,000. Its value at time zero is $102,000/(1.12) 11 = $29,323. Adding these values, 29,323 + 59,377 = $88,700. Accessibility: Keyboard Navigation Difficulty: 2 Medium Gradable: manual

8-12 Copyright © 2018 McGraw-Hill Education. All rights reserved. No reproduction or distribution without the prior written consent of McGraw-Hill Education.


45. Kilborn Corporation’s balance sheet is shown below. The current rate on treasury bonds is 7%. The yield to maturity on Kilborn’s bonds is 10%. Assume a market risk premium of 8%. Kilborn’s tax rate is 40%. Kilborn’s stock price is $45, and it has 2 million shares outstanding. Kilborn’s beta is unknown because it only recently went public, but it is known that a comparable firm’s stock has a beta of 1.2 and that the comparable firm’s debt ratio ( D/(D+E)) is 0.4. What is the weighted-average cost of capital for Kilborn Corporation?

Assets Cash and securities Accounts receivable Inventory

Kilborn Corporation Balance Sheet ($ millions) Liabilities and Equity 10 Bonds 50 Common stock 50 Retained earnings

Net fixed assets Total assets

20 50 80

40 150

Total liab. and equity

150

Note that for Kilborn: D = 20; E = 45 ×2 = 90; V = D + E = 110 Unlever the comparable firm’s beta: βA = (E/V)βE = (0.6)1.2 = 0.72 Then relever at Kilborn’s debt ratio: βE = βA/(E/V) = 0.72/(90/110) = 0.88 Cost of Equity: KE = 7% + 0.88(8%) = 14.04% Cost of Debt: KD = 10% D KW =

(1 − t)KD

E

+ KE D+E

D+E

KW = (1 − 0.4) 0.1 (20/110) + 0.1404 (90/110) KW = 0.126 = 12.6% Accessibility: Keyboard Navigation Difficulty: 2 Medium Gradable: manual

8-13 Copyright © 2018 McGraw-Hill Education. All rights reserved. No reproduction or distribution without the prior written consent of McGraw-Hill Education.


Chapter 08 Test Bank Summary Category Accessibility: Keyboard Navigation Difficulty: 1 Easy Difficulty: 2 Medium Difficulty: 3 Hard Gradable: automatic Gradable: manual

# of Questions 45 26 17 2 39 6

8-14 Copyright © 2018 McGraw-Hill Education. All rights reserved. No reproduction or distribution without the prior written consent of McGraw-Hill Education.


Chapter 09 TestBank 1. Acquisitions create shareholder value on average. TRUE Accessibility: Keyboard Navigation Difficulty: 1 Easy Gradable: automatic

2. When an acquirer purchases all of a target firm’s equity, it must assume the target’s liabilities. TRUE Accessibility: Keyboard Navigation Difficulty: 1 Easy Gradable: automatic

3. An acquirer should be willing to pay a higher control premium for a poorly managed company than for a wellmanaged company. TRUE Accessibility: Keyboard Navigation Difficulty: 1 Easy Gradable: automatic

4. When an acquirer values a potential target, it should discount the target’s cash flows at the target’s cost of capital. TRUE Accessibility: Keyboard Navigation Difficulty: 1 Easy Gradable: automatic

5. Rational investors would never value a company’s stock below its liquidation value. FALSE Accessibility: Keyboard Navigation Difficulty: 1 Easy Gradable: automatic

6. In business valuation, a typical discount for lack of marketability is about 10 percent. FALSE Accessibility: Keyboard Navigation Difficulty: 2 Medium Gradable: automatic

7. Terminal value estimates based on book value tend to understate a company’s terminal value. TRUE Accessibility: Keyboard Navigation Difficulty: 1 Easy Gradable: automatic

9-1 Copyright © 2018 McGraw-Hill Education. All rights reserved. No reproduction or distribution without the prior written consent of McGraw-Hill Education.


8. All else equal, a terminal value based on a no-growth perpetuity would be higher than a terminal value based on a perpetuity with 2-percent growth. FALSE Accessibility: Keyboard Navigation Difficulty: 1 Easy Gradable: automatic

9. An acquirer should never consider a target that would reduce the acquirer’s earnings per share. FALSE Accessibility: Keyboard Navigation Difficulty: 1 Easy Gradable: automatic

10. In venture-capital valuation, the post-money valuation is equal to the pre-money valuation plus the amount of the venture capitalist’s investment. TRUE Accessibility: Keyboard Navigation Difficulty: 2 Medium Gradable: automatic

11. Which of the following statements are correct? I. Liquidation value of a firm is equal to the present worth of expected future cash flows from operating activities. II. When an acquiring firm purchases a target firm’s equity, the acquirer must assume the target’s liabilities. III. The market value of a public company reflects the worth of the business to minority investors. IV. The fair market value of a business is usually the lower of its liquidation value and its going-concern value. A. I and III only B. II and IV only C. II and III only D. I, II, and III only E. II, III, and IV only F. None of the options are correct. Accessibility: Keyboard Navigation Difficulty: 2 Medium Gradable: automatic

9-2 Copyright © 2018 McGraw-Hill Education. All rights reserved. No reproduction or distribution without the prior written consent of McGraw-Hill Education.


12. Ginormous Oil entered into an agreement to purchase all of the outstanding shares of Slick Company for $60 per share. The number of outstanding shares at the time of the announcement was 82 million. The book value of liabilities on the balance sheet of Slick Co. was $1.46 billion. What was the cost of this acquisition to the shareholders of Ginormous Oil? A. $1.46 billion B. $3.46 billion C. $4.92 billion D. $6.38 billion E. $8.38 billion F. None of the options are correct. The value of the bid to Ginormous’s shareholders is the value of the assets acquired in the merger. This would include the value of the equity acquired and the liabilities that accompany the equity. Therefore, the cost of the acquisition was ($60 × 82 million shares) + $1.46 billion = 6.38 billion. Accessibility: Keyboard Navigation Difficulty: 1 Easy Gradable: automatic

13. Ginormous Oil entered into an agreement to purchase all of the outstanding shares of Slick Company for $60 per share. The number of outstanding shares at the time of the announcement was 82 million. The book value of liabilities on the balance sheet of Slick Co. was $1.46 billion. Immediately prior to the Ginormous Oil bid, the shares of Slick Co. traded at $33 per share. What value did Ginormous Oil place on the control of Slick Co.? A. $2.21 billion B. $2.71 billion C. $4.17 billion D. $6.38 billion E. None of the options are correct. Ginormous paid $60 per share for a firm that minority shareholders valued at $33 per share, so they placed a value of 60 – 33 = $27 per share on control of Slick. $27 × 82 million = $2.21 billion. Accessibility: Keyboard Navigation Difficulty: 1 Easy Gradable: automatic

14. Which of the following statements is/are correct? I. Going-concern value of a firm is equal to the present value of expected net income. II. When a buyer values a target firm, the appropriate discount rate is the buyer’s weighted-average cost of capital. III. The liquidation value estimate of terminal value usually vastly understates a healthy company’s terminal value. IV. The value of a firm’s equity equals the discounted cash flow value of the firm minus all liabilities. A. II only B. III only C. I and II only D. II and III only E. II, III, and IV only F. None of the options are correct. Accessibility: Keyboard Navigation Difficulty: 2 Medium Gradable: automatic

9-3 Copyright © 2018 McGraw-Hill Education. All rights reserved. No reproduction or distribution without the prior written consent of McGraw-Hill Education.


15. Which of the following statements are correct? I. Going-concern value of a firm is equal to the present value of expected future cash flows to owners and creditors. II. When an acquiring firm purchases a target firm’s equity, the acquirer need not assume the target’s liabilities. III. The market value of a public company reflects the worth of the business to minority investors. IV. The fair market value of a business is usually the lower of its liquidation value and its going-concern value. A. I and III only B. II and IV only C. II and III only D. I, II, and III only E. II, III, and IV only F. None of the options are correct. Accessibility: Keyboard Navigation Difficulty: 2 Medium Gradable: automatic

16. The following table presents forecasted financial and other information for Havasham Industries: 2015 2016 2017 Projected EBIT $ 317 $ 339 $ 363 Earnings after tax 197 210 225 Free cash flow 135 144 155 Havasham's WACC Expected growth rate in FCFs after 2017 Warranted MV firm/FCF in 2017 Warranted P/E in 2017

8.2% 4.0% 19.4 18.7

What is an appropriate estimate of Havasham’s terminal value as of the end of 2017 using the perpetual-growth equation as your estimate? A. $161 million B. $363 million C. $3,690 million D. $3,888 million E. $5,357 million F. None of the options are correct. FCF2018 = 155 × (1 + 0.04) = $161 million. Terminal value2017 = FCF2018/(KW − g ) = $161 million/(0.082 − 0.04) = $3,833 million. Accessibility: Keyboard Navigation Difficulty: 2 Medium Gradable: automatic

9-4 Copyright © 2018 McGraw-Hill Education. All rights reserved. No reproduction or distribution without the prior written consent of McGraw-Hill Education.


17. The following table presents forecasted financial and other information for Havasham Industries:

Projected EBIT Earnings after tax Free cash flow

2015 $ 317 197 135

Havasham's WACC Expected growth rate in FCFs after 2017 Warranted MV firm/FCF in 2017 Warranted P/E in 2017

8.2% 4.0% 19.4 18.7

2016 $ 339 210 144

2017 $ 363 225 155

What is an appropriate estimate of Havasham’s terminal value as of the end of 2017 using a warranted price-to-earnings multiple as your estimate? A. $225 million B. $3,833.0 million C. $4,207.5 million D. $4,365.0 million E. $6,788.1 million F. None of the options are correct. Terminal value2017 = 18.7 × $225 million = $4,207.5 million. Accessibility: Keyboard Navigation Difficulty: 2 Medium Gradable: automatic

18. The following table presents forecasted financial and other information for Havasham Industries: 2015 2016 2017 Projected EBIT $ 317 $ 339 $ 363 Earnings after tax 197 210 225 Free cash flow 135 144 155 Havasham's WACC Expected growth rate in FCFs after 2017 Warranted MV firm/FCF in 2017 Warranted P/E in 2017

8.2% 4.0% 19.4 18.7

What is an appropriate estimate of Havasham’s terminal value as of the end of 2017 using a warranted multiple of free cash flow as your estimate? A. $155 million B. $2,898.5 million C. $3,007.0 million D. $4,365.0 million E. $7,042.2 million F. None of the options are correct. Terminal value2017 = 19.4 × $155 million = $3,007.0 million. Accessibility: Keyboard Navigation Difficulty: 1 Easy Gradable: automatic

9-5 Copyright © 2018 McGraw-Hill Education. All rights reserved. No reproduction or distribution without the prior written consent of McGraw-Hill Education.


19. Atmosphere, Inc. has offered $860 million cash for all of the common stock in ACE Corporation. Based on recent market information, ACE is worth $710 million as an independent operation. For the merger to make economic sense for Atmosphere, what would the minimum estimated present value of the enhancements from the merger have to be? A. $0 B. $75 million C. $150 million D. $710 million E. $860 million F. None of the options are correct. Minimum economic value in PV terms = $860 million − $710 million = $150 million Accessibility: Keyboard Navigation Difficulty: 1 Easy Gradable: automatic

20. Consider the following premerger information about a bidding firm (Buyitall Inc.) and a target firm (Tarjay Corp.). Assume that neither firm has any debt outstanding. Buyitall Tarjay Shares outstanding 1,500 1,100 Price per share $32 $26 Buyitall has estimated that the present value of any enhancements that Buyitall expects from acquiring Tarjay is $2,600. What is the NPV of the merger assuming that Tarjay is willing to be acquired for $28 per share in cash? A. $400 B. $600 C. $1,800 D. $2,200 E. $2,600 F. None of the options are correct. The NPV of the merger is the market value of the target firm, plus the value of the enhancements, minus the acquisition costs: NPV = 1,100 ($26) + $2,600 − 1,100($28) = $400. Accessibility: Keyboard Navigation Difficulty: 1 Easy Gradable: automatic

9-6 Copyright © 2018 McGraw-Hill Education. All rights reserved. No reproduction or distribution without the prior written consent of McGraw-Hill Education.


[The following information applies to the questions displayed below.]

In March of 2011, Macklemore Corp. considered an acquisition of Blue Scholar Learning, Inc. (BSL), a privately held educational software firm. As a first step in deciding what price to bid for BSL, Macklemore’s CFO, Ryan Lewis, has prepared a five-year financial projection for the company assuming the acquisition takes place. Blue Scholar Learning, Inc. (BSL) 5-year Financial Projection ($ millions) Actual 2010 2011 Income Statement Net sales Cost of sales Gross income Depreciation Interest expense Operating expenses Net income before tax Provision for taxes Net income after tax Balance sheet Total current assets Gross property and equipment Accumulated depreciation Net property and equipment Goodwill Total assests Accounts payable Short-term debt Current portion long-term debt Accrued expenses Total current liabilities Long-term debt Deferred taxes Shareholders' equity Total liabilities and equity Free cash flows

2012

2013

2014

2015

$ 1,996 644 1,352 492 171 212 477 186 $ 291

$ 2,267 $ 2,508 742 830 1,525 1,678 785 1,061 178 191 239 270 323 156 126 61 $ 197 $ 95

$ 2,827 $ 3,138 $ 959 1,087 1,868 2,051 1,301 1,009 175 142 306 334 86 566 34 221 $ 52 $ 345 $

3,571 1,241 2,330 917 110 374 929 363 566

1,121 4,180 868

1,234 5,149 1,654 3,495 1,069 5,798 77 482 140 97 796 1,554 334 3,104 $ 5,798 $

1,650 7,449 4,015 3,434 1,069 6,153 110 814 200 134 1,258 1,189 454 3,252 $ 6,153 $

2,179 9,016 5,941 3,075 1,069 6,323 135 393 267 168 963 699 496 4,165 6,323

3,312 1,069 5,502 104 335 41 86 566 1,694 335 2,907 $ 5,502

1,412 6,410 2,714 3,696 1,069 6,177 91 842 165 120 1,218 1,389 370 3,200 6,177

$ (130)

$ 215

1,923 8,200 5,024 3,176 1,069 6,168 117 585 223 174 1,099 966 505 3,598 6,168 $

$ 464

$ 490

9-7 Copyright © 2018 McGraw-Hill Education. All rights reserved. No reproduction or distribution without the prior written consent of McGraw-Hill Education.


21. Use BSL’s actual financial data for 2010 and its projections for 2011 as shown above. What is BSL’s projected free cash flow (in $ millions) for 2011? A. −$938 B. −$792 C. −$7 D. $122 E. $1,091 F. None of the options are correct. (NOTE to instructor: for this question, you might choose to remove the last row of the table above.) FCF = EBIT(1 − Tax rate) + Depreciation − Capital expenditures − Working capital investment. EBIT = Income before tax + Interest = 323 + 178 = $501. Tax rate = 126/323 = 0.390 or 39.0% Capital expenditures = change in gross property and equipment = 5,149 − 4,180 = 969 Change in working capital: working capital is current assets less operating current liabilities (this excludes short-term debt and the current portion of long-term debt.) Hence, the working capital balance is 1,234 − (77 + 97) = 1,060 in 2011, and 931 in 2010. The change in working capital is 1,060 − 931 = 129; this increase is a cash outflow. Thus, 2011 Free Cash Flow = 501(1 − 0.39) + 785 − 969 − 129 = −$7. Accessibility: Keyboard Navigation Difficulty: 2 Medium Gradable: automatic

22. Use BSL’s actual financial data for 2010 and its projections for 2011 to 2015 as shown above. Estimate the present value of BSL’s free cash flow (in $ millions) for the years 2011 to 2015. The WACC of the acquiring firm (Macklemore) is 8.0 percent, BSL’s WACC is 11.5 percent, and the average of the two companies’ WACCs, weighted by sales, is 8.2 percent. (NOTE to instructor: for this question, you might choose to include the FCF for 2011 of −$7 in the last row of the table above.) A. −$1.29 B. $628.79 C. $720.58 D. $726.68 E. $743.94 F. None of the options are correct. FCF = EBIT(1 − Tax rate) + Depreciation − Capital expenditures − Working capital investment. EBIT = Income before tax + Interest = 323 + 178 = $501. Tax rate = 126/323 = 0.390 or 39.0% Capital expenditures = change in gross property and equipment = 5,149 − 4,180 = 969 Change in working capital: working capital is current assets less operating current liabilities (this excludes short-term debt and the current portion of long-term debt.) Hence, the working capital balance is 1,234 − (77 + 97) = 1,060 in 2011, and 931 in 2010. The change in working capital is 1,060 − 931 = 129; this increase is a cash outflow. Thus, 2011 Free Cash Flow = 501(1 − 0.39) + 785 − 969 − 129 = −$7. PV@ 11.5% {FCFs, 2011-2015} = $628.79 The fundamental principle is that the discount rate should reflect the risks of the cash flows discounted. Here, the cash flows are BSL’s, so BSL’s WACC is the appropriate discount rate. Some argue incorrectly that because BSL will disappear in the merger, the cash flows will become Macklemore’s, so Macklemore’s WACC is the appropriate discount rate. However, the relevant criterion is the risk of the cash flows, not who owns them or what we call them. Accessibility: Keyboard Navigation Difficulty: 3 Hard Gradable: automatic

9-8 Copyright © 2018 McGraw-Hill Education. All rights reserved. No reproduction or distribution without the prior written consent of McGraw-Hill Education.


23. Use BSL’s actual financial data for 2010 and its projections for 2011 to 2015 as shown above. Estimate BSL’s value (in $ millions) at the end of 2010 assuming it is worth the book value of its assets at the end of 2015. The WACC of the acquiring firm (Macklemore) is 8.0 percent, BSL’s WACC is 11.5 percent, and the average of the two companies’ WACCs, weighted by sales, is 8.2 percent. A. $628.24 B. $3,669.01 C. $4,297.80 D. $4,412.94 E. $4,984.28 F. $6,951.24 G. None of the options are correct. (NOTE to instructor: for this question, you might choose to include the FCF for 2011 of −$7 in the last row of the table above.) FCF = EBIT(1 − Tax rate) + Depreciation − Capital expenditures − Working capital investment. EBIT = Income before tax + Interest = 323 + 178 = $501. Tax rate = 126/323 = 0.390 or 39.0% Capital expenditures = change in gross property and equipment = 5,149 − 4,180 = 969 Change in working capital: working capital is current assets less operating current liabilities (this excludes short-term debt and the current portion of long-term debt.) Hence, the working capital balance is 1,234 − (77 + 97) = 1,060 in 2011, and 931 in 2010. The change in working capital is 1,060 − 931 = 129; this increase is a cash outflow. Thus, 2011 Free Cash Flow = 501(1 − 0.39) + 785 − 969 − 129 = −$7. PV@ 11.5% {FCFs, 2011-2015} = $628.79 Terminal value in 2015 = $6,323. PV = 6,323/1.1155 = 3,669.01. Estimated firm value = $628.79 + 3,669.01 = $4,297.80 Accessibility: Keyboard Navigation Difficulty: 3 Hard Gradable: automatic

24. Use BSL’s actual financial data for 2010 and its projections for 2011 to 2015 as shown above. Assume BSL is worth the book value of its assets at the end of 2015. The WACC of the acquiring firm (Macklemore) is 8.0 percent, BSL’s WACC is 11.5 percent, and the average of the two companies’ WACCs, weighted by sales, is 8.2 percent. What is the maximum acquisition price (in $ millions) Macklemore should pay to acquire BSL’s equity? A. $1,702.80 B. $2,227.80 C. $2,342.94 D. $2,383.94 E. $2,603.80 F. $4,297.80 G. None of the options are correct. (NOTE to instructor: for this question, you might choose to include the FCF for 2011 of −$7 in the last row of the table above.) FCF = EBIT(1 − Tax rate) + Depreciation − Capital expenditures − Working capital investment. EBIT = Income before tax + Interest = 323 + 178 = $501. Tax rate = 126/323 = 0.390 or 39.0% Capital expenditures = change in gross property and equipment = 5,149 − 4,180 = 969

9-9 Copyright © 2018 McGraw-Hill Education. All rights reserved. No reproduction or distribution without the prior written consent of McGraw-Hill Education.


Change in working capital: working capital is current assets less operating current liabilities (this excludes short-term debt and the current portion of long-term debt.) Hence, the working capital balance is 1,234 − (77 + 97) = 1,060 in 2011, and 931 in 2010. The change in working capital is 1,060 − 931 = 129; this increase is a cash outflow. Thus, 2011 Free Cash Flow = 501(1 − 0.39) + 785 − 969 − 129 = −$7. PV@ 11.5% {FCFs, 2011-2015} = $628.79 Terminal value in 2015 = $6,323 PV of terminal value = 6,323/1.1155 = 3,669.01 Estimated firm value = $628.79 + 3,669.01 = $4,297.80 Equity value = firm value − existing interest-bearing debt = $4,297.80 − (335 + 41 + 1,694) = $2,227.80 Accessibility: Keyboard Navigation Difficulty: 3 Hard Gradable: automatic

25. Use BSL’s actual financial data for 2010 and its projections for 2011 to 2015 as shown above. Estimate BSL’s value (in $ millions) at the end of 2010 assuming that in the years after 2015, the company’s free cash flow grows 4 percent per year in perpetuity. The WACC of the acquiring firm (Macklemore) is 8.0 percent, BSL’s WACC is 11.5 percent, and the average of the two companies’ WACCs, weighted by sales, is 8.2 percent. A. $4,297.25 B. $4,571.49 C. $4,686.78 D. $6,181.49 E. $5,351.19 F. $7,423.16 G. None of the options are correct. (NOTE to instructor: for this question, you might choose to include the FCF for 2011 of −$7 in the last row of the table above.) FCF = EBIT(1 − Tax rate) + Depreciation − Capital expenditures − Working capital investment. EBIT = Income before tax + Interest = 323 + 178 = $501. Tax rate = 126/323 = 0.390 or 39.0% Capital expenditures = change in gross property and equipment = 5,149 − 4,180 = 969 Change in working capital: working capital is current assets less operating current liabilities (this excludes short-term debt and the current portion of long-term debt.) Hence, the working capital balance is 1,234 − (77 + 97) = 1,060 in 2011, and 931 in 2010. The change in working capital is 1,060 − 931 = 129; this increase is a cash outflow. Thus, 2011 Free Cash Flow = 501(1 − 0.39) + 785 − 969 − 129 = −$7. PV@ 11.5% {FCFs, 2011-2015} = $628.79 Terminal value in 2015 = (490 × (1 + 0.04))/(0.115 − 0.04) = $6,794.67 PV of terminal value = 6,794.67/1.1155 = 3,942.70. Estimated firm value = $628.79 + 3,942.70 = $4,571.49 Accessibility: Keyboard Navigation Difficulty: 3 Hard Gradable: automatic

9-10 Copyright © 2018 McGraw-Hill Education. All rights reserved. No reproduction or distribution without the prior written consent of McGraw-Hill Education.


26. Use BSL’s actual financial data for 2010 and its projections for 2011 to 2015 as shown above. Assume that in the years after 2015, the company’s free cash flow grows 4 percent per year in perpetuity. The WACC of the acquiring firm (Macklemore) is 8.0 percent, BSL’s WACC is 11.5 percent, and the average of the two companies’ WACCs, weighted by sales, is 8.2 percent. What is the maximum acquisition price (in $ millions) Macklemore should pay to acquire BSL’s equity at the end of 2010? A. $1,976.49 B. $2,501.49 C. $2,877.49 D. $4,195.49 E. $4,571.49 F. None of the options are correct. (NOTE to instructor: for this question, you might choose to include the FCF for 2011 of −$7 in the last row of the table above.) FCF = EBIT(1 − Tax rate) + Depreciation − Capital expenditures − Working capital investment. EBIT = Income before tax + Interest = 323 + 178 = $501. Tax rate = 126/323 = 0.390 or 39.0% Capital expenditures = change in gross property and equipment = 5,149 − 4,180 = 969 Change in working capital: working capital is current assets less operating current liabilities (this excludes short-term debt and the current portion of long-term debt.) Hence, the working capital balance is 1,234 − (77 + 97) = 1,060 in 2011, and 931 in 2010. The change in working capital is 1,060 − 931 = 129; this increase is a cash outflow. Thus, 2011 Free Cash Flow = 501(1 − 0.39) + 785 − 969 − 129 = −$7. PV@ 11.5% {FCFs, 2011-2015} = $628.79 Terminal value in 2015 = (490 × (1 + 0.04))/(0.115 − 0.04) = $6,794.67 PV of terminal value = 6,794.67/1.1155 = 3,942.70 Estimated firm value = $628.79 + 3,942.70 = $4,571.49 Equity value = firm value − existing interest-bearing debt = $4,571.49 − (335 + 41 + 1,694) = $2,501.49 Accessibility: Keyboard Navigation Difficulty: 3 Hard Gradable: automatic

27. Use BSL’s actual financial data for 2010 and its projections for 2011 to 2015 as shown above. Estimate BSL’s value (in $ millions) at the end of 2010 assuming that at year-end 2015, the company’s equity is worth 15 times earnings after tax, and its debt is worth book value. The WACC of the acquiring firm (Macklemore) is 8.0 percent, BSL’s WACC is 11.5 percent, and the average of the two companies’ WACCs, weighted by sales, is 8.2 percent. A. $628.24 B. $3,669.01 C. $7,429.74 D. $6,343.81 E. $6,755.83 F. $7,008.06 G. None of the options are correct. (NOTE to instructor: for this question, you might choose to include the FCF for 2011 of −$7 in the last row of the table above.) FCF = EBIT(1 − Tax rate) + Depreciation − Capital expenditures − Working capital investment. EBIT = Income before tax + Interest = 323 + 178 = $501. Tax rate = 126/323 = 0.390 or 39.0% Capital expenditures = change in gross property and equipment = 5,149 − 4,180 = 969

9-11 Copyright © 2018 McGraw-Hill Education. All rights reserved. No reproduction or distribution without the prior written consent of McGraw-Hill Education.


Change in working capital: working capital is current assets less operating current liabilities (this excludes short-term debt and the current portion of long-term debt.) Hence, the working capital balance is 1,234 − (77 + 97) = 1,060 in 2011, and 931 in 2010. The change in working capital is 1,060 − 931 = 129; this increase is a cash outflow. Thus, 2011 Free Cash Flow = 501(1 − 0.39) + 785 − 969 − 129 = −$7. PV@ 11.5% {FCFs, 2011-2015} = $628.79 Terminal value in 2015 = value of equity + value of debt = 15 × 566 + (393 + 267 + 699) = 8,490 + 1,359 = $9,849 PV of terminal value = 9,849/1.1155 = $5,715.02 Estimated firm value = $628.79 + $5,715.02 = $6,343.81 Accessibility: Keyboard Navigation Difficulty: 3 Hard Gradable: automatic

28. Use BSL’s actual financial data for 2010 and its projections for 2011 to 2015 as shown above. Assume that at yearend 2015, the company’s equity is worth 15 times earnings after tax, and its debt is worth book value. The WACC of the acquiring firm (Macklemore) is 8.0 percent, BSL’s WACC is 11.5 percent, and the average of the two companies’ WACCs, weighted by sales, is 8.2 percent. What is the maximum acquisition price (in $ millions) Macklemore should pay to acquire BSL’s equity at the end of 2010? (NOTE to instructor: for this question, you might choose to include the FCF for 2011 of −$7 in the last row of the table above.)

A. $3,484.68 B. $4,723.81 C. $4,938.06 D. $5,554.68 E. $6,343.81 F. None of the options are correct. FCF = EBIT(1 −Tax rate) + Depreciation − Capital expenditures − Working capital investment. EBIT = Income before tax + Interest = 323 + 178 = $501. Tax rate = 126/323 = 0.390 or 39.0% Capital expenditures = change in gross property and equipment = 5,149 − 4,180 = 969 Change in working capital: working capital is current assets less operating current liabilities (this excludes short-term debt and the current portion of long-term debt.) Hence, the working capital balance is 1,234 − (77 + 97) = 1,060 in 2011, and 931 in 2010. The change in working capital is 1,060 − 931 = 129; this increase is a cash outflow. Thus, 2011 Free Cash Flow = 501(1 − 0.39) + 785 − 969 − 129 = −$7. PV@ 11.5% {FCFs, 2011-2015} = $628.79 Terminal value in 2015 = value of equity + value of debt = 15 × 566 + (393 + 267 + 699) = 8,490 + 1,359 = $9,849 PV of terminal value = 9,849/1.1155 = $5,715.02 Estimated firm value = $628.79 + $5,715.02 = $6,343.81 Equity value = firm value − existing interest-bearing debt = $6,343.81 − (335 + 41 + 1,694) = $4,723.81 Accessibility: Keyboard Navigation Difficulty: 3 Hard Gradable: automatic

9-12 Copyright © 2018 McGraw-Hill Education. All rights reserved. No reproduction or distribution without the prior written consent of McGraw-Hill Education.


29. A recent annual income statement for Stone Creek Roofing is shown below.

Net sales Cost of sales Gross profit Operating expense Depreciation expense Operating income Interest expense Income before tax Tax

$5,000 3,200 1,800 800 200 800 100 700 175

Income after tax

$ 525

Assume that during the year, Stone Creek spent $180 on new capital equipment and increased current assets net of noninterest-bearing current liabilities by $120. What was Stone Creek’s free cash flow in this year?

A. $425 B. $500 C. $700 D. $725 E. $740 F. None of the options are correct. FCF = EBIT(1 − t) + Depreciation − Fixed investment − Working capital investment FCF = 800(1 − 0.25) + 200 − 180 − 120 = 500 Accessibility: Keyboard Navigation Difficulty: 2 Medium Gradable: automatic

30. STU Corporation has $3 million in earnings on $20 million in sales and has 1 million shares outstanding. Earnings per share of comparable firm 1 is $5, and earnings per share of comparable firm 2 is $2. Comparable firm 1’s stock is trading for $50, and comparable firm 2’s stock is trading for $28. What is the estimated stock price of STU using the method of comparables? (Use average multiples of the comparable firms when doing the calculations.) A. $33.43 B. $36.00 C. $39.00 D. $40.00 Comp. 1 P/E = 10, Comp. 2 P/E = 14, Avg. P/E = 12 STU = 12 × $3.00 (EPS) = $36.00 Accessibility: Keyboard Navigation Difficulty: 1 Easy Gradable: automatic

9-13 Copyright © 2018 McGraw-Hill Education. All rights reserved. No reproduction or distribution without the prior written consent of McGraw-Hill Education.


31. Tutter Corporation is being valued using discounted cash flow methodology with terminal value calculated as a growing perpetuity. Not including the terminal value, the present value of projected free cash flows for years 1 through 5 is $200 million (total). In year 5, projections show free cash flow of $60 million. What is the estimated fair market value of Tutter Corporation? Assume a WACC of 10% and a growth rate of 2%. A. $666 million B. $675 million C. $950 million D. $965 million FMV = PV{FCF, 1 − 5} + PV{Terminal value}. Terminal value = FCF(1 + g)/(KW − g ) = $61.2/0.08 = $765 million. PV of Terminal value = $765 million/1.115 = $475 million. FMV = 200 million + 475 million = $675 million. Accessibility: Keyboard Navigation Difficulty: 2 Medium Gradable: automatic

32. The following information is available about Chiantivino Corp. (CC):

Stock price per share Common shares outstanding (millions) Market value of interest-bearing debt (millions) Weighted-average cost of capital

$ $

8.00 10 75 14%

An activist investor is confident that by terminating CC’s money-losing fortified wine division, she can increase free cash flow by $4 million annually for the next decade. In addition, she estimates that an immediate, special dividend of $10 million can be financed by the sale of the division. a. Assuming these actions do not affect CC’s cost of capital, what is the maximum price per share the investor would be justified in bidding for control of CC? What percentage premium does this represent? b. Show your answer if you conduct a sensitivity analysis by assuming the cost of capital is 15 percent and the increased cash flow is only $3.5 million per year. a. The maximum justifiable premium = the fair market value of CC under new management − the fair market value of CC under existing management. A plausible estimate of CC’s fair market value under existing management is its standalone value = current market value of firm = $8 × 10 million + 75 million = $155 million. Fair market value under new management = $155 million + present value of enhancements = $155 million + present value of a $4 million annuity for 10 years at 14% + $10 million from sale of the division. Input:

10

14

?

4

0

n

i

PV

PMT

FV

Output:

−20.86

In Excel:

=PV(0.14,10,4) =−20.86

9-14 Copyright © 2018 McGraw-Hill Education. All rights reserved. No reproduction or distribution without the prior written consent of McGraw-Hill Education.


Fair market value = 155 million + 20.86 million + 10 million = $185.86 million. Fair market value of equity = $185.86 −75 = $110.86 million. Fair market of equity per share = $110.86/10 = $11.09. This is a 38.6% premium over the existing $8 share price. b. The fair market value of the firm assuming a 15 percent discount rate and a $3.5 million annuity = 155 + 17.57 + 10 = $182.57 million. Value of equity = 182.57 − 75 = 107.57. Value per share = 107.57/10 = $10.76. This is a 34.5% premium over the existing price. Input:

10

15

?

3.5

0

n

i

PV

PMT

FV

Output:

−17,57

In Excel:

=PV(0.15,10,3.5) =−17.57 Accessibility: Keyboard Navigation Difficulty: 2 Medium Gradable: manual

33. Below is a recent income statement for Gatlin Camera: Net sales Cost of sales (including depreciation of $800) Gross profit Selling and admin. expenses (including interest expense of $570) Income before tax Tax

$8,000 4,700 3,300 1,500 1,800 612

Income after tax

$1,188

Calculate Gatlin’s free cash flow in this year assuming it spent $510 on new capital equipment and increased current assets net of noninterest-bearing current liabilities by $340. Free cash flow = EBIT(1 − Tax rate) + Depreciation − Fixed investment − Working capital investment. EBIT = Income before tax + Interest = 1,800 + 570 = $2,370. Tax rate = 612/1,800 = 0.34 Free cash flow = 2,370(1 − 0.34) + 800 − 510 − 340 = $1,514.20. Accessibility: Keyboard Navigation Difficulty: 2 Medium Gradable: manual

9-15 Copyright © 2018 McGraw-Hill Education. All rights reserved. No reproduction or distribution without the prior written consent of McGraw-Hill Education.


34. Given the forecast below, estimate the fair market value of Kenmore Air at the end of 2017. Assume that after 2021, earnings before interest and tax will remain constant at $220 million, depreciation will equal capital expenditures in each year, and working capital will not change. Kenmore Air’s weighted-average cost of capital is 11 percent and its tax rate is 40 percent. Forecast for Kenmore Air, Inc. Year Free cash flow ($ millions)

2018 −40

2019 2020 2021 85 97 112

FMV = PV{FCF, 2018-2021} + PV{Terminal value}. Discounting the FCFs at an 11 percent cost of capital, PV{FCF, 2018-2021} = $177.7 million. Terminal value = EBIT(1 − Tax rate)/0.11 = $132/0.11 = $1,200 million. PV{Terminal value} = $1,200 million/(1 + 0.11)4 = $790.5 million. FMV = 177.7 million + 790.5 million = $968.2 million. Accessibility: Keyboard Navigation Difficulty: 2 Medium Gradable: manual

35. Given the forecast below, estimate the fair market value per share of Kenmore Air’s equity at the end of 2017 if the company has 50 million shares outstanding and the market value of its interest-bearing liabilities on the valuation date equals $300 million. Assume that after 2021, earnings before interest and tax will remain constant at $220 million, depreciation will equal capital expenditures in each year, and working capital will not change. Kenmore Air’s weightedaverage cost of capital is 11 percent and its tax rate is 40 percent. Forecast for Kenmore Air, Inc. Year Free cash flow ($ millions)

2018 −40

2019 2020 2021 85 97 112

FMV = PV{FCF, 2018-2021} + PV{Terminal value}. Discounting the FCFs at an 11 percent cost of capital, PV{FCF, 2018-2021} = $177.7 million. Terminal value = EBIT(1 − Tax rate)/0.11 = $132/0.11 = $1,200 million. PV{Terminal value} = $1,200 million/(1 + 0.11)4 = $790.5 million. FMV = 177.7 million + 790.5 million = $968.2 million. FMV of equity per share = ($968.2 − $300)/50 = $13.36. Accessibility: Keyboard Navigation Difficulty: 2 Medium Gradable: manual

36. Given the forecast below, estimate the fair market value of Kenmore Air’s equity per share at the end of 2017 under the following assumptions: EBIT in year 2021 is $210 million and then grows at 4 percent per year forever. To support the perpetual growth in EBIT, capital expenditures in year 2022 exceed depreciation by $25 million, and this difference grows 4 percent per year forever. Similarly, working capital investments are $10 million in 2022, and this amount grows 4 percent per year forever. Kenmore Air’s weighted-average cost of capital is 11 percent, and its tax rate is 40 percent. Kenmore Air has 50 million shares outstanding, and the market value of its interest-bearing liabilities on the valuation date equals $300 million. Forecast for Kenmore Air, Inc. Year Free cash flow ($ millions)

2018 −40

2019 2020 2021 85 97 112

9-16 Copyright © 2018 McGraw-Hill Education. All rights reserved. No reproduction or distribution without the prior written consent of McGraw-Hill Education.


FMV = PV{FCF, 2018-2021} + PV{Terminal value}. Discounting the FCFs at an 11 percent cost of capital, PV{FCF, 2018-2021} = $177.7 million. Terminal value = FCF in 2022/(0.11 − 0.04). FCF in 2022 = $210(1.04)(1 − 0.4) − 25 − 10 = $96.0 million. So, terminal value = $96/(0.11 − 0.04) = $1,371.4 million. PV{Terminal value} = $1,371.4/(1 + 0.11)4 = $903.4 million. FMV of company = $177.7 + $903.4 = $1,081.1 million. FMV of equity per share = ($1,081.1 − $300)/50 = $15.62. Accessibility: Keyboard Navigation Difficulty: 3 Hard Gradable: manual

37. Given the forecast below, estimate the fair market value of Kenmore Air’s equity per share at the end of 2017 under the following assumptions: EBIT in year 2021 will be $210 million. At year-end 2021, Kenmore Air has reached maturity, and analysts expect its equity will sell for 15 times year 2021 net income. At year-end 2021, Kenmore Air has $300 million book value of interest-bearing liabilities outstanding at an average interest rate of 10 percent. Kenmore Air’s weighted-average cost of capital is 11 percent, and its tax rate is 40 percent. Kenmore Air has 50 million shares outstanding, and the market value of its interest-bearing liabilities on the valuation date equals $300 million. Forecast for Kenmore Air, Inc. Year Free cash flow ($ millions)

2018 −40

2019 2020 2021 85 97 112

FMV = PV{FCF, 2018−2021} + PV{Terminal value}. Discounting the FCFs at an 11 percent cost of capital, PV{FCF, 2018−2021} = $177.7 million. Terminal value = Value of equity + Value of interest-bearing liabilities. Value of equity = 15 × Net income in 2021 = 15 × (210 − 0.10 × 300)(1 − 0.40) = $1,620 million. Terminal value = $1,620 + $300 = $1,920 million. PV{Terminal value} = $1,920/(1 + 0.11)4 = $1,264.8 million. FMV of company on valuation date = $177.7 + $1,264.8 = $1,442.5 million. FMV of equity per share = ($1,442.5 − $300)/50 = $22.85. Accessibility: Keyboard Navigation Difficulty: 3 Hard Gradable: manual

9-17 Copyright © 2018 McGraw-Hill Education. All rights reserved. No reproduction or distribution without the prior written consent of McGraw-Hill Education.


38. Ketema, Inc. is a manufacturer of electronic instruments. Use the following information on Ketema and five other similar companies to value Ketema, Inc. on December 31, 2017. Ketema, Inc., 2017 ($ millions) Net income Number of common share (millions) Earnings before interest and tax Tax rate Book value of equity Book value of interest-bearing debt

$ $ $ $

66.10 32.82 112.30 22% 335.10 470.80

Comparison of Ketema, Inc. with Comparable Companies Aloha Ketema Electric Growth Rates, Financial Risks, Size, Returns 5-year growth rate in sales (%) 3.3 6.8 5-year growth rate in eps (%) 5.1 1.3 Interest coverage ratio (X) 4.8 8.0 Total liabilities to assets (X) 0.7 0.6 Total assets ($ millions) 1,029 15,046 Indicators of Value Price/earnings (X) MV firm/EBIT(1−Tax rate) (X) MV equity/BV equity (X) MV firm/BV firm (X) Price/sales (X) MV firm/sales (X)

23.2 23.3 3.9 2.7 1.5 1.9

Butler Electric

Baker National AJ Smith Modesto Power

1.4 9.2 38.0 0.4 196

8.1 (5.4) 3.0 0.7 1,294

10.2 19.6 12.3 0.5 585

15.2 3.4 * 0.1 1,421

16.1 16.2 3.5 3.3 1.4 1.4

32.0 34.7 1.0 1.0 0.4 0.8

12.4 13.2 2.1 1.8 1.0 1.1

25.0 25.0 2.3 2.3 2.0 2.0

*National Power has no interest-bearing debt outstanding. MV = Market value; BV = Book value. MV firm is estimated as book value of interest-bearing debt + market value of equity. The median and mean values for Ketema’s peers are presented below:

5-year growth rate in sales (%) 5-year growth rate in eps (%) Interest coverage ratio (X) Total liabilities to assets (X) Total assets ($ millions)

(Excluding Ketema) Median Mean 8.1 8.3 3.4 5.6 10.1 15.3 0.5 0.4 1,294 3,708

Price/earnings (X) MV firm/EBIT(1−Tax rate) (X) MV equity/BV equity (X) MV firm/BV firm (X) Price/sales (X) MV firm/sales (X)

23.2 23.3 2.3 2.3 1.4 1.4

21.7 22.5 2.6 2.2 1.3 1.4

9-18 Copyright © 2018 McGraw-Hill Education. All rights reserved. No reproduction or distribution without the prior written consent of McGraw-Hill Education.


The following estimates require subjective reasoning. In coming to these estimates, Ketema, Inc. is judged as exhibiting representative earnings per share growth, but considerably higher financial leverage, and a below-average five-year growth rate in sales. The company’s higher-than-average leverage suggests that its firm value ratios will be particularly key, as equity value ratios can be distorted by Ketema’s higher leverage. Since the firm value ratios abstract from differences in financing, values for these ratios are selected that are closer to the sample averages. Turning to equity value ratios, Ketema’s modest growth and higher financial leverage suggest a 10 to 20 percent discount from the group average for its price/earnings and price/sales indicators. Here are estimated indicators of value for Ketema, Inc. Estimated Indicators for Ketema, Inc. Price/earnings (X) MV firm/EBIT(1−Tax rate) (X) Price/sales (X) MV firm/sales (X) MV equity/BV equity MV firm/BV firm

18.0 20.0 1.1 1.3 2.7 2.1

And here are associated implied values of Ketema stock:

Price/earnings (X) MV firm/EBIT(1−Tax rate) (X) Price/sales (X) MV firm/sales (X) MV equity/BV equity MV firm/BV firm

Implied Value of Ketema Stock $36.26 = 18 × Net income / # shares $39.26 = [20 × EBIT(1 − Tax rate) − Debt] / # shares $34.35 = 1.1 × Sales / # shares $26.25 = (1.3 × Sales − Debt) / # shares $27.57 = 2.7 × BV equity / # shares $37.23 = (2.1 × BV firm - Debt) / # shares

Based on these numbers, a reasonable estimate of a fair price for Ketema, Inc.’s shares at year-end 2017 is $36.00. Many other estimates are, of course, possible. Accessibility: Keyboard Navigation Difficulty: 3 Hard Gradable: manual

39. Rainy City Coffee’s (RCC) free cash flow next year will be $100 million, and it is expected to grow at a 4 percent annual rate indefinitely. The company’s weighted average cost of capital is 10 percent, the market value of its liabilities is $1 billion, and it has 20 million shares outstanding. A. Estimate the price per share of RCC’s common stock. b. A hedge fund believes that by selling the company’s private jet and instituting other cost savings, it can increase RCC’s free cash flow next year to $110 million and can add a full percentage point to RCC’s growth rate without affecting its cost of capital. What is the maximum price per share the hedge fund can justify bidding for control of RCC? a. Price per share of RCC stock = (100/(0.10 − 0.04) − 1,000)/20 = $33.33. b. Fair market value of equity per share after change in ownership = (110/(0.10 − 0.05) − 1,000)/20 = $60.00. Accessibility: Keyboard Navigation Difficulty: 2 Medium Gradable: manual

9-19 Copyright © 2018 McGraw-Hill Education. All rights reserved. No reproduction or distribution without the prior written consent of McGraw-Hill Education.


40. Empirical evidence indicates that the returns to shareholders of the target firm vary significantly from the returns to the shareholders of the acquiring firm. Identify the shareholders that tend to realize the smaller return. Does your answer depend on the way the acquisition is financed? The empirical evidence strongly indicates that the shareholders of the target firm realize large wealth gains (premiums of 20 to 40 percent) as a result of a takeover bid but the shareholders in the acquiring firm gain little or nothing. While a definitive answer is elusive, the following have been offered as possible explanations for these low returns to acquiring shareholders: size differentials, competition in the takeover market, lack of achieving expected synergy gains, management goals other than the best interests of the shareholders, and early announcements of corporate acquisition intent. The empirical evidence also suggests that leveraged buyouts lead to sizeable improvements in operating performance and attractive returns to buyers. Hence, financing policy does appear to change the outcome for buyers. Accessibility: Keyboard Navigation Difficulty: 1 Easy Gradable: manual

41. Estimate a current stock price for Montana Corporation using the following information. Projected free cash flows for the next three years are shown in the table below. Assume the growth rate in years 1 through 3 continues into the future. Calculate terminal value as a growing perpetuity. Montana’s WACC is 18%, the market value of its debt is $5 million, and it has 1.2 million shares of stock outstanding. Year Free cash flow

1 $1,000,000

2 $1,050,000

3 $1,102,500

PV of cash flows, years 1 to 3 = 1,000,000/1.18 + 1,050,000/1.182 + 1,102,500/1.183 = $2,272,567 Terminal value = 1,102,500(1.05)/(0.18 − 0.05) = $8,904,808 PV(Terminal value) = 8,904,808/1.183 = $5,419,741 Firm value = 2,272,567 + 5,419,741 = $7,692,308 Equity value = 7,692,308 − 5,000,000 = $2,692,308 Stock price = 2,692,308/1,200,000 = $2.24 Accessibility: Keyboard Navigation Difficulty: 2 Medium Gradable: manual

42. A venture capital firm wants to invest $5 million in Artichoke Corp., a startup biotech firm. Artichoke is expected to go public in 4 years. Earnings will be negligible until year 4 but are projected to be $4 million in year 4. Comparable biotech firms are trading at P/E ratios of 18 on average. Artichoke has 3 million shares of stock outstanding. The VC firm will apply a discount rate of 40% to the investment. How many shares of stock should the VC firm be given for its $5 million investment? Terminal value = $4 million × 18 = $72 million PV(Terminal value) = $72 million/1.44 = $18,742,191 Required ownership percentage = 5,000,000/18,742,191 = 26.7% Number of new shares = (New owners% / existing owners%) × current shares =(0.267/0.733) × 3,000,000 =1,092,769 Accessibility: Keyboard Navigation Difficulty: 2 Medium Gradable: manual

9-20 Copyright © 2018 McGraw-Hill Education. All rights reserved. No reproduction or distribution without the prior written consent of McGraw-Hill Education.


Chapter 09 Test Bank Summary Category Accessibility: Keyboard Navigation Difficulty: 1 Easy Difficulty: 2 Medium Difficulty: 3 Hard Gradable: automatic Gradable: manual

# of Questions 42 15 17 10 31 11

9-21 Copyright © 2018 McGraw-Hill Education. All rights reserved. No reproduction or distribution without the prior written consent of McGraw-Hill Education.


Turn static files into dynamic content formats.

Create a flipbook
Issuu converts static files into: digital portfolios, online yearbooks, online catalogs, digital photo albums and more. Sign up and create your flipbook.